You are on page 1of 291

Tutorials, Past Test and Examination Booklets in Mathematics 2021

School of Mathematics
University of the Witwatersrand

CALCULUS (MATH1036A)
Tutorial Questions: Semester One
and
Past Test and Examination Questions (2012-2020)
with Selected Solutions

THE WIT
F W
O
Y A
IT

T
E
S

RS
UNIVER

RAND

J G
O R
HA
N N E SBU

MATH I NAME STUDENT NUMBER


This book
belongs to
Mathematics I (Major) 2012–2020 Past Test and Examination Questions Booklet i

Contents
1 MATH1036 Tutorial Questions: Semester One 1
2 MATH1036 Selected Tutorial Answers: Semester One 24
3 Calculus Tests and Examinations 34
3.1 Calculus 2013 . . . . . . . . . . . . . . . . . . . . . . . . . . . . . . . . . . . . . . . . . . . . . 34
3.1.1 Calculus March 2013 . . . . . . . . . . . . . . . . . . . . . . . . . . . . . . . . . . . . . 34
3.1.2 Calculus April 2013 . . . . . . . . . . . . . . . . . . . . . . . . . . . . . . . . . . . . . 36
3.1.3 Calculus June 2013 . . . . . . . . . . . . . . . . . . . . . . . . . . . . . . . . . . . . . . 39
3.1.4 Calculus August 2013 . . . . . . . . . . . . . . . . . . . . . . . . . . . . . . . . . . . . 41
3.1.5 Calculus October 2013 . . . . . . . . . . . . . . . . . . . . . . . . . . . . . . . . . . . . 43
3.2 Calculus 2014 . . . . . . . . . . . . . . . . . . . . . . . . . . . . . . . . . . . . . . . . . . . . . 46
3.2.1 Calculus March 2014 . . . . . . . . . . . . . . . . . . . . . . . . . . . . . . . . . . . . . 46
3.2.2 Calculus April 2014 . . . . . . . . . . . . . . . . . . . . . . . . . . . . . . . . . . . . . 50
3.2.3 Calculus June 2014 . . . . . . . . . . . . . . . . . . . . . . . . . . . . . . . . . . . . . . 53
3.2.4 Calculus September 2014 . . . . . . . . . . . . . . . . . . . . . . . . . . . . . . . . . . 54
3.2.5 Calculus November 2014 . . . . . . . . . . . . . . . . . . . . . . . . . . . . . . . . . . . 56
3.3 Calculus 2015 . . . . . . . . . . . . . . . . . . . . . . . . . . . . . . . . . . . . . . . . . . . . . 60
3.3.1 Calculus March 2015 . . . . . . . . . . . . . . . . . . . . . . . . . . . . . . . . . . . . . 60
3.3.2 Calculus April 2015 . . . . . . . . . . . . . . . . . . . . . . . . . . . . . . . . . . . . . 62
3.3.3 Calculus June 2015 . . . . . . . . . . . . . . . . . . . . . . . . . . . . . . . . . . . . . . 64
3.3.4 Calculus September 2015 . . . . . . . . . . . . . . . . . . . . . . . . . . . . . . . . . . 66
3.3.5 Calculus November 2015 . . . . . . . . . . . . . . . . . . . . . . . . . . . . . . . . . . . 68
3.4 Calculus 2016 . . . . . . . . . . . . . . . . . . . . . . . . . . . . . . . . . . . . . . . . . . . . . 71
3.4.1 Calculus March 2016 . . . . . . . . . . . . . . . . . . . . . . . . . . . . . . . . . . . . . 71
3.4.2 Calculus April 2016 . . . . . . . . . . . . . . . . . . . . . . . . . . . . . . . . . . . . . 72
3.4.3 Calculus June 2016 . . . . . . . . . . . . . . . . . . . . . . . . . . . . . . . . . . . . . . 74
3.4.4 Calculus August 2016 . . . . . . . . . . . . . . . . . . . . . . . . . . . . . . . . . . . . 75
3.4.5 Calculus November 2016 . . . . . . . . . . . . . . . . . . . . . . . . . . . . . . . . . . . 77
3.5 Calculus 2017 . . . . . . . . . . . . . . . . . . . . . . . . . . . . . . . . . . . . . . . . . . . . . 80
3.5.1 Calculus March 2017 . . . . . . . . . . . . . . . . . . . . . . . . . . . . . . . . . . . . . 80
3.5.2 Calculus April 2017 . . . . . . . . . . . . . . . . . . . . . . . . . . . . . . . . . . . . . 80
3.5.3 Calculus June 2017 . . . . . . . . . . . . . . . . . . . . . . . . . . . . . . . . . . . . . . 82
3.5.4 Calculus August 2017 . . . . . . . . . . . . . . . . . . . . . . . . . . . . . . . . . . . . 84
3.5.5 Calculus November 2017 . . . . . . . . . . . . . . . . . . . . . . . . . . . . . . . . . . . 85
3.5.6 Calculus Diagonal D March 2018 . . . . . . . . . . . . . . . . . . . . . . . . . . . . . . 90
3.5.7 Calculus Diagonal E March 2018 . . . . . . . . . . . . . . . . . . . . . . . . . . . . . . 92
3.5.8 Calculus Diagonal D April 2018 . . . . . . . . . . . . . . . . . . . . . . . . . . . . . . . 93
3.5.9 Calculus Diagonal E April 2018 . . . . . . . . . . . . . . . . . . . . . . . . . . . . . . . 94
3.5.10 Calculus June 2018 . . . . . . . . . . . . . . . . . . . . . . . . . . . . . . . . . . . . . . 95
3.5.11 Calculus Diagonal D August 2018 . . . . . . . . . . . . . . . . . . . . . . . . . . . . . . 97
3.5.12 Calculus Diagonal E August 2018 . . . . . . . . . . . . . . . . . . . . . . . . . . . . . . 99
3.5.13 Calculus November 2018 . . . . . . . . . . . . . . . . . . . . . . . . . . . . . . . . . . . 1
4 Calculus Solutions 4
ii 2012–2020 Past Test and Examination Questions Booklet Mathematics I (Major)

4.1 Calculus Solutions 2013 . . . . . . . . . . . . . . . . . . . . . . . . . . . . . . . . . . . . . . . 4


4.1.1 Calculus Solutions April 2013 . . . . . . . . . . . . . . . . . . . . . . . . . . . . . . . . 4
4.1.2 Calculus Solutions June 2013 . . . . . . . . . . . . . . . . . . . . . . . . . . . . . . . . 5
4.1.3 Calculus Solutions June 2013 . . . . . . . . . . . . . . . . . . . . . . . . . . . . . . . . 5
4.1.4 Calculus Solutions August 2013 . . . . . . . . . . . . . . . . . . . . . . . . . . . . . . . 10
4.2 Calculus Solutions 2014 . . . . . . . . . . . . . . . . . . . . . . . . . . . . . . . . . . . . . . . 12
4.2.1 Calculus Solutions March 2014 . . . . . . . . . . . . . . . . . . . . . . . . . . . . . . . 12
4.2.2 Calculus Solutions April 2014 . . . . . . . . . . . . . . . . . . . . . . . . . . . . . . . . 17
4.2.3 Calculus Solutions June 2014 . . . . . . . . . . . . . . . . . . . . . . . . . . . . . . . . 19
4.2.4 Calculus Solutions September 2014 . . . . . . . . . . . . . . . . . . . . . . . . . . . . . 22
4.2.5 Calculus Solutions November 2014 . . . . . . . . . . . . . . . . . . . . . . . . . . . . . 25
4.3 Calculus Solutions 2015 . . . . . . . . . . . . . . . . . . . . . . . . . . . . . . . . . . . . . . . 31
4.3.1 Calculus Solutions March 2015 . . . . . . . . . . . . . . . . . . . . . . . . . . . . . . . 31
4.3.2 Calculus Solutions April 2015 . . . . . . . . . . . . . . . . . . . . . . . . . . . . . . . . 36
4.3.3 Calculus Solutions June 2015 . . . . . . . . . . . . . . . . . . . . . . . . . . . . . . . . 38
4.3.4 Calculus Solutions September 2015 . . . . . . . . . . . . . . . . . . . . . . . . . . . . . 40
4.3.5 Calculus Solutions November 2015 . . . . . . . . . . . . . . . . . . . . . . . . . . . . . 43
4.4 Calculus Solutions 2016 . . . . . . . . . . . . . . . . . . . . . . . . . . . . . . . . . . . . . . . 52
4.4.1 Calculus Solutions March 2016 . . . . . . . . . . . . . . . . . . . . . . . . . . . . . . . 52
4.4.2 Calculus Solutions April 2016 . . . . . . . . . . . . . . . . . . . . . . . . . . . . . . . . 56
4.4.3 Calculus Solutions June 2016 . . . . . . . . . . . . . . . . . . . . . . . . . . . . . . . . 62
4.4.4 Calculus Solutions August 2016 . . . . . . . . . . . . . . . . . . . . . . . . . . . . . . . 65
4.4.5 Calculus Solutions November 2016 . . . . . . . . . . . . . . . . . . . . . . . . . . . . . 70
4.5 Calculus Solutions 2017 . . . . . . . . . . . . . . . . . . . . . . . . . . . . . . . . . . . . . . . 83
4.5.1 Calculus Solutions March 2017 . . . . . . . . . . . . . . . . . . . . . . . . . . . . . . . 83
4.5.2 Calculus Solutions April 2017 . . . . . . . . . . . . . . . . . . . . . . . . . . . . . . . . 86
4.5.3 Calculus Solutions June 2017 . . . . . . . . . . . . . . . . . . . . . . . . . . . . . . . . 89
4.5.4 Calculus Solutions August 2017 . . . . . . . . . . . . . . . . . . . . . . . . . . . . . . . 93
4.5.5 Calculus Solutions November 2017 . . . . . . . . . . . . . . . . . . . . . . . . . . . . . 97
4.6 Calculus Solutions 2018 . . . . . . . . . . . . . . . . . . . . . . . . . . . . . . . . . . . . . . . 103
4.6.1 Diagonal D Calculus Solutions March 2018 . . . . . . . . . . . . . . . . . . . . . . . . . 103
4.6.2 Diagonal E Calculus Solutions March 2018 . . . . . . . . . . . . . . . . . . . . . . . . . 109
4.6.3 Calculus Diagonal D Solutions April 2018 . . . . . . . . . . . . . . . . . . . . . . . . . 113
4.6.4 Calculus Diagonal E Solutions April 2018 . . . . . . . . . . . . . . . . . . . . . . . . . . 116
4.6.5 Calculus Solutions June 2018 . . . . . . . . . . . . . . . . . . . . . . . . . . . . . . . . 117
4.6.6 Calculus Solutions August 2018 . . . . . . . . . . . . . . . . . . . . . . . . . . . . . . . 123
4.6.7 Calculus Solutions November 2018 . . . . . . . . . . . . . . . . . . . . . . . . . . . . . 123
4.7 Calculus Solutions 2019 . . . . . . . . . . . . . . . . . . . . . . . . . . . . . . . . . . . . . . . 130
4.7.1 Calculus Solutions March (D diagonal) 2019 . . . . . . . . . . . . . . . . . . . . . . . . 130
4.7.2 Calculus Solutions April (E diagonal) 2019 . . . . . . . . . . . . . . . . . . . . . . . . . 134
4.7.3 Calculus Solutions June 2019 . . . . . . . . . . . . . . . . . . . . . . . . . . . . . . . . 138
4.7.4 Calculus Solutions August (D diagonal) 2019 . . . . . . . . . . . . . . . . . . . . . . . . 141
4.7.5 Calculus Solutions November 2019 . . . . . . . . . . . . . . . . . . . . . . . . . . . . . 143
4.8 Calculus Solutions 2012 . . . . . . . . . . . . . . . . . . . . . . . . . . . . . . . . . . . . . . . 154
Mathematics I (Major) 2012–2020 Past Test and Examination Questions Booklet iii

4.8.1 Calculus Solutions March 2012 . . . . . . . . . . . . . . . . . . . . . . . . . . . . . . . 154


4.8.2 Calculus Solutions May 2012 . . . . . . . . . . . . . . . . . . . . . . . . . . . . . . . . 156
4.8.3 Calculus Solutions June 2012 . . . . . . . . . . . . . . . . . . . . . . . . . . . . . . . . 157
4.8.4 Calculus Solutions August 2012 . . . . . . . . . . . . . . . . . . . . . . . . . . . . . . . 159
4.9 Calculus Solutions 2020 . . . . . . . . . . . . . . . . . . . . . . . . . . . . . . . . . . . . . . . 160
4.9.1 Calculus Solutions March (D diagonal) 2020 . . . . . . . . . . . . . . . . . . . . . . . . 160
4.9.2 Calculus Solutions March (E diagonal) 2020 . . . . . . . . . . . . . . . . . . . . . . . . 165
4.9.3 Calculus Solutions May 2020 . . . . . . . . . . . . . . . . . . . . . . . . . . . . . . . . 170
4.9.4 Calculus Solutions June 2020 . . . . . . . . . . . . . . . . . . . . . . . . . . . . . . . . 174
4.9.5 Calculus Solutions August 2020 . . . . . . . . . . . . . . . . . . . . . . . . . . . . . . . 181
4.9.6 Calculus Solutions November Exam 2020 . . . . . . . . . . . . . . . . . . . . . . . . . . 183
Mathematics I (Major) 2012–2020 Past Test and Examination Questions Booklet 1

1 MATH1036 Tutorial Questions: Semester One


In some tutorials, you will see that there are some references to questions in the textbook by Thomas. These are
extra for those students who feel they would like some more practise. Also feel free to find your own questions in
the recommended textbooks. In many textbooks, only odd numbered questions have answers provided.

Tutorial 0.1.1

For this very first tutorial, don’t worry if there is something you haven’t seen before. The important thing it that
you learn what everything means and understand the questions and answers before moving on.

The questions in this tutorial are not based on any particular section of your study guide, but are a collection of
things that new first years often do not understand fully. It will really benefit you to do this tutorial very thoroughly.

Remember: To learn you need to makes mistakes.

1. TRUE or FALSE:

(a) π = 22
7

2
(b) x = x ∀ x in R
(c) It is possible to draw a function which intersects its asymptote.
(d) loge ea = a = eloge a
−2x
(e) If x is negative then √
3|x|
is positive.
x2 − 4
(f) x − 2 = ∀x ∈ R
x+2
2. What is the difference in meaning between the symbol ∀ and the symbol ∃?
2 2012–2020 Past Test and Examination Questions Booklet Mathematics I (Major)

3. If dxe denotes the smallest integer larger or equal to x then dπe =

4. How many numbers are there between 2 and 4?


1 1 1
5. If cosec x := , sec x := and cot x := then express cosec2 x in terms of cot2 x and express
sin x cos x tan x
sec2 x in terms of tan2 x. HINT: Recall that sin2 x + cos2 x = 1.

6. If A ⇒ B means “if A then B” or “A implies B”, then what do you think the following expressions mean:

(a) A ⇐ B
(b) A ⇔ B

7. Calculate the final mark if a student obtains the following results during the year: (See the preamble in your
study guide to see the two different ways the mark is calculated and choose the higher mark.)
Mark Total
March test 13 40
April test 25 40
June test 15 40
September test 20 30
November exam 62 90
Tut tests and assignments 28 30

Tutorial 1.1.1

1. Thomas Exercises 11th ed. 2.1 pp. 75–76: 1, 2, 4, 7, 8 or 12th ed. 2.2 p. 73: 1, 2, 4, 7, 8.
2. If f is a function such that f (2) = 4, can you conclude anything about lim f (x)? Explain your answer and
x→2
provide a graph for f to substantiate.
3. If g is a function such that lim g(x) = 4, can you conclude anything about g(2)? Explain your answer and
x→2
provide a graph for g to substantiate.
4. Let f (x) = x − bxc. For each integer n, find lim− f (x) and lim+ f (x) if they exist.
x→n x→n

2
5. (a) Estimate the value of lim t t+9−3
2 .
t→0
Hint: Use a table for

several values of t near 0. Also, check for values of t made sufficiently small (i.e. ±0.0001; ±0.00001; ±0.000001
2
(b) Evaluate lim t t+9−32 .
t→0
Hint: You can’t apply the quotient law immediately, first rationalize the numerator.

6. Explain what it means to say that lim− f (x) = 1 and lim+ f (x) = 5.
x→2 x→2
In this situation is it possible that lim f (x) exists?. Explain.
t→1

7. Sketch the graph of the function f (x) and use it to determine the values of a for which lim f (x) exists.
x→a

 1 + x, if x < −1 
 
 
f (x) = x2 , if − 1 ≤ x < 1
2 − x, if x ≥ 1

 

8. Sketch the graph of an example of a function f that satisfies all the given conditions.
lim f (x) = 1, lim+ f (x) = −1, lim− f (x) = 0, lim+ f (x) = 1, f (2) = 1, f (0) is undefined .
x→0− x→0 x→2 x→2
Mathematics I (Major) 2012–2020 Past Test and Examination Questions Booklet 3

Tutorial 1.1.2

1. Thomas Exercises 11th ed. 2.5 pp. 117–118: 1, 2, 6, 11, 13, 16, 21, 42 or 12th ed. 2.6 pp. 114-115: 37, 38, 42,
47, 49, 52, 57, 72.

2. If ( √
x − 4, if x > 4
)
f (x) =
8 − 2x, if x < 4

Determine whether lim f (x) exists.


x→4

1 1 1
3. Determine lim− 3 and lim+ 3 by evaluating f (x) = (x3 −1)
for values of x that approach 1 from the left and
x→1 x −1 x→1 x −1
from the right.

2−x
4. Determine the limit of lim 2, if it exists.
x→1 (x−1)

5. Find limπ + 1x sec x


x→( 2 )

6. Evaluate lim− x cos ecx


x→2π

x2 −2x−8
Q.7 Determine the limit of lim+ 2
x→2 x −5x+6

x2 +1
Q.8 Find the vertical asymptotes of the function y = 3x−2x2

Tutorial 1.2.1

1. Thomas Exercises 11th ed. 2.2 pp. 83–85: 1, 3, 4, 5, 10, 15, 16, 20, 28, 31, 36, 39, 42, 55, 58 or 12th ed. 2.2
pp. 74–76: 11–14, 18, 21, 22, 24, 34, 37, 42, 53, 56, 79, 82.

2. Given that lim f (x) = 2, lim g(x) = 0 and lim h(x) = −3 find the limits that exist. If the limit does not exist,
x→a x→a x→a
explain why.

a. lim[ f (x) − h(x)], b. lim( f (x))3 , c. lim 3 2 f (x) − 4h(x),


p
x→a x→a x→a
g(x) f (x)
d. lim , e. lim .
x→a h(x) x→a g(x)

3. Find the limit, if it exists.


1 1
√ √
3 + x x2 + 4 − 5 1 1
Å ã
x3 +8
a. lim , b. lim √ , c. lim √ − . d. lim x+2 Hint: Sum and difference of two
x→−3 3 + x x→1 x+3−2 x→0 x 1+x x x→−2
cubes.
x2 − 1
4. Find the limit of lim √ √
x→1 2x + 1 − 3
5. If you are given that lim g(x) = π1 , what can you say about g(e)?
x→e
4 2012–2020 Past Test and Examination Questions Booklet Mathematics I (Major)

6. If you are given that lim g(x) = π1 , what can you say about lim− g(x)?
x→e x→e

7. If you are given that f (−5) D.N.E., what can you say about lim f (x)?
x→−5

8. Evaluate the limits and justify each step by indicating the appropriate Limit Law(s).
4
(a) lim 2t2t−3t+2
−2
.
t→−2
Hint: Use√theorems 4,1,3,8 and 10.
(b) lim u4 + 3u + 6.
u→−2
Hint: Use theorems
√  5,1,2,3,10,8 and 9.
(c) lim 1 + 3 x (2 − 6x2 + x3 )
x→8
Hint: Use theorems 4,1,2,3,8,11 and 10.

ä2
t2 −2
Ä
9. Evaluate the limit lim t3 −3t+5
.
t→2
Hint: Use theorems 7,5,1,2,3,10,8 and 9.

»
2x2 +1
10. Find lim 3x−2 .
x→2
Hint: Use theorems 12,5,1,2,3,10,9 and 8.


x2 +9−5
11. Evaluate the limit, if it exists. lim x+4 .
x→−4

1
−1
(x+h2 ) x2
12. Evaluate lim h .
h→0

Tutorial 1.3.1

1. Thomas Exercises 11th ed. 2.4 p. 108: 52, 58, 59, 63, 64, 65 or 12th ed. 2.6 p. 114: 14, 20, 21, 29, 30, 31.
2. Thomas Exercises 12th ed. 2.6 p. 116: 80, 82, 85.
3
3. Find the limit of lim 3/2
x→−∞ x

x3 + πx
4. Find the limit of lim
x→∞ (x + 1)2

2x
5. Find the limit of lim
x→−∞ 3 x

6. Determine whether the following function have vertical, horizontal or oblique asymptote:
a. y = 2x−3
x−2
Hint: use the one-sided limits and the infinity limit
x2
b. y = x−1
Hint: Remember to use long division.

3
x−5x+3
7. Find the limit as x → −∞ of f (x) = √3
2x+ x2 −4

8. Evaluate lim √ x−3


x→∞ 4x2 +25
4−3x3
9. Determine the limit of lim √
x→−∞ x6 +9
Mathematics I (Major) 2012–2020 Past Test and Examination Questions Booklet 5

Tutorial 1.4.1

1. Thomas Exercises 11th ed. 2.4 p. 108–109: 12–15, 17–20, 82 or 12th ed. 2.4 pp. 91–92: 12–15, 17–20, 52.
2. For the following functions f (x), determine if the one-sided limits at the indicated point a exist, find whether
the limit at a exists, and compare with the value of f (a) if f is defined at a.
1 1 x2 − 1
a. f (x) = − at a = 0, b. f (x) = at a = 1, c. f (x) = bxc + b−xc at a = −3.
x |x| |x − 1|
3. Find the limit of lim 2x+12
|x+6| , if it exists. If the limit does not exist, explain why.
x→−6
x2 +x−6
4. Let g(x) = |x−2|
(a) Find (i) lim+ g(x) and (ii) lim− g(x).
x→2 x→2
(b) Does lim g(x) exist?
x→2
5. Let
x, if x < 1 
 

 
 3,

if x = 1 

f (x) =

 2 − x2 , if 1 < x ≤ 2 

 
x − 3, if x > 2
 

Evaluate each of the following, if it exists. (a) lim− f (x); (b)lim f (x); (c) f (1); (d) lim− f (x); (e) lim+ f (x); (f)lim f (x).
x→1 x→1 x→2 x→2 x→2
6. Find the limit of lim 2x−1 3 2
x→0.5− |2x −x |

Tutorial 1.5.1

1. Thomas Exercises 11th ed. 2.2 p. 84: 49, 50, 51a, 52a or 12th ed. 2.2 p. 75: 63, 64, 65a, 66a.

2. Thomas Exercises 11th ed. 2.4 p. 108: 22, 25, 28, 29, 31, 33, 36 or 12th ed. 2.4 p. 92: 22, 25, 28, 29, 31, 33,
35, 40.

3. Thomas Chapter 2 Additional and Advanced Exercises 11th ed. p. 143: 27–30 or 12th ed. p. 121: 27–30.

x if x is rational,
(
4. Let f (x) = Show that lim f (x) = 0.
0 if x is irrational. x→0

5. Use the Squeeze Theorem to show that lim x2 cos 20πx = 0. Consider f (x) = −x2 , g(x) = x2 cos 20πx and

x→0
h(x) = x2 .
√ √ √
6. Use the Squeeze Theorem to show that lim x3 + x2 sin πx = 0. Consider f (x) = − x3 + x2 , g(x) = x3 + x2 sin πx
√ x→0
and h(x) = x3 + x2 .

7. If 4x − 9 ≤ f (x) ≤ x2 − 4x + 7 for x ≥ 0, find lim f (x).


x→4

8. Prove that lim x4 cos 2x = 0.


x→0

9. Prove that lim+ x [1 + sin2 2π
] = 0.

x→0 x

Tutorial 2.1.1
6 2012–2020 Past Test and Examination Questions Booklet Mathematics I (Major)

1. Find the values of a and b which make the function




 x−1 if x ≤ −2,

f (x) = ax2 + b if − 2 < x < 1,

x+1 if x ≥ 1,

continuous at x = −2 and x = 1.
2. Find the values of a and b which make the function f continuous everywhere.
 2
x −4

 x−2 ,
 if x < 2 

f (x) = ax2 − bx + 3, if 2 ≤ x < 3
 
2x − a + b, if x ≥ 3
 

3. Show that f is continuous on (−∞, ∞).

1 − x2 , if x ≤ 1
( )
f (x) = √
x − 1, if x > 1

4. Show that f is continuous on (−∞, ∞).


π
sin x, if x <
® ´
4
f (x) = π
cos x, if x ≥ 4

x2 −3x+2
5. How would you ”remove the discontinuity” of f in order to make the function continuous at 2? f (x) = x2 +x−6

Tutorial 2.2.1

1. Consider the function


 bxc

if x , 0,
f (x) = x
−1 if x = 0.
Investigate continuity from the left and the right at x = 0, x = π and x = 1.

2. Use the definition of continuity and the properties of limits to show that the function is continuous at a given
point a. √
(a) f (x) = x2 + 7 − x,√ a = 4.
3
(b) f (x) = 3x4 − 5x + x2 + 4, a = 2.

3. At which points is the function f continuous from the the right, from the left, or neither?
 2
 x , if x < −1 

 
f (x) = x, if − 1 ≤ x < 1
 1
if x ≥ 1
 
x,

4. For what value of the constant c is the function continuous on (−∞, ∞).
( 2
cx + 2x, if x < 2
)
f (x) =
x3 − cx, if x ≥ 2

5. If f is continuous at 5 and f (5) = 2 and f (4) = 3, then lim f (4x2 − 11) = 2.


x→2
TRUE or FALSE. Give reasons.
Mathematics I (Major) 2012–2020 Past Test and Examination Questions Booklet 7

Tutorial 2.3.1

1. The function g(x) is continuous at x = − 12 iff...

2. The function f (x) is continuous on (−3, e] iff...

3. Show that the √ function is continuous on the given interval.


(a) f (x) = x + x − 4, (4, ∞).
x−1
(b) g(x) = , (−∞, −2).
3x + 6
4. Why is the function continuous at every point in its domain. State the domain.
2
(a) f (x) = 2xx2−x−1
+1
√3
(x−2)
(b) h(x) = x3 −2
(c) g(x) = √tan x
4−x2
5. Show that√ the function is continuous on its domain. State the domain.
(a) f (x) = √4 x + x3 cos x.
2
(b) g(x) = xx2 −2−9 .

Tutorial 2.4.1

1. Thomas Exercises 11th ed. 2.6 pp. 129–130: 13–20, 23–33, 40, 47–50, 53–57, 59 or 12th ed. 2.5 pp. 101–103:
13–20, 23–35, 47, 55–58, 61–64, 65, 67.
2. Thomas Chapter 2 Additional and Advanced Exercises 11th ed. p. 142: 19 or 12th ed. p. 120: 19.
3. The equation x3 − 12x + 8 = 0 has three ANSWERS. Find intervals between successive integers that contain
these ANSWERS.
4. Use the Intermediate Value Theorem to show that there is a root of the given equation in the specified interval.
(a) x4 + x − 3 = 0, (1, 2)
(b) sin x = x2 − x, (1, 2)
(c) 2 sin x = 3 − 2x, (0, 1)

5. If f is continuous on [-1,1] and f (−1) = 4 and f (1) = 3, then there exists a number r such that |r| < 1 and
f (r) = π.
6. If f (x) = x2 + 10 sin x, show that there is a number c such that f (c) = 1000.
7. Suppose f is continuous on [1,5] and the only ANSWERS of the equation f (x) = 6 are x = 1 and x = 4. If
f (2) = 8, explain why f (3) > 6.

Tutorial 2.5.1

1. Thomas Exercises 11th ed. 2.6 p. 130: 51, 52 or 12th ed. 2.5 p. 102: 59, 60.
2. Thomas Chapter 2 Practice Exercises 11th ed. p. 139: 1, 2 or 12th ed. p. 117: 1, 2. Also indicate the type of
discontinuities.
3 2
3. Does the function f (x) = x −x
x−2
−2x
have a removable discontinuity at 2? If the discontinuity is removable, find
the function g that agrees with f for x , 2 and is continuous at 2.
4. Find the numbers at which f is discontinuous. At which of these numbers is f continuous from the right, from
the left, or neither?
8 2012–2020 Past Test and Examination Questions Booklet Mathematics I (Major)

(a)  2
x + 1, if x ≤ 1 


 
f (x) = 3 − x, if 1 < x ≤ 4
 √ x,

if x > 4 

(b)
1 + x2 , if x ≤ 0 
 

 
f (x) = 2 − x, if 0 < x ≤ 2
 (x − 2)2 ,

if x > 2 

5. Let  √
−x, if x < 0 


 
f (x) = 3 − x, if 0 ≤ x < 3
 (x − 3)2 ,

if x > 3 

(a) Evaluate each limit, if it exists.


(i) lim+ f (x); (ii) lim− f (x); (iii)lim f (x); (iv) lim− f (x); (v) lim+ f (x); (vi) lim f (x).
x→0 x→0 x→0 x→3 x→3 x→3
(b) Where is f discontinuous?

(c) Sketch the graph of f

Tutorial 3.1.1

1. Thomas Exercises 11th ed. 3.1 pp. 152–153 or 12th ed. 3.2 p. 131: 1, 2, 4, 6, 12.
2. Find f 0 (a) if it exists, where
x3 if x ≤ 1
(
(a) f (x) = and a = 1.
cx + b if x > 1

3x2 if x ≤ 1
(
(b) f (x) = and a = 1.
3
2x + 1 if x > 1
3. Thomas Chapter 2 Practice Exercises 11th ed. p. 139: 13–16 or 12th ed. p. 118: 13–16. Each limit represents
the limit of some function f at some number a. State such an f and a in each case.
4. A curve has equation y = f (x).
(a) Write an expression for the slope of the secant line through the points P(3, f (3)) and Q(x, f (x)).
(b) Write an expression for the slope of the tangent line at P.
5. If f (x) = 3x2 − x3 , find f 0 (1) and use it to find an equation of the tangent line to the curve y = 3x2 − x3 at the
point (1,2).
6. Find f 0 (a).
2
(a) f (x) = 3x
√ − 4x + 1
(b) f (x) = 1 − 2x
4
(c) f (x) = √1−x

7. Each limit represents the derivative of some function f at some number a. State such an f and a in each case.
h+3
(a) lim 2 h −8
h→0
1
4 −4
(b) lim x− 14
x→ 14
sin θ− 21
(c) limπ θ− π6
θ→ 6
Mathematics I (Major) 2012–2020 Past Test and Examination Questions Booklet 9

Tutorial 3.2.1

1. Thomas Exercises 11th ed. 2.7 pp. 136–137: 5, 8, 11, 16, 17, 31, 32 or 12th ed. 3.1 p. 125: 5, 8, 11, 16, 17,
31, 33, 34.
2. Find an equation of the tangent line to the curve at the given point.
2
(a) y = 4x
√ − 3x , (2, −4)
(b) y = x, (1, 1)
(c) y = 2x+1
x+2 , (1, 1)

3. (a) Find the slope of the tangent of the curve y = 3 + 4x2 − 2x3 at the point where x = a.
(b) Find equations of the tangent lines at the points (1,5) and (2,3).
(c) Graph the curve and both tangents on common screen.

4. Find an equation of the tangent line to the graph of y = g(x) at x = 5 if g(5) = −3 and g0 (5) = 4.
5. (a) If G(x) = 4x2 − x3 , find G0 (a) and use it to find equations of the tangent lines to the curve y = 4x2 − x3 at
the points (2,8) and (3,9).
(b) Graph the curve and the tangent lines on the same screen.

6. Find an equation of the tangent to the curve y = 4 sin2 x at ( π6 , 1).



7. Find equations of the tangent line to the curve y = 1 + 4 sin x, at (0,1).

Tutorial 3.2.2

1. Thomas Exercises 11th ed. 2.7 pp. 137–138: 33, 39, 44 or 12th ed. 3.1 p. 126: 35, 41, 46.
2. Sketch the graph of a function g that is continuous on its domain (-5,5) and where g(0) = 1, g0 (0) = 1, g0 (−2) =
0, lim + g(x) = ∞, and lim− g(x) = 3.
x→−5 x→5

3. Sketch the graph of a function g for which g(0) = g(2) = g(4) = 0, g0 (1) = g0 (3) = 0, g0 (0) = g0 (4) = 1, g0 (2) =
−1, lim− g(x) = ∞, and lim + g(x) = −∞.
x→5 x→−1

4. Sketch the graph of a function f where the domain is (-2,2), f 0 (0) = −2, lim− f (x) = ∞, f is continuous at all
x→2
numbers in its domain except ±1 and f is odd.

5. Let f (x) = 3 x.
(a) If a , 0, find f 0 (a).
(b) Show that f 0 (0)√does not exist.
(c) Show that y = 3 x has a vertical tangent line at (0,0).

6. (a) If g(x) = x2/3 , show that g0 (0) does not exist.


(b) If a , 0, find g0 (a).
(c) Show that g(x) = x2/3 has a vertical tangent line at (0,0).

Tutorial 3.4.1

1. Thomas Exercises 11th ed. 3.1 pp. 153–154: 27–32 or 12th ed. 3.2 p. 132: 27–32.
2. Let f (x) = x2 .
(a) Find the values of f 0 (0), f 0 ( 21 ), f 0 (1), and f 0 (2).
(b) Deduce the values of f 0 (− 12 ), f 0 (−1), and f 0 (−2).
(c) Use the results in parts (a) and (b) to guess a formula for f 0 (x).
(d) Use the definition of the derivative to prove that your guess in part (c) is correct.
10 2012–2020 Past Test and Examination Questions Booklet Mathematics I (Major)

3. Find the derivative of the function using the definition of the derivative. State the domain of the function and
the domain of its derivative.
(a) f (x) = 3x − 8
(b) f (x) = 4 + 8x − 5x2
x2 −1
(c) f (x) = 2x−3 √
(d) f (x) = x + x

4. Use the definition of the derivative to find f 0 (x) and f 00 (x). f (x) = 3x2 + 2x + 1.
Check to see if your answers are reasonable.
0 0
5. Use the
√ definition of the derivative to find f (x). What are the domains of f and f .
f (x) = 6 − x.

Tutorial 3.5.1

1. Thomas Exercises 11th ed. 3.2 pp. 167–169: 9, 10, 12, 13, 15, 17, 20, 22, 43, 46, 52, 55 or 12th ed. 3.3 pp.
143–145: 9, 10, 12, 13, 15, 17, 20, 22, 57, 60, 74, 77.
2. Thomas Chapter 3 Practice Exercises 11th ed. pp. 255–257: 4, 9, 99, 101, 107, 109 or 12th ed. pp. 213–215:
4, 9, 99, 101, 107, 109.
3. Differentiate the function.
(a) g(x) = 74 x2 − 3x + 12
(b) g(x) √= x2 (1 − 2x)
(c) y = √3 x(2 + x)
(d) y = xx+x 2
x2 +4x3
(e) y = √
x

4. Find the derivative of f (x) = (1 + 2x2 )(x − x2 ) in two ways: by using the Product rule and by performing the
multiplication first.
Do your answers agree?

5. Find the
4
derivative
3
√ of the function
F(x) = x −5xx2 + x
in two ways: by using the Quotient rule and by simplifying first. Show that your answers are equivalent. Which
method do you prefer?

6. (a) Use the Product rule twice to prove that if f, g, and h are differentiable, then ( f gh)0 = f 0 gh + f g0 h + f gh0 .
(b) Taking f = g = h in part (a), show that
d 3 2 0
dx [ f (x)] = 3[ f (x)] f (x)
(c) Use part (b) to differentiate y = (x4 + 3x3 + 17x + 82)3 .

Tutorial 3.5.2

1. Thomas Exercises 11th ed. 3.5 pp. 199–202: 1, 2, 9, 11, 14, 23, 31, 34, 67, 71, 72–74, 116 or 12th ed. 3.6 pp.
167–169: 1, 2, 9, 11, 13, 23, 31, 34, 79, 83, 84, 87, 88, 104.
2. For the following functions, find the points on the graphs where the tangents are horizontal or vertical. (a)
1
f (x) = x − ,
x
4 1
(b) g(x) = x 3 + 4x 3 ,
3
(c) h(x) = x(x + 2) 5 ,
Mathematics I (Major) 2012–2020 Past Test and Examination Questions Booklet 11

1
(d) d(x) = xr − x r on [0, ∞], where r > 1.
3. Are there points on the curve y = x − e(−x) where the slope of the tangent is 2? If so, find them.
Hint: the first derivative of the function y with respect to x is 2.

4. Write the composite function in the form f (g(x)).


Identify the inner function u = g(x) and the outer function y = f (u). Then find the derivative dy/dx.
2 10
(a) y = (1
√− x )
(b) y = 4 + 3x

5. Find the derivative of the function.


(a) f (x) = (2x − 3)4 (x2 + x + 1)5
(r2 −1)3
(b) H(r) = (2r+1) 5


q »
(c) y = x + x + x

6. Find y0 and y00 for y = √4x .


x+1
7.Find an equation of the tangent line to the curve at the given point.
−6
(a) y = (3x
√ − 1) , (0, 1)
3
(b) y = 1 + x , (2, 3)

Tutorial 3.6.1

1. Thomas Exercises 11th ed. 3.4 pp. 186–187: 1, 2, 5, 10, 11, 24, 26 or 12th ed. 3.5 pp. 159–160: 1, 2, 6, 9, 14,
15, 18, 20.
2. Thomas Exercises 11th ed. 3.5 pp. 199–200: 3, 6, 7, 28, 43, 44, 46, 57, 69, 70 or 12th ed. 3.6 pp. 167–168: 3,
6, 7, 28, 45, 46, 48, 63, 81, 82.
3. Thomas Chapter 3 Practice Exercises 11th ed. p. 255: 20, 39 or 12th ed. pp. 213–214: 20, 39.
4. Does the curve y = sin (x − sin x) have horizontal tangents at the x − axis? Give reasons.
Hint: y0 is 0 for horizontal tangents.
5. Differentiate.
(a) f (x) = x2 sin x
(b) g(t) = 4 sec t + tan t
(c) y = 2 sec x − csc x
x
(d) y = 2−tan x
(e) y = x2 sin x tan x

6. Find an equation of the tangent line to the curve.


(a) y = 2x sin x at the point ( π2 , π).
(b) y = tan(πx2 /4) at the point (1, 1).

7. Find the limit.


(a) lim sin 3xx2sin 5x
x→0
(b) lim csc x sin(sin x)
x→0
sin(x−1)
(c) lim 2
x→1 x +x−2

8. Write the composite function in the form f (g(x)).


Identify the inner function u = g(x) and the outer function y = f (u). Then find the derivative dy/dx.
(a) y = tan(sin
√ x)
(b) y = sin x
12 2012–2020 Past Test and Examination Questions Booklet Mathematics I (Major)

9. Find the derivative of the function.


(a) y = cos4 (sin3 x)
(b) y = [x + (x + sin2 x)3 ]4

Tutorial 3.7.1

1. Thomas Exercises 11th ed. 3.6 pp. 209–210: 19, 22, 25, 28, 30, 31, 35, 47, 54, 56, 59, 61 or 12th ed. 3.7 p.
174: 1, 4, 7, 10, 13, 19, 31, 38, 40, 43, 45.
dy
if y sin 1y = 1 − xy.
Ä ä
2. Find dx

3. (a) find y0 by implicit differentiation.


(b) Solve the equation explicitly for y and differentiate to get y0 in terms of x.
(c) Check that your ANSWERS to parts (a) and (b0 are consistent by substituting the expression for y into the
solution for part (a).
2
(i) xy
√ + 2x√+ 3x = 4
(ii) x + y = 1

4. Find dy/dx by implicit differentiation.


(a) x3 − xy2 + y3 = 1
x2
(b) x+y = y2 + 1
(c) x4 (x + y) = y2 (3x − y)

(d) xy = 1 + x2 y
(e) x sin y + y sin x = 1

5. Use implicit differentiation to find an equation of the tangent line to the curve at the given point.
(a) y sin 2x = x cos 2y, (π/2, π/4).
(b) 2(x2 y2 )2 = 25(x2 − y2 ), (3, 1).

6. If x2 + xy + y3 = 1, find the value of y000 at the point where x = 1.

Tutorial 3.8.1

1. Thomas Exercises 11th ed. 3.2 p. 167: 24, 27, 28 or 12th ed. 3.3 p. 143: 30, 31, 40.
2. Thomas Exercises 11th ed. 3.6 p. 209: 32 or 12th ed. 3.7 p. 174: 16.
3. Thomas Practice Exercises 11th ed. pp. 255–256: 44, 73, 74 or 12th ed. p. 214: 44, 73, 74.
4. Using the Chain Rule we have
d f (x)
e =
dx
5. Using the Chain Rule we have
d f (x)
a =
dx
6. Differentiate
√ the function.
(a) f (x) = e
(b) f (x) = (3x2 − 5x)e x
(c) y = etan θ
2 x
(d) f (x) = xx2 +e
e
x

7. Find the limit.


3x −3x
(a) lim ee3x −e
+e−3x
x→∞
(b) lim− e3/(2−x)
x→2
Mathematics I (Major) 2012–2020 Past Test and Examination Questions Booklet 13

(c) lim (e−2x cos x)


x→∞

8. Find an equation of the tangent line to the curve at the given point.
(a) y = e2x cos πx, (0, 1)
x
(b) y = ex , (1, e)

Tutorial 3.8.2

1. Thomas Exercises 11th ed. 3.7 pp. 221–222: 13, 14, 21, 22, 28, 30, 33, 38, 57–60, 76, 77, 86 or 12th ed. 3.8
pp. 184–185: 13, 14, 21, 22, 28, 30, 33, 38, 57–60, 76, 77, 86.

2. Using the Chain Rule we have


d
loga f (x) =
dx

3. Differentiate.
(a) f (x) = x ln x − x
(b) f (x) = cos(ln x)
2
(c) f (x) = ln(sin
√5 x)
(d) f (x) = ln x
(e) f (x) = sin x ln(5x)
(f) y = ln(csc x − cot x)
(g) y = log2 (x log5 x)

4. Find y√0 and y00 .


(a) y = x ln x
ln x
(b) y = 1+ln x

5. Differentiate f and find the domain of f .


x
(a) f (x) = 1−ln(x−1)

(b) f (x) = 2 + ln x

6. Find an equation of the tangent line to the curve at the given point.
(a) y = ln(x2 − 3x + 1), (3, 0).
(b) y = x2 ln x, (1, 0).

Tutorial 3.9.1

1. Thomas Exercises 11th ed. 3.10 pp. 250–251: 1, 2, 3, 6, 8, 9, 11, 13, 14, 21, 30, 31, 33, 38, 39, 43, 44 or 12th
ed. 3.11 pp. 210–211: 1, 2, 3, 6, 8, 9, 11, 13, 14, 21, 30, 31, 33, 38, 39, 43, 44.
2. Find the linearization L(x) of the function at a.
(a) f (x) = x3 − x2 + 3, a = −2.
(b) f (x) = sin x, a = π/6.

3. Find the differential dy of each function.


(a) y = (x2 − 3)−2
(b) y = θ2 sin
√ 2θ
(c) y = tan t
(d) y = 1x sin x
14 2012–2020 Past Test and Examination Questions Booklet Mathematics I (Major)

4. (a) Find
√ the differential dy and (b) evaluate dy for the given values of x and dx.
(i) y = 3 + x2 , x = 1, dx = −0.1
(ii) y = x+1
x−1 , x = 2, dx = 0.05

5. Use linear approximation (or differentials) to estimate the given number.


4
(a) (1.999)
√3
(b) 1001

Tutorial 4.1.1

1. Thomas Exercises 11th ed. 4.8 pp. 338–340: 2, 3, 7, 36, 39, 40, 55, 65, 66, 72, 73, 75, 76, 77, 83, 86 or 12th
ed. 4.8 pp. 285–286: 2, 3, 7, 36, 39, 40, 55, 65, 66, 72, 73, 75, 76, 77, 83, 86.
1
2. Integrate the function f (x) = (2+6x+9x 2
)
Hint: Complete the square.
3. Verify by differentiation that the formula is correct.
R √ 2
(a) x a + bx dx = 15b2
(3bx − 2a)(a + bx)3/2 + C
R
(b) x cos x2 dx = 12 x + 41 sin 2x + C

4. Evaluate the integral.


R (1+e x )2
(a) ex dx
R 2
(b) x2 x dx
R
(c) e x cos(e x ) dx
R (ln x)2
(d) x dx
R
(e) etan x sec2 x dx
R sin2x
(f) 1+cos2 x
dx
R x
(g) 1+x4
dx

5. Show that
R 1 1 x
(a) dx = tan−1 +C

x2 +a2 a a

R
(b) √ 1 dx = 1
sin−1 x
+C

a2 −x2 a a

Tutorial 4.2.1

1. Thomas Exercises 11th ed. 5.4 p. 392: 1, 2, 14, 23, 31, 32 or 12th ed. 5.4 p. 333: 1, 2, 6, 7, 12, 16, 20, 21, 23,
24, 33, 34.
2. Evaluate the integral.
Mathematics I (Major) 2012–2020 Past Test and Examination Questions Booklet 15

R1
(a) 0
(xe + e x ) dx
R4 3
(b) 2 x
dx
R2 e1/x
(c) 1 x2
dx
R 9 Ä√
√1
ä
(d) 4
x+ x
dx
R1 √
1+e−x
(e) 0 ex dx
Re x2 +x+1
(f) 1 x dx
R √
3 8
(g) √
1/ 3 1+x2
dx
3. Determine whether this statement is true or false.
R 16 dx
(a) 2 x = 3 ln 2
R π/2 cos x π
(b) 0 1+sin2 x
dx = 4

R1 ex π
(c) 0 1+e2x
dx = arctan e − 4

4. What is wrong with this equation?


R1 −3 1
(a) −2 x4 dx = x−3 −2 = − 83
Rπ π
(b) π/3
sec θ tan θ dθ = sec θ π/3
= −3

Tutorial 4.3.1

1. Thomas Exercises 11th ed. 5.5 pp. 402–403: 2, 7, 10, 19, 24, 34, 40, 55-57, 67 or 12th ed. 5.5 pp. 342–344:
3, 4, 5, 8, 11, 14, 21, 26, 36, 52, 67-69, 79.
2. Thomas Exercises 11th ed. 5.6 pp. 410–414: 1, 2, 4, 5, 8, 13, 23, 29, 31, 32, 37, 40, 111, 112, 115, 116 or 12th
ed. 5.6 pp. 350–353: 1, 2, 4, 5, 8, 13, 23, 29, 31, 32, 37, 40, 111, 112, 113, 115, 116.
3. The Substitution Rule gives neat formulas for functions with multiples. For example, if k is a constant, then
using the substitution u = kx will obtain:

Z
ekx dx =
Z
sin kx dx =
Z
cos kx dx =
Z
sec2 kx dx =
Z
sec kx tan kx dx =
16 2012–2020 Past Test and Examination Questions Booklet Mathematics I (Major)

4. Evaluate this integral by using substitution:


Z
eθ cos ec eθ + 1 dθ


5. Evaluate the integral by making the given substitution.


R
(a) cos 2x dx, u = 2x
R
(b) x(2x2 + 3)4 dx, u = 2x2 + 3
R √
(c) x2 x3 + 1 dx, u = x3 + 1
R
(d) sin2 θ cos θ dθ, u = sin θ

6. Evaluate the indefinite integral.


R
(a) (x2 + 1)(x3 + 3x)4 dx
R cos(π/x)
(b) x2
dx
R sec2 x
(c) tan2 x
dx

7. Evaluate the definite integral.


R π/6 sin t
(a) 0 cos2 t
dt
R π/4
(b) −π/4
(x3 + x4 t tan x) dx
R 13 1
(c) 0

3
dx
(1+2x)2
R2 2
(d) 0
(x − 1)e(x−1) dx

Tutorial 4.4.1

1. Thomas Exercises 11th ed. 8.2 pp. 552–554: 1, 6, 9, 12, 14, 25, 26, 29, 30 or 12th ed. 8.1 pp. 459–460: 1, 6,
11,R13,
π
16, 18, 25, 26, 29, 30.
4
2. 0
sec3 x dx =
3. Evaluate the integral using integration by parts with the indicated choices of u and dv.
R
(a) xe2x dx; u = x, dv = e2x dx
R √ √
(b) x ln x dx; u = ln x, dv = xdx

4. Evaluate the integral.


R
(a) (x2 + 2x) cos x dx
Mathematics I (Major) 2012–2020 Past Test and Examination Questions Booklet 17


(b) 0
x sin x cos x dx
R xe2x
(c) (1+2x)2
dx
R
(d) (arcsin x)2 dx

5. First make a substitution and then use integration by parts to evaluate the integral.
R
(a) cos(ln x) dx
R
(b) x ln(1 + x) dx

6. Use integration by parts to prove the reduction formula.


R R
(a) (ln x)n dx = x(ln x)n − n (ln x)n−1 dx
R tann−1 x
R
(b) tan x dx = n−1 − tann−2 x dx

Tutorial 5.1.1

1. Thomas Exercises 11th ed. 3.2 p. 167: 29, 31, 33, 35, 36 or 12th ed. 3.3 p. 143: 41, 45, 47, 51, 52.
2. Thomas Exercises 11th ed. 3.4 p. 186: 25 or 12th ed. 3.5 p. 160: 33.
3. Thomas Exercises 11th ed. 3.6 p. 209: 39–44 or 12th ed. 3.7 p. 174: 23–26.
4. Thomas Exercises 11th ed. 11.8 p. 794: 2, 4, 5, 7, 8 or 12th ed. 10.8 p. 606: 2, 4, 6, 7, 9, 10.
5. If f (x) = e2x , find the formula for f (n) (x).
6. Find the thousandth derivative of f (x) = xe−x .
7. Find a formula for f (n) (x) if f (x) = ln(x − 1)

Tutorial 5.2.1

1. Thomas Exercises 11th ed. 3.7, p. 221: 3, 4, 7, 9 or 12th ed. 3.8 p. 184: 3, 4, 7, 9.

2. Let f (x) = 1 − x2 , 0 ≤ x ≤ 1.
(a) Find f −1 . How is it related to f ?
(b) Identify the grah of f and explain your answer to part (a).

3. Suppose f −1 is the inverse function of a differentiable function f and f (4) = 5, f 0 (4) = 23 .


Find ( f −1 )0 (5).
Rx√
4. If f (x) = 3 1 + t3 dt, find ( f −1 )0 (0).
5. Differentiate the function.
3
(a) y = eax
2 x
(b) f (x) = xx2 +e
e
x

(c) f (x) = (3x2 − 5x)e x

6. Differentiate the function.


(a) y = xsin x
(b) y = (cos x) x
(c) g(x) = x4 4 x
18 2012–2020 Past Test and Examination Questions Booklet Mathematics I (Major)

7. Let f (x) = arctan(1/x) if x , 0 and f (0) = 0.


(a) Is f continuous at 0?
(b) Is f differentiable at 0?

Tutorial 5.3.1

1. Thomas Exercises 11th ed. 3.8 pp. 230–231: 49, 50, 61, 62, 64, 65, 67, 68 or 12th ed. 3.9 p. 191: 21, 22, 33,
34, 36, 37, 39, 40.
2. Thomas Exercises 11th ed. 5.5 p. 403: 49, 50, 53, 54 or 12th ed. 5.5 p. 343: 61, 62, 65, 66.
3. Thomas Exercises 11th ed. 8.6 p. 584: 23, 24, 31, 32 or 12th ed. 8.5 p. 485: 17–20. For each integral, use
appropriate methods to find the antiderivates.
4. Using the Chain Rule,
d
arcsin f (x) =
dx
5. Using the Chain Rule,
d
arctan f (x) =
dx
6. Find the
√ derivative of the function. Simplify where possible.
(a) y = tan−1 √
(b) y = x sin−1 x + 1 − x2
(c) y = sin−1 (2x + 1)
(d) F(x) = √x sec−1 (x3 )
(e) g(x) = x2 − 1 sec−1 x
(f) y = cos−1 (e2x )
(g) y = arccos b+a cos x
a+b cos x , 0 ≤ x ≤ π, a > b > 0.


7. Find the derivative of the function f (x) = arcsin(e x ). Find the domains of the function and its derivative.
8. Find y0 if tan−1 (x2 y) = x + xy2 .
9. Find an equation of the tangent line to the curve y = 3 arccos(x/2) at the point (1, π).

10. Find f 0 (x) if f (x) = 1 − x2 arcsin x. Check that your answer is reasonable by comparing the graphs of f and
f 0.
11. Evaluate the integral.
R 1/2 −1 x
(a) 0 sin√ dx
R 1−x2
dx
(b) √
2 −1
1−x sin x

Tutorial 5.4.1

1. Thomas Exercises 11th ed. 3.7 pp. 221–222: 41, 42, 49, 51–54, 89–96 or 12th ed. 3.8 pp. 184–185: 41, 42,
49, 51–54, 89–96.
2. Use logarithmic differentiation to find the derivative of the function.
5 4 6
(a) y = (2x
» + 1) (X − 3)
x−1
(b) y = 4
√ x +1 2
(c) y = xe x −x (x + 1)2/3
sin x
(d) y = x
(e) y = (ln x)cos x

3. Differentiate f and find the domain of f .


x
(a) f (x) = 1−ln(x−1)
Mathematics I (Major) 2012–2020 Past Test and Examination Questions Booklet 19

(b) f (x) = ln(x2 − 2x)

4. Find an equation of the tangent line to the curve y = ln(x2 − 3x + 1), at the given point (3,0).
5. Let f (x) = cx + ln(cos x). For what values of c is f 0 (π/4) = 6?

Tutorial 5.5.1

1. Thomas Exercises 14.3 11th ed. pp. 978–980: 1, 7, 11, 17, 19, 23, 25, 29, 41, 47, 63, 66, 68, 69, 74 or 12th ed.
pp. 790-792: 1, 7, 11, 17, 19, 23, 25, 29, 41, 47, 73, 76, 79, 80, 81, 86.
2. Find the indicated partial derivative.
(a) f (x, y) = y sin−1 (xy); fy (1, 21 )

1− x2 +y2 +z2
(b) f (x, y, z) = ln √ 2 2 2 ; fy (1, 2, 2)
1+ x +y +z

3. Use implicit differentiation to find ∂z/∂x and ∂z/∂y.


(a) x2 + 2y2 + 3z2 = 1
(b) yz + x ln y = z2

4. Find all second partial derivatives.


(a) f (x, y) = x4 y − 2x3 y2
y
(b) z = 2x+3y
(c) f (x, y) = ln(ax + by)

5. Find the indicated partial derivative.


(a) f (x, y) = x4 y2 − x3 y; f xxx , f xyx
(b) f (x, y) = sin(2x + 5y); fyxy
2
(c) f (x, y, z) = e xyz ; f xyz

Tutorial 6.1.1

1. Thomas Exercises 11th ed. 4.6 pp. 323-324: 3, 6, 13, 16, 19, 20, 21, 27, 28, 29, 32, 33, 34, 37, 45, 47, 50, 51,
52, 57, 59, 61, 62, 63 or 12th ed. 4.5 p. 261–262: 3, 6, 13, 16, 19, 20, 21, 27, 28, 29, 32, 33, 34, 37, 45, 51, 54,
55, 56, 67, 69, 75, 76, 77.
2. Find the limit. Use the l’Hospital’s Rule where appropriate. If there is a more elementary method, consider
using it. If the l’Hospital’s Rule does not apply, explain why.
(a) lim xx−3
2 −9
x→3
x2 −2x−8
(b) lim
x→4 x−4
2x
(c) lim cosex−(x+1)
−1
x→0 √ √
(d) lim 1+2x−x 1−4x
x→0
x
(e) lim 3x3
x
x→0 −1
(f) lim+ cosln(ex ln(x−a)
x −ea )
x→a
(g) lim sin 5x csc 3x
x→0
(h) lim+ (tan 2x) x
x→0

3. Use the l’Hospital’s Rule to help sketch the curve.


(a) y = 1x + ln x
(b) y = e x /x
20 2012–2020 Past Test and Examination Questions Booklet Mathematics I (Major)

4. What happens if you try to use l’Hospital’s Rule to find the limit? Evaluate the limit using another method.
(a) lim √ x2
x→∞ x +1
sec x
(b) lim
x→(π/2)− tan x

Tutorial 6.2.1

1. Thomas Exercises 11th ed. 3.3 pp. 177-178: 1, 4, 6 or 12th ed. 3.4 p. 152: 1, 4, 6.
2. Thomas Exercises 11th ed. 3.9 pp. 236–240: 3, 8, 10, 17, 19, 20, 21, 23, 30 or 12th ed. 3.10 pp. 198–200: 11,
13, 18, 20, 27, 29, 30, 31, 33, 39.

3. Suppose y = 2x + 1, where x and y are function of t.
(a) If dx/dt = 3, find dy/dt when x = 4.
(b) If dy/dt = 5, find dx/dt when x = 12.

4. Suppose 4x2 + 9y2 = 36, where x and y are function


√ of t.
(a) If dy/dt = 31 , find dx/dt when x = 2 and y = 23 5.

(b) If dx/dt = 3, find dy/dt when x = −2 and y = 23 5.

5. If x2 + y2 + z2 = 9, dx/dt = 5 and dy/dt = 4, find dz/dt when (x, y, z) = (2, 2, 1).

6. (a) If A is the area of a circle with radius r and the circle expands as time passes, find dA/dt in terms of dr/dt.

(b) Suppose oil spills from a ruptured tanker and spreads in a circular pattern. If the radius of the oil spill increases
at a constant rate of 1 m/s, how fast the area of the spill increasing when the radius is 30 m?
7. The area of a triangle with sides of lengths a and b and contained angle θ is
1
A= ab sin θ
2
(a) If a = 2cm, b = 3cm, and θ increases at a rate of 0.2 rad/min, how fast is the area increasing when θ = π/3?
(b) If a = 2cm, b increases at a rate of 1.5 cm/min, and θ increases at a rate of 0.2 rad/min, how fast is the area
increasing when b = 3cm and θ = π/3?
(c) If a increases at a rate of 2.5 cm/min=2 cm, b increases at a rate of 1.5 cm/min, and θ increases at a rate of 0.2
rad/min, how fast is the area increasing when a = 2cm, b = 3cm and θ = π/3?

Tutorial 6.3.1

1. Thomas Exercises 11th ed. 4.1 pp. 272–273: 1, 5, 6, 7, 10, 14, 17, 20, 29, 33, 41, 45, 46, 48, 49, 52, 53 or 12th
ed. 4.1 pp. 227–229: 1, 5, 6, 7, 10, 14, 18, 23, 26, 35, 39, 49, 52, 55, 59, 60, 62, 63, 66, 67.
2. (a) Sketch the graph of a function on [-1,2] that has an absolute maximun but no local maximum.

(b) Sketch the graph of a function on [-1,2] that has a local maximun but no absolute maximum.

3. Sketch the graph of f by hand and use your sketch to find the absolute and local maximun and minimum values
of f .
(a) f (x) = 12 (3x − 1), x ≤ 3
(b) f (x) = 2 − 31 x, x ≥ −2
(c) f (x) = 1/x, 1 > x > 3
(d) f (x) = (
sin x, 0 < x ≤ π/2
2x + 1, if 0 ≤ x < 1
)
(e) f (x) =
4 − 2x, if 1 ≤ x ≤ 3
Mathematics I (Major) 2012–2020 Past Test and Examination Questions Booklet 21

4. Find the critical numbers of the function.


(a) f (x) = 5x2 + 4x
(b) f (x) = 2x3 + x2 + 2x

210 sin x
5. A formula for the derivative of a function is given: f 0 (x) = 1 + x2 −6x+10
.
How many critical numbers does f have?

6. Find the absolute maximum and absolute minimum values of f on the given interval.
(a) f (x) = 3x2 − 12x + 5, [0, 3]
x
(b) f (x) = x2 −x+1 , [0, 3]
(c) f (t) = 2 cos t + sin 2t, [0, π/2]

Tutorial 6.4.1

1. Thomas Exercises 11th ed. 4.2 pp. 282–284: 1, 3, 4, 5, 7–10, 12, 13, 15, 16, 19, 22, 51, 56, 57, 58 or 12th ed.
4.2 pp. 236–237: 1, 3, 5, 6, 11, 12, 15, 16, 18, 19, 21, 22, 25, 28, 57, 66, 67.
2. Show that the functions y(x) = Ce x , C ∈ R, are the only functions which satisfy y0 = y. Hint: Consider the
function y(x)e−x .
3. Verify that the function satisfies the three hypotheses of the Rolle’s Theorem on the given interval. Then find
all numbers c that satisfy the conclusion of the Rolle’s theorem.
(a) f (x) = 2x2 − 4x + 5, [−1, 3]
(b) f (x) = sin(π/2), [π/2, 3π/2]

4. Let f (x) = 1 − x2/3 . Show that f (−1) = f (1) but there is no number c in (-1, 1) such that f 0 (c) = 0. Why does
this not contradict Rolle’s Theorem?
5. Verify that the function satisfies the hypotheses of the Mean Value Theorem on the given interval. Then find all
numbers c that satisfy the conclusion of the Mean Value Theorem.
2
(a) f (x) = 2x
√3 − 3x + 1, [0, 2]
(b) f (x) = x, [0, 1]

6. Let f (x) = (x − 3)−2 . Show that there is no value c in (1, 4) such that f (4) − f (1) = f 0 (c)(4 − 1) = 0. Why does
this not contradict Mean Value Theorem?
7. Use the Mean Value Theorem to prove the inequality

| sin a − sin b| ≤ |a − b|

for all a and b

Tutorial 6.5.1

1. Thomas Exercises 11th ed. 4.3 pp. 289–290: 1, 5, 8, 9, 16, 17, 21, 23, 47, 48 or 12th ed. 4.3 pp. 241–242: 1,
5, 12, 19, 26, 27, 33, 35, 67, 68.
2. Thomas Exercises 11th ed. 4.4 pp. 298–301: 1, 2, 3, 5, 67, 68, 75, 77, 79, 83, 84 or 12th ed. 4.4 pp. 251–254:
1, 2, 3, 5, 103, 104, 111, 113, 115, 117, 118.
3. (a) Find the intervals on which f is increasing or decreasing.

(b) Find the local maximum and minimum values of f .


22 2012–2020 Past Test and Examination Questions Booklet Mathematics I (Major)

(c) Find the intervals of concavity and the inflection points.

(i) f (x) = x3 − 3x2 − 9x + 4


2
(ii) f (x) = x2x+3
(iii) f (x) = sin x + cos x, 0 ≤ x ≤ 2π

4. Sketch the graph of a function that satisfies all the given conditions.
(a) f 0 (1) = f 0 (−1) = 0, f 0 (x) < 0 if |x| < 1,
f 0 (x) > 0 if 1 < |x| < 2, f 0 (x) = −1 if |x| > 2,
f 00 (x) < 0 if − 2 < x < 0, infection point (0,1)
(b) f 0 (x) > 0 for all x , 1, vertical asymptote x = 1,
f 00 (x) > 0 if x < 1 or x > 3, f 00 (x) < 0 if 1 < x < 3

(c) f 0 (0) = f 0 (4) = 0, f 0 (x) = 1ifx < −1,


f 0 (x) > 0 if 0 < x < 2,
f 0 (x) < 0 if − 1 < x < 0 or 2 < x < 4 or x > 4,
lim− f (x) = ∞, lim+ f (x) = −∞
x→2 x→2
00
f (x) > 0 if −1 < x < 2 or 2 < x < 4,
f 00 (x) < 0 if x > 4

Tutorial 6.5.2

1.Thomas Exercises 11th ed. 4.4 p. 299: 21, 25, 27, 29, 30, 33, 39, 41, 42 or 12th ed. 4.4 p. 252–253: 23, 35, 37,
41, 49, 53, 55, 58, 85, 87, 88, 97.
2. Find the horizontal and vertical asymptotes of each curve.
(a) y = 5+4x
x+3
2
(b) y = 2xx2 +x−2
+x−1

(c) y = √ x−9 2
4x +3x+2

3. Find the limits as x → ∞ and as x → −∞. Use this information, together with the intercepts, to give a rough
sketch of the curve.
(a) y = 2x3 − x4
(b) y = x3 (x + 2)2 (x + 1)
(c) y = x4 − x6

4. Find the horizontal asymptotes of each curve and use them, together with concavity and intervals of increase
and decrease, to sketch of the curve.
2
(a) y = 1+2x
1+x2
(b) y = 1−x
1+x
(c) y = x2x+1

5. Sketch the curve.


(a) y = x3 + 6x2 + 9
2
(b) y = xx2 −2x
−4
sin x
(c) y = 2+cos x

Tutorial 6.6.1

1. Thomas Exercises 11th ed. 4.5 pp. 309–315: 4, 7, 8, 11, 16, 18, 19, 22, 24, 33, 43, 44, 52a, 59a or 12th ed. 4.6
Mathematics I (Major) 2012–2020 Past Test and Examination Questions Booklet 23

pp. 268–273: 4, 7, 8, 11, 16, 18, 19, 22, 24, 39, 51, 52, 60a, 67a.
2. Thomas Chapter 4 Additional and Advanced Exercises 11th ed. pp. 348–350: 15, 17, 18, 26 or 12th ed. pp.
294–295: 15, 17, 18, 26.
3. What is the maximum vertical distance between the line y = x + 2 and the parabola y = x2 for −1 ≤ x ≤ 2?
4. What is the minimum vertical distance between the parabolas y = x2 + 1 and y = x − x2 ?
5. Find the point on the line y = 2x + 3 that is closest to the origin.

6. Find the point on the curve y = x that is closest to the point (3,0).

Tutorial 7.0.1

1. Thomas Exercises 7.3 (12th ed.) or 7.4 (11th ed.): 1, 3, 5, 6, 7, 9–12, 14, 15, 17, 21, 23, 25, 27, 31, 37, 40,
41, 45, 55, 61, 62, 67, 69 and 71.
2. Prove the following formulas:
Ä √ ä
(a) sinh−1 x = ln x + x2 + 1 ,
Ä √ ä
(b) cosh−1 x = ln x + x2 − 1 ,
1 1+x
Å ã
−1
(c) tanh x = ln .
2 1−x
24 2012–2020 Past Test and Examination Questions Booklet Mathematics I (Major)

2 MATH1036 Selected Tutorial Answers: Semester One

Tutorial 0.1.1

1. TRUE or FALSE:

(a) π = 22
7 F

(b) x2 = x ∀ x in R F
(c) It is possible to draw a function which intersects its asymptote. T
(d) loge ea = a = eloge a T
−2x
(e) If x is negative then √
3|x|
is positive. T

x2 − 4
(f) x − 2 = ∀x ∈ R F
x+2
2. What is the difference in meaning between the symbol ∀ and the symbol ∃? (Ans: ∀ stands for “for all” and
it means that the statement holds for every single case. ∃ stands for “there exists” and it means you can find
at least one example where the statement holds.

3. If dxe denotes the smallest integer larger or equal to x then dπe = (Ans=4)

4. How many real numbers are there between 2 and 4? (Answer: infinitely many)
1 1 1
5. If cosec x := , sec x := and cot x := then express cosec2 x in terms of cot2 x and express
sin x cos x tan x
sec2 x in terms of tan2 x. HINT: Recall that sin2 x + cos2 x = 1. (Ans: tan2 x + 1 = sec2 x and 1 + cot2 x =
cosec2 x. To find this, you divide both sides of the equation given in the hint by cos2 x and then by sin2 x.)

6. If A ⇒ B means “if A then B” or “A implies B”, then what do you think the following expressions mean:

(a) A ⇐ B (Ans: “if statement B is true then statement A is true ” or “B implies A”


(b) A ⇔ B (Ans: this is a two-way implication which means that the statements A and B mean the same
thing. I.e., each implies the other.
Mathematics I (Major) 2012–2020 Past Test and Examination Questions Booklet 25

7. Calculate the final mark if a student obtains the following results during the year: (See the preamble in your
study guide to see the two different ways the mark is calculated and choose the higher mark.)
Mark Total
March test 13 40
April test 25 40
June test 15 40
September test 20 30
November exam 62 90
Tut tests and assignments 28 30
(Ans: The class record is everything except the November exam mark, and in this case that is 13 + 25 +
15 + 20 + 28 = 101 out of a possible 180 marks. The two different ways of calculating the mark give
101+62 101 62
270 = 60.4% and 360 + 180 = 62.5%. Hence this student gets a final mark of 62.5%.)

Tutorial 1.1.1: ANSWERS

Q2. No.
Q3. No.
Q4. lim− = 1 and lim+ = 0 (Sketch the graph?)
x→n x→n
Q5.(a) 0.1666666 . . .
(b) 16

t2 + 9 is very close to 3 when t is sufficiently small.
Q6. The limit does not exist because the left- and right-hand limits are different.

Tutorial 1.2.1 ANSWERS


√3 f (x) f (x)
Q2. a.5 b.8 c. 16 d.0 e. D.N.E. lim− = −∞ and lim+ =∞
x→a g(x) x→a g(x)

Q3. a. −1
b. √4 c. −1
d. 12
9 5 2
Q4.

x2 − 1
lim √ √
x→1 2x + 1 − 3
√ √
(x − 1)(x + 1)( 2x + 1 + 3)
= lim √ √ √ √
x→1 ( 2x + 1 − 3)( 2x + 1 + 3)
√ √
(x − 1)(x + 1)( 2x + 1 + 3)
= lim
x→1 (2x + 1) − 3
√ √
(x + 1)( 2x + 1 + 3)
= lim
x→1 2

=2 3

Q5. Nothing.

Q6. lim− g(x) = π1 .


x→e

Q7. Nothing.

Q8. The right- and left-hand limis are equal.


26 2012–2020 Past Test and Examination Questions Booklet Mathematics I (Major)

Tutorial 1.3.1: ANSWERS

3
Q3. lim = D.N.E.
x→−∞ x3/2
Q4.

x3 πx
x3 + πx +
lim = lim x2 x2
x→∞ (x + 1)2 x→∞ x2 2x 1
+ +
x2 x2 x2
π
x+
= lim x
x→∞ 2 1
1+ + 2
x x
= ∞ (D.N.E.)

Å ãx
2x 2
Q5. lim x = lim = ∞ (D.N.E.)
x→−∞ 3 x→−∞ 3
Q6.a. Let consider the function y = 2x−3
x−2 :
lim+ 2x−3
x−2 = ∞
x→2
lim 2x−3 = −∞
x→2− x−2
Therefore x = 2 is a vertical asymptote.
2x−3 2− 3
lim x−2 = lim 1− 2x . Hint: multiply the fraction by a form of 1, in this case use 1x .
x→∞ x→∞ x
2− 3
lim 1− 2x = 2.
x→∞ x
Therefore y = 2 is a horizontal asymptote.
x2
b. Let consider the function y = x−1 :
2
lim x =∞
x→1+ x−1
x2
lim− x−1 = −∞
x→1
Therefore x = 1 is a vertical asymptote.
x2 1
y = x−1 = (x + 1) + x−1 , after long division.
1
As x → ∞; x−1 → 0
Therefore, y = (x + 1) is an oblique asymptote.

Tutorial 1.4.1: ANSWERS

Q.2 a. left limit D.N.E. (= −∞) right limit D.N.E. Limit D.N.E. Function not defined at x = 0.

Tutorial 2.3.1: ANSWERS

Q1. The function g(x) is continuous at x = − 21 iff

g(−1/2) = lim g(x)


x→−1/2

Q2. The function f (x) is continuous on (−3, e] iff

f (x) is continuous at every point in the interval (−3, e) AND f (x) is left-continuous at x = e.

Tutorial 3.5.2: ANSWERS


Mathematics I (Major) 2012–2020 Past Test and Examination Questions Booklet 27

dy
Q3. = 1 + e(−x) = 2; therefore e(−x) = 1 → x = 0.
 
dx

For x = 0, y = −1 → Point (0, −1).

Tutorial 3.6.1: ANSWERS

Q4. y = sin (x − sin x)


(y0 ) = cos (x − sin x) .[1 − cos x]
For horizontal tangents:(y0 ) = 0
cos (x − sin x) = 0 or [1 − cos x] = 0
Let consider [1 − cos x] = 0; cos x = 1 and x = 2nπ
For x = 2nπ; y = sin (2nπ − sin (2nπ)) = sin (2nπ − 0) = sin (2nπ) = 0.
y = 0 is the x − axis, so yes the curve does have horizontal tangents at the x − axis.

Tutorial 3.7.1: ANSWERS

Q2. y sin 1y = 1 − xy.


Ä ä

dy −2 dy
Ä dy
1 1
Ä ä Ä ä ä
sin cos −y 0 − x .

dx + y . dx = dx + y
dy
Äyä Äy ä dy dy
1 1 1
dx sin yÄ − äy
cos y dx + x dx = −y.
dy 1 1
Ä ä
−2
dx [y sin y − cos y + xy] = −y .
−2
dy Ä ä−y Ä ä
dx = .
y sin y −cos 1y +xy
1

Tutorial 3.8.1

1. Thomas Exercises 11th ed. 3.2 p. 167: 24, 27, 28 or 12th ed. 3.3 p. 143: 30, 31, 40.
2. Thomas Exercises 11th ed. 3.6 p. 209: 32 or 12th ed. 3.7 p. 174: 16.
3. Thomas Practice Exercises 11th ed. pp. 255–256: 44, 73, 74 or 12th ed. p. 214: 44, 73, 74.
4. Using the Chain Rule we have
d f (x)
e =
dx
5. Using the Chain Rule we have
d f (x)
a =
dx
6. Differentiate
√ the function.
(a) f (x) = e
(b) f (x) = (3x2 − 5x)e x
(c) y = etan θ
2 x
(d) f (x) = xx2 +e
e
x

7. Find the limit.


3x −3x
(a) lim ee3x −e
+e−3x
x→∞
(b) lim− e3/(2−x)
x→2
(c) lim (e−2x cos x)
x→∞

8. Find an equation of the tangent line to the curve at the given point.
28 2012–2020 Past Test and Examination Questions Booklet Mathematics I (Major)

(a) y = e2x cos πx, (0, 1)


x
(b) y = ex , (1, e)

Tutorial 4.1.1: ANSWERS

Q2.
Z Z Z
1 1 1 1
 dx =  dx = 2
 dx = arctan (1 + 3x) + C
2 + 6x + 9x2 1 + 1 + 6x + 9x2 1 + (1 + 3x) 3

Tutorial 4.4.1: ANSWERS

Let u = eθ + 1
du = eθ dθ
Z
eθ cos ec eθ + 1 dθ = cos ec u du = ln | cos ec u − cot u| + C = ln | cos ec eθ + 1 − cot eθ + 1 | + C
  

Tutorial 6.4.1: ANSWERS

Q2. Let f (x) be a function s.t. f (x) = f 0 (x).


We then show that f (x) must be of the form Ce x by differentiating f (x)e−x as follows:

d
dx f (x)e−x = f 0 (x)e−x + f (x)(−e−x ) (by the product rule)

= f (x)e−x + f (x)(−e−x ) since f = f 0

= f (x)(e−x − e−x ) = f (x)(0) = 0.

d
So, since dx f (x)e−x = 0, we can integrate both sides with respect to x to get f (x)e−x = C where C is a constant.

Multiplying both sides by e x shows us that f (x) must necessarily be of the form f (x) = Ce x .
Mathematics I (Major) 2012–2020 Past Test and Examination Questions Booklet 29

Tutorial 7.0.1: ANSWERS

1. T1. sinh x = −3/4,


cosech x =p1/ sinh x = −4/3,
cosh x = 1 + sinh2 x = 5/4,
sech x = 1/ cosh x = 4/5,
tanh x = sinh x/ cosh x = −3/5,
coth x = 1/ tanh x = −5/3.
T3. cosh x = 17/15,
sech x = 1/
p cosh x = 15/17,
sinh x = cosh2 x − 1 = 8/15
(we take the positive square root since x > 0 so that sinh x > 0),
sech x = 1/ cosh x = 15/8,
tanh x = sinh x/ cosh x = 8/17,
coth x = 1/ tanh x = 17/8.
−1
T5. 2 cosh(ln x) = eln x + e− ln x = x + eln(x ) = x + 1/x.
1 1 2 −2 1
T6. sinh(2 ln x) = (e2 ln x − e−2 ln x ) = (eln(x ) − eln(x ) ) = (x2 − 1/x2 ).
2 2 2
1 5x 1
T7. cosh 5x + sinh 5x = (e + e−5x ) + (e5x − e−5x ) = e5x .
2 2
4 x 4 4x
T9. (sinh x + cosh x) = (e ) = e .
T10. ln(cosh x + sinh x) + ln(cosh x − sinh x) = ln(e x ) + ln(e−x ) = x − x = 0.
T11. For the first identity we have
1 x+y
sinh(x + y) = (e − e−x−y )
2
and

sinh x cosh y + cosh x sinh y


1 1
= (e x − e−x )(ey + e−y ) + (e x + e−x )(ey − e−y )
4 4
1 x+y x−y 1
= (e + e − e −x+y
− e−x−y ) + (e x+y − e x−y + e−x+y − e−x−y )
4 4
1 x+y −x−y
= (e − e ).
2
Clearly sinh(x + y) = sinh x cosh y + cosh x sinh y. For the second identity we have
1 x+y
cosh(x + y) = (e + e−x−y )
2
and

cosh x cosh y + sinh x sinh y


1 1
= (e x + e−x )(ey + e−y ) + (e x − e−x )(ey − e−y )
4 4
1 x+y 1
= (e + e x−y + e−x+y + e−x−y ) + (e x+y − e x−y − e−x+y + e−x−y )
4 4
1 x+y
= (e + e−x−y )
2
so that cosh(x + y) = cosh x cosh y + sinh x sinh y. Now we can easily show that the following two
identities hold:
a. sinh 2x = sinh(x + x) = sinh x cosh x + cosh x sinh x = 2 sinh x cosh x.
30 2012–2020 Past Test and Examination Questions Booklet Mathematics I (Major)

b. cosh 2x = cosh(x + x) = cosh x cosh x + sinh x sinh x = cosh2 x + sinh2 x.


T12. Straightforward calculation yields

1 x 1
cosh2 x − sinh2 x = (e + e−x )2 − (e x − e−x )2
4 4
1 2x 1
= (e + 2 + e−2x ) − (e2x − 2 + e−2x )
4 4
= 1.

d 1 1 d
Å ã
T14. y0 = sinh(2x + 1) = cosh(2x + 1) (2x + 1) = cosh(2x + 1).
dx 2 2 dx
Alternative solution:
d 1 1 d 2x+1  1
Å ã
0
y = sinh(2x + 1) = e − e−2x−1 = (2e2x+1 + 2e−2x−1 ) = cosh(2x + 1).
dx 2 4 dx 4
2

d √ sech t
T15. Here we use the chain rule: tanh t = √ . Thus
dt 2 t
d √ √ 1 √ √ d √ 1 √ √
y0 = 2 t tanh t = √ tanh t + 2 t tanh t = √ tanh t + sech2 t.
dt t dt t
d 1 d 1
T17. y0 = ln(sinh z) = sinh z = cosh z = coth z.
dz sinh z dz sinh z

T21. Differentiating and then simplifying the result yields

d 1
Å ã
0 2
y = ln cosh v − tanh v
dv 2
1 d d
= cosh − tanh v tanh v
cosh v dv dv
= tanh v − tanh v sech2 v
= tanh v(1 − sech2 v) = tanh3 v.

T23. We follow the hint and express y in terms of exponentials

2 2x2 + 2 2x3 + 2x
y = (x2 + 1) sech(ln x) = (x2 + 1) = = 2 = 2x.
eln(x) + e− ln(x) x + 1x x +1

It follows that y0 = 2.
d √ 1 d √ 1
T25. y0 = sinh−1 x = » √ x= √ .
dx 1 + ( x)2 dx 2 x(1 + x)
T27. Applying the product rule, we find that

d
y0 = (1 − θ) tanh−1 θ

d
= − tanh−1 θ + (1 − θ) tanh−1 θ

1
= − tanh−1 θ + (1 − θ)
1 − θ2
1
= − tanh−1 θ.
1+θ

d 1 1
T31. y0 = (cos−1 x − x sech−1 x) = − √ − sech−1 x + x √ = − sech−1 x.
dx 1−x 2 x 1 − x2
Mathematics I (Major) 2012–2020 Past Test and Examination Questions Booklet 31

T37. a. Applying the chain rule yields


d 1 d
tan−1 (sinh x) + C = sinh x

dx 2
1 + sinh x dx
cosh x cosh x
= =
1 + sinh x cosh2 x
2

= sech x.
p
b. Since sech x ≥ 0 we have sech2 x = sech x and
d 1 d
sin−1 (tanh x) + C = p tanh x

dx 2 dx
1 − tanh x
sech2 x sech2 x
= p = p
1 − tanh2 x sech2 x
= sech x.

T40. The chain rule and product rule provide

d 1 x 1 d
Å ã
2
x tanh x + ln(1 − x ) + C = tanh−1 x +
−1
+ (1 − x2 )
dx 2 1 − x2 2(1 − x2 ) dx
x x
= tanh−1 x + −
1 − x2 1 − x2
= tanh−1 x.

T41. Let u = 2x so that dx = 12 du. Then


Z Z
1 1 1
sinh 2x dx = sinh u du = cosh u + C = cosh 2x + C.
2 2 2

T45. The substitution u = cosh(x/7) in the following integral yields


Z Z
x sinh 7x  x
tanh dx = x dx = 7 ln cosh + C.
7 cosh 7 7

T55. The substitution u = tan θ, so that du = sec2 θ dθ, yields


Z π/4 Z 1
cosh(tan θ) sec2 θ dθ = cosh u du = [sinh u]1−1 = sinh 1 − sinh(−1)
−π/4 −1
1 1
= (e − e−1 ) − (e−1 − e) = e − e−1 .
2 2

T61. sinh−1 (−5/12) = ln −5/12 + 169/144 = ln(2/3).


T62. cosh−1 (5/3) = ln 5/3 + 16/9 = ln 3.


Z 2 √3  i2 √3 √ √ √ √
dx h
−1 x
T67. √ = sinh = sinh−1 3 = ln( 3 + 4) = ln(2 + 3).
0 4+x 2 2 0

T69. Since x ∈ (5/4, 2) implies that 1 < x2 we have


Z 2
dx 2 5 1 1 ln 3
= coth−1 x 5/4 = coth−1 2 − coth−1 = ln 3 − ln 9 = −

2
.
5/4 1 − x 4 2 2 2

T71. Let u = 4x. Then Z Z


3/13 12/13
dx du
√ = √ .
1/5 x 1 − 16x2 4/5 u 1 − u2
32 2012–2020 Past Test and Examination Questions Booklet Mathematics I (Major)

Since u ∈ (4/5, 12/13) implies that 0 < u < 1 we have


Z 12/13
du 12/13 12 4
. = − sech−1 u 4/5 = − sech−1 + sech−1


4/5 u 1−u 2 13 5
5
1 + 13 1 + 35
Ç å Ç å
= − ln 12
+ ln 4
= 2 ln 2 − ln 3.
13 5

2. (a) Let y = sinh−1 x. We use the fact that sinh(sinh−1 x) = x for x ∈ R (since sinh x is one-to-one on R):
−1 −1
−1 esinh x
− e− sinh x
(ey )2 − 1
x = sinh(sinh x) = = .
2 2ey
Rearranging the equation yields a quadratic equation in ey ,

(ey )2 − 2xey − 1 = 0,

which has the roots √


ey = x ± x2 + 1.

Since ey > 0 and x2 + 1 > |x|, we take the positive square root so that
Ä √ ä
sinh−1 x = y = ln x + x2 + 1 .

(b) Let y = cosh−1 x ≥ 0. Note that cosh x is an even function. We insist that y ≥ 0 since cosh x is
one-to-one function on [0, ∞) We use the fact that cosh(cosh−1 x) = x for x ∈ R with x ≥ 1. It follows
that −1 −1
ecosh x + e− cosh x (ey )2 + 1
x = cosh(cosh−1 x) = = .
2 2ey
Rearranging the equation yields a quadratic equation in ey ,

(ey )2 − 2xey + 1 = 0,

which has the roots √


ey = x ± x2 − 1.

Since, by assumption, y ≥ 0 we have ey ≥ 1. For large x we notice that x − x2 − 1 tends to zero.
Thus we take the positive square root, so that
Ä √ ä
cosh−1 x = y = ln x + x2 − 1 .

Alternative proof:
We could also have used the formula for sinh−1 x from (a) as follows:

x2 = cosh2 (cosh−1 x)
= 1 + sinh2 (cosh−1 x) = 1 + sinh2 y


so that sinh y = x2 − 1, where we take the positive square root so that y = cosh−1 x ≥ 0. Thus

y = sinh−1 (sinh y) = sinh−1 ( x2 − 1)
Å√ » √ ã
= ln x2 − 1 + ( x2 − 1)2 + 1
Ä√ √ ä
= ln x2 − 1 + x2
Ä √ ä
= ln x + x2 − 1

since x = x2 ≥ 1.
Mathematics I (Major) 2012–2020 Past Test and Examination Questions Booklet 33

(c) Let y = tanh−1 x for x ∈ (−1, 1). Since tanh(tanh−1 x) = x we have


−1 −1
etanh x
− e− tanh x
(ey )2 − 1
x = tanh(tanh−1 x) = = .
etanh
−1
x + e− tanh
−1
x (ey )2 + 1

Rearranging the equation yields a quadratic equation in ey ,

1+x
(ey )2 =
1−x
which has the non-negative root
1+x

y
e = .
1−x
Thus
1+x 1 1+x
… Å ã
−1
tanh x = y = ln = ln .
1−x 2 1−x
Alternative proof:
Since cosh2 x = 1 + sech2 x, we find

x2 = tanh2 (tanh−1 x) = tanh2 (y)


1
=1−
cosh2 y

and solving for cosh2 y yields


1
cosh2 y = .
1 − x2
Using the formula from (b), we obtain

1
Å ã
|y| = cosh−1 √
1 − x2
Ñ s é
ã2
1 1
Å
= ln √ + √ −1
1 − x2 1 − x2
!
1 x2 1 |x|
Å ã
= ln √ + = ln √ + √
1 − x2 1 − x2 1 − x2 1 − x2
1 + |x| 1 + |x|
Å ã Å ã
= ln √ = ln √ √
1−x 2 1 − |x| 1 + |x|
Ç å
1 + |x| 1 1 + |x|
Å ã
= ln = ln .
1 − |x| 2 1 − |x|

Noting that y = tanh−1 x has the same sign as x provides

1 1+x
Å ã
y = ln when x ≥ 0,
2 1−x
1 1−x
Å ã
−y = ln when x < 0.
2 1+x

Both cases yield


1 1+x
Å ã
−1
tanh x = y = ln .
2 1−x
Are there any other methods to arrive at this result?
34 2012–2020 Past Test and Examination Questions Booklet Mathematics I (Major)

3 Calculus Tests and Examinations


3.1 Calculus 2013
3.1.1 Calculus March 2013

Section A: Multiple choice questions

Instructions:
Section A, Questions 1–6 are multiple choice questions. Each of these questions has exactly one correct answer.
Circle the letter of your choice.

Question 1 [2]
Let f and g be real-valued functions such that lim f (x) = L and lim g(x) = M exist. Which of the following
x→a x→a
statements is false?

(A) lim f (x) + g(x) = L + M.


 
x→a
√3
(B) If L ≤ 0, then lim 3
f (x) = L.
p
x→a

f (x)
(C) If L = 0 and M = 0, lim does not exist.
x→a g(x)

(D) lim k f (x) = kL, where k is any constant.


 
x→
n
(E) If n ∈ Z and L , 0, lim f (x) , 0.

x→a

Question 2 [2]
Which of the following statements is false?

(A) lim (e x − e−x ) = ∞.


x→∞

(B) lim (e x − e−x ) = −∞.


x→−∞

(C) lim 3 x = ∞.
x→∞

(D) lim π−x = −∞.


x→−∞

(E) lim e−x = ∞.


x→−∞

Question 3 [2]
Let L be a non-negative number. Suppose that lim f (x) + L = 0. Then:
x→a

(A) lim f (x) = −L and lim+ f (x) = L.


x→a− x→a

(B) lim f (x) = −L and lim− f (x) = L.


x→a+ x→a

(C) There is not enough information to say that lim f (x) exists.
x→a

(D) lim f (x) = L.


x→a

(E) lim f (x) = −L.


x→a
Mathematics I (Major) 2012–2020 Past Test and Examination Questions Booklet 35

Question 4 [2]
 sin(πx) if x > 0


Let f (x) = 0 if x = 0
 2
x if x < 0

(A) lim f (x) does not exist.


x→0

(B) f is continuous from the left at 0 but not from the right at 0.

(C) f is continuous from the right at 0 but not from the left at 0.

(D) f is continuous at 0.

(E) None of the above.

Question 5 [2]
The graph representing the function f can be seen below.

(A) f is defined at x = 2 and x = 2 is a vertical asymptote.

(B) The line y = 3 is horizontal asymptote.

(C) f is continuous on (2, 8).

(D) lim f (x) = 3.


x→∞

(E) f is continuous on the right of x = 8.

Section B

Question 1 [17]
Evaluate the following limits if they exist:

2x3 −54
(a) lim 2 (3)
x→3 x −9

3x cos x
(b) lim √
x+4−2
(3)
x→0

(c) lim 1−cos(θ)


θ2
(3)
θ→0
√ √
x2 +5− 2x+5
(d) lim x→2 2
3x −12
(4)

9x2 + 5
(e) lim (4)
x→−∞ 1 − 2x

Question 2 [4]

(a) State the squeeze Theorem. (2)


sin x
(b) You are given that for all x ∈ (−1, 1), the function f (x) satisfies x ≤ f (x) ≤ 1 + x2 .
Show that lim f (x) = 1. (2)
x→0
36 2012–2020 Past Test and Examination Questions Booklet Mathematics I (Major)

Question 3 [9]
Recall that for a real number a, bac is the greatest integer less or equal to a, while dae is the smallest integer greater
or equal to a.
Let n be a non-negative integer and f the function defined as follows:

 n − dxe if x < n


f (x) = 0 if x = 0
n − bxc if x > 0

1. Evaluate:
(a) lim− f (x), (3)
x→n
(b) lim+ f (x). (3)
x→n

2. Is the function f continuous at the point n? Justify your answer. (3)

3.1.2 Calculus April 2013

Section A: Multiple choice questions

Instructions:
Section A, Questions 1–6 are multiple choice questions. In each of these questions, circle the letter that corre-
sponds to the correct answer.
Question 1 [2]
y
5

1 2 3 x

The graph above represents a function f . Which of the following is a true statement? The limit of the above
function:

A. Exists for all x ∈ (1, 2).


B. Exists for all x ∈ [1, 2].
C. Exists for all x ∈ (1, 3).
D. Does not exist at x = 3.
E. Exists at x = 2.

Question 2 [2]
√ x2
We are given that 3 − 2x2 ≤ f (x) ≤ 3− for all x ∈ (−1, 1). Then
2
Mathematics I (Major) 2012–2020 Past Test and Examination Questions Booklet 37

A. lim f (x) does not exist because the one-sided limits are not equal.
x→0

B. The left-hand limit exists but the right-hand limit does not exist.

C. The limit lim f (x) exists.


x→0

√ x2
D. lim 3 − 2x2 < lim f (x) < lim 3− .
x→0 x→0 x→0 2
E. There is not enough information to determine whether or not lim f (x) exists.
x→0

Question 3 [2]

3x2 − 12
Let f (x) = . Which of the following statements is true?
x−2

A. lim f (x) = ∞.
x→2−

B. lim f (x) = 0.
x→2

C. lim f (x) = lim+ f (x).


x→2− x→2

D. lim f (x) = ∞.
x→2+

E. lim f (x) = 2 3.
x→2

Question 4 [2]
Let g(x) = tan x. The function g:

A. is continuous on (0, 2π).

2n − 1 2n + 1
Å ã
B. is continuous on all intervals of the form π, π .
2 2
π
C. is undefined at any integer multiple of .
2
D. has no asymptotes.
π
E. has a horizontal asymptote at y = .
2

Question 5 [2]
Suppose that lim | f (x)| = 2. Then
x→a

A. lim f (x) = −2.


x→a

B. lim f (x) = 2 or lim f (x) = −2.


x→a x→a

C. lim f (x) does not exist.


x→a

D. there is not enough information to determine whether or not lim f (x) exists.
x→a

E. lim f (x) exists but we do not know its value.


x→a
38 2012–2020 Past Test and Examination Questions Booklet Mathematics I (Major)

Question 6 [2]

3 6 9 x

The graph above represents the function f . Which of the following statements is false?

A. The function is defined at x = 3 and x = 3 is a vertical asymptote.


B. The function is undefined at x = 9 and lim f (x) does not exist.
x→9

C. lim f (x) = ∞.
x→∞

D. lim f (x) = −∞.


x→−∞

(e) The function is continuous on (3, 9).

Section B

Question 1 [7]

(a) State the definition of the derivative of f (x) at x = a (2)


(b) State and prove the Quotient Rule for derivatives (5)

Question 2 [7]

1) Give the definition of an anti-derivative. (1)


2) Find the derivatives of the following functions: You do not need to simplify your answers.
5x
(a) f (x) = . (3)
1
ex +
ex
x3 +ln 9
(b) g(x) = e . (3)

Question 3 [8]

dy
(a) Use implicit differentiation to find if e xy + y2 cos x + e x = 6. (3)
dx
(b) Find the slope of the tangent line to e xy + y2 cos x + e x = 6 at (0, 2). (2)
Mathematics I (Major) 2012–2020 Past Test and Examination Questions Booklet 39

(c) Find the equation of the normal line to e xy + y2 cos x + e x = 6 at (0, 2). (3)

Question 4 [6]

(a) Give the linear approximation of a differentiable function f at a. (2)



(b) Find the linear appriximation of the function f (x) = 3 x at x = 1. (2)
√3
(c) Use (b) above to approximate the value of 1.0006. (2)

3.1.3 Calculus June 2013

Section A: Multiple choice questions

[10]
Instructions:
Section A, Questions 1–5 are multiple choice questions. In each of these questions, circle the letter that corre-
sponds to the correct answer.
Question 1 [2]
You are given that f (x) and F(x) are functions which satisfy
Z
f (x)dx = F(x) + c, where c is constant.

Then it follows by definition (or otherwise) that

d2 F(x)
A. = f (x).
dx2
Rb
B. a
f (x)dx = F(a) − F(b).

d R
C. f (x)dx) = F(x).

dx
d
D. [F(x)] = f (x).
dx
E. None of the above.

Question 2 [2]
Let a be a positive constant. Then

d x
A. [a ] = xa x−1 .
dx
d a
B. [x ] = xa ln x.
dx
R xa+1
C. xa dx = + c.
a+1
R a x+1
D. a x dx = + c.
x+1
E. None of the above.

Question 3 [2]
R
arcsin xdx =
40 2012–2020 Past Test and Examination Questions Booklet Mathematics I (Major)

1
A. √ + c.
1 − x2
B. − arccos x + c.
C. arccos x + c.
Z
1
D. √ dx.
1 − x2
E. None of the above.

Question 4 [2]
Let f (x) = ln x be defined on the interval [1, 2]. Then

A. f (x) is not continuous on [1, 2].


B. f (x) is not differentiable on (1, 2).
C. There exists c in (1, 2) such that f 0 (c) = 0.
1 f (2) − f (1)
D. There exists c in (1, 2) such that = .
c 2−1
E. None of the above.

Question 5 [2]
n
You are given that f (x) = x , where n is a positive integer and x is a positive real number. Then

A. f 0 (2) = n2n−2 .
B. f (2) (x) = n(n − 1)xn−1 .
C. f (3) (1) = n(n − 1)(n − 2)(n − 3).
D. f (n) (1) = 0.
E. f (n) (x) = n!

Section B

Question 1 [10]

(a) Use the Substitution Rule to evaluate (5)


Z π 1
8−2
tan4 (2x + 1) sec2 (2x + 1)dx.
− 12

(b) Evaluate (5)


Z
(x − 1)2 cos xdx.

Question 2 [10]
2
x 1
Let f (x) = (x − 1) 3 .
3
(a) Is the function f (x) continuous on the closed interval [0, 2]? Explain. (1)
(b) Find the absolute maximum and minimum values of f (x) on the interval [0, 2]. (5)
Mathematics I (Major) 2012–2020 Past Test and Examination Questions Booklet 41

(c) If c is a critical number of f (x), then the point with coordinates (c, f (c)) is called a critical point.
By drawing up a sign table of the first derivative, classify all the critical and end points of f (x) defined in
the interval [0, 2] as local maxima, minima or vertical tangents. (4)

Question 3 [10]

(a) Find the derivative of (tan x)cos x . (5)


1
(b) Use l’Hôpital’s rule to evaluate lim (1 + 3x) . 2x (5)
x→0

3.1.4 Calculus August 2013

Section A: Multiple choice questions

[8]
Instructions:
Section A, Questions 1–4 are multiple choice questions. In each of these questions, circle the letter that corre-
sponds to the correct answer.
Question 1 [2]
b−a
Let a = x0 < x1 < x2 < · · · < xn−1 < xn = b be a partition of [a, b], and ∆x = , where a = −2 and b = 3. Let
n
ck be any sample point in the subinterval [xk−1 , xk ], where k = 1, 2 · · · , n. Then
n
X
A. lim 3c2k ∆x = 34.
n→∞
k=1
n
X
B. lim 3c2k ∆x = 35.
n→∞
k=1
n
X
C. lim 3c2k ∆x = 36.
n→∞
k=1
n
X
D. lim 3c2k ∆x = 37.
n→∞
k=1

E. None of the above.

Question 2 [2]
Z 10 Z 10 Z 10
Suppose that u, v and w are integrable functions and u(x)dx = −3, u(x)dx = 5, v(x)dx = 4,
Z 10 Z 8 2 8 8

w(x)dx = 2 and w(x)dx = 6.


−2 −2
Which of the following is false?
Z 8
A. u(x)dx = −8.
2
Z 10
B. w(x)dx = −4.
8
Z 10
C. w(x)dx = 4.
8
Z 10 î ó
D. − 6u(x) + 5v(x) dx = −10.
8
42 2012–2020 Past Test and Examination Questions Booklet Mathematics I (Major)

E. None of the above.

Question 3 [2]
2
The average value of the function f (x) = 6x − 2x over the interval [1, 3] is:

f (3) − f (1)
A. = 24.
2
f 0 (3) − f 0 (1)
B. = 12.
2
C. 44.
D. 22.
E. None of the above.

Question 4 [2]
Which of the followings is false?
Z a Z a
A. f (x)dx = 2 f (x)dx if f is even.
−a 0
Z a Z a
B. f (x)dx = −2 f (x)dx if f is even.
−a 0
Z a
C. f (x)dx = 0 if f is odd.
−a
Z b Z c Z c
D. f (x)dx = f (x)dx − f (x)dx if f is even.
a a b

E. None of the above.

Section B

Question 1 [7]
(a) Solve cos 2x = sin 2x for x ∈ [0, π]. (2)
(b) Find the area of the region bounded above and below by the graphs y = sin 2x and y = cos 2x, where
x ∈ [0, π]. (5)

Question 2 [5]
n
(a) Simplify ln x , where n ∈ Z and x > 0. (0.5)
(b) Find the derivative of f (x) = 1 + ln x. (0.5)
(c) Determine the area of the region bounded by (4)

(3 + ln x3 )2
y= , y = 0, x = 1, x = e.
x

Question 3 [5]
(a) State the Fundamental Theorem of Calculus 1, (FTC1) with all conditions. (2)
(b) Evaluate the following derivatives: (3)
Z sec x
d 1
dt,
dx tan x 1 + t2
Mathematics I (Major) 2012–2020 Past Test and Examination Questions Booklet 43

Question 4 [4]
Find the volume of the solid generated by revolving the region between the parabola x = y2 + 2
and the line x = 5 about the line x = 5.
y

x = y2 + 2

x
1 2 3 4 5

x=5

3.1.5 Calculus October 2013

Section A: Multiple choice questions

[24]
Instructions: Section A, Questions 1–8 are multiple choice questions. In each of these questions, circle the letter
that corresponds to the correct answer.
Question 1 [3]
Which of the following is false.

dy
A. The general solution of the differential equation 2x(1 + y) − y = 0 is:
dx
x2 − y + ln |1 + y| = k.

B. The above differential equation is linear.


x
C. The differential equation y = 2ydx is separable.
d
D. The differential equation ye xy dx + xe xy dy = 0 is exact.
Å 2
y x2 y2
ã
E. The differential equation + dy − dx = 0 is homogeneous of degree 1.
x y x

Question 2 [3]
dy
The general solution of the differential equation xy − x + ln x = 0 is:
dx

x (ln x)2
A. y2 = − + c.
2 2
y2
B. = x − (ln x)2 + c.
2
y2 ln x
C. = x− + c.
2 2
44 2012–2020 Past Test and Examination Questions Booklet Mathematics I (Major)

D. y2 = 2x − (ln x)2 + c.
(ln x)2
E. y= x− + c.
2
Question 3 [3]
Let p be a non-negative integer. Which of the following is false?
X∞
1 1
A. If p
converges, then lim p = 0.
n=1
n n→∞ n

X∞
1
B. If p
diverges, then 0 ≤ p ≤ 1.
n=1
n

X 1 ∞
1
C. If lim = 0, then converges.
n→∞ n p np
n=1

X∞
1
D. If p > 1, then p
converges.
n=1
n
∞ Å
X 1 1
ã
E. − converges.
n=1
n n+1

Question 4 [3]
1
The power series representing the function is:
(x + 1)2

A. 1 − x + x2 − x3 + · · ·
B. 1 − x2 + x4 + −x6 + · · ·
C. −1 + 2x − 3x2 + 4x3 + · · ·
D. 1 + 2x + 3x2 + 4x3 + · · ·
E. 1 − 2x + 3x2 − 4x3 + · · ·

Question 5 [3]
Z ∞
1
The following improper integral dx
1 xp
A. diverges for p > 1
B. converges for p < 1
C. converges for p = 1
1
D. is equal to if p > 1
p−1
1
E. is equal to if p < 1
p−1

Question 6 [3]
Z ∞
1
The improper integral dx is equal to
3 (x − 1)2
1
A. .
2
Mathematics I (Major) 2012–2020 Past Test and Examination Questions Booklet 45

B. 1.
C. 2.
D. 3.
E. None of the above.

Question 7 [3]
Z
x x
The solution of the following integral e sin(e )dx is:

A. cos(e x )
B. − cos(e x )
C. cos(e x ) + c
D. − cos(e x ) + e x + c
E. − cos(e x ) + c

Question 8 [3]
Which of the following substitutions is appropriate for the following integral, where |x| < 2?
Z
x4
√ dx.
4 − x2
A. u = x4 .
π π
B. x = 4 sin θ, − <θ< .
2 2
π π
C. x = 2 sin θ, − < θ < .
2 2
π 3π
D. x = 2 sec θ, 0 < θ < or , π < θ < .
2 2
π 3π
E. x = 2 csc θ, 0 < θ < or , π < θ < .
2 2

Section B

[66]
Question 1 [5]
1) Give the general solution of the differential equation: (3)

dy x − 2x2
= .
dx e2y
2) Give the particular solution of the above differential equation, where the initial condition is y(0) = 2. (2)

Question 2 [11]
1) Let F1 (θ) = ln | sec θ| + c. Show that the derivative of F1 (θ) is F10 (θ) = tan θ (3)
2) Evaluate
Z
a) F2 (θ) = sec2 θ tan θdθ. (1)
Z
b) F3 (θ) = sec2 θ tan3 θdθ. (1)
46 2012–2020 Past Test and Examination Questions Booklet Mathematics I (Major)

R
3) Evaluate F(θ) = tan5 θdθ using your answers in questions a) and b) above. (6)

Question 3 [14]
Let q(x) be the polynomial function
q(x) = x3 − 3x + 2.

1) Evaluate q(1). (1)


2) Use the above result to write q(x) as a product of linear factors. (3)
3) Use the result in 2) to find the partial fractions decomposition of the rational
function (4)
9x
f (x) = ·
x3 − 3x + 2
4)
a) Find the integral (3)
Z
9x
dx.
x3 − 3x + 2
b) Evaluate and simplify your answer (3)
Z 4
9x
dx.
2 x3 − 3x + 2

Question 4 [18]
1)
a) When is a series said to be conditionally convergent? (2)
cos(nπ) (−1)n
b) Show that = , where n is a positive integer. (1)
n n
X∞
cos(nπ)
c) Is the series conditionally convergent? Justify your anwser. (4)
n=1
n
X∞ Å
5 1
ã
2) Find the sum of the series − n . (11)
n=1
n(n + 1) 3
Question 5 [10]
1
1) Find the power series representation of the function f (x) = and its radius of convergence. (4)
1+x
1
2) Give with justification the power series representation of the function g(x) = . (6)
(1 + x)3

Question 6 [8]
Z
1
1) Evaluate dx. (4)
x2 − x
2) Using your answer from Question 6-1), determine whether the following integral is
convergent or divergent (4)
Z ∞
1
2
dx.
1 x −x

3.2 Calculus 2014


3.2.1 Calculus March 2014

Question 1 [3]
Mathematics I (Major) 2012–2020 Past Test and Examination Questions Booklet 47

Let the domain of a function f be (−5, −1) and the range of f be (1, 5) \ {2}. You are given that lim − f (x) = 2 =
x→−2
lim + f (x). Here are some true or false statements about f .
x→−2

i. Since 2 is not in the range of f , lim f (x) does not exist.


x→−2

ii. lim f (x) = 2 implies f (−2) = 2.


x→−2

iii. lim f (x) = 2.


x→−2

iv. f (−2) = 2.

A. Only i is correct.
B. Only ii is correct.
C. Only iii is correct.
D. Only iv is correct.
E. iii and iv are correct.

Question 2 [3]
Let functions f and g be given by f (x) = e and g(x) = π defined on the real line. lim ( f (x) + g(x)) is given by:
x→π

A. e x because e and π are irrational numbers.


B. π because f + g approaches π.
C. e + π because this is lim π + lim e.
x→π x→π

D. Nothing because the limit does not exist.


E. None of the answers above.

Question 3 [3]
y
5

1 2 3 x

The graph above represents a function f (x). f (2) = 0 (i.e x = 2 is a zero of the function). The limit from the right
of f i.e lim+ f (x) = f (2) and lim− f (x) = 2 . Here are some true or false statements about f .
x→2 x→2

i. lim f (x) = f (2).


x→2

ii. lim f (x) = 2.


x→2

iii. lim f (x) does not exist.


x→3
48 2012–2020 Past Test and Examination Questions Booklet Mathematics I (Major)

Which of the following is true?

A. Only i is correct.
B. Only ii is correct.
C. Only iii is correct.
D. All of the statements are correct.
E. None of the statements is correct.

Question 4 [2]
2
A student wrote f (x) = loge ((x − 2) /c) = − ln c + 2 loge |(x − 2)|, where c is a positive constant and x , 2. What
makes the statement incorrect?

A. − ln c makes the statement incorrect and should be loge c.


B. The entire simplification is incorrect.
C. 2 in front of 2 loge |(x − 2)| makes the statement incorrect.
D. Nothing makes the statement incorrect hence the statement is correct.
E. 2 loge |(x − 2)| makes the statement incorrect. It should have simplified to loge (x2 /22 ) then the statement
would be correct.

Question 5 [2]
Given the graph below
y

y = f (x) = bxc
1
−2 −1 1 2
x

−2

Here are some true/false statement/s;

i. f (x) is not continuous at x = 1 because lim f (x) = 0.


x→1

ii. f (x) is continuous at x = 1 because f (1) = 1.


iii. f (x) is not continuous at x = 1 because f (x) is not defined at x = 1.
iv. f (x) is not continuous at x = 1 because lim− f (x) = 0 , f (1) = lim+ f (x).
x→1 x→1

A. Only i is correct.
Mathematics I (Major) 2012–2020 Past Test and Examination Questions Booklet 49

B. Only ii is correct.
C. Only iii is correct.
D. Only iv is correct.
E. All of the above are not true.

Total Section A: [13] marks

Section B

In this section you are expected to show all your working to earn the marks allocated.
Question 1 [4]
1
Let h(x) = csc x = . The function sin x has zeros or roots at x = nπ where n ∈ Z. Write down your answer
sin x
next to the letters A, B, C and D in the space provided below.

A. lim h(x) =
x→2π−

B. lim h(x) =
x→2π+

C. lim h(x) =
x→2π

D. Asymptote/s for h(x) is/are given by x =

Question 2 [2]
We are given that some function h(x) is defined at x = a and lim h(x) exists. In order to conclude that h is a
x→a
continuous at a, state other condition/s that must be fulfilled for h to be continuous?
Question 3 [12]
(a) Find a simpler expression for the following i. and ii. below: (4)
x
i) ln(e(e ) )
ii) e(ln πx − ln 2) for x > 0.
x3 + 1 0
(b) Directly substituting x = −1 into g(x) = we get g(−1) = . Use limit laws and factorisation to evaluate
x+1 0
lim g(x). (4)
x→−1
(c) It can be shown that the inequalities (4)

x2 x sin x
1+ < <1
6 2 − 2 cos x
hold for all values of x close to zero. What, if anything, does this tell you about

x sin x
lim ?
x→0 2 − 2 cos x
Give reasons for your answer.
Question 4 [9]
1) For what value(s) of a is x −a
if x≥0
a+1



f (x) =

x2 + a

if x<0

50 2012–2020 Past Test and Examination Questions Booklet Mathematics I (Major)

continuous everywhere? (4)


2) Determine if the one-sided limits at x = 0 exist (i.e limit from the left of x = 0 and limit from right of x = 0).
What do you conclude about the limit and continuity of f (x) at x = 0.
1 1
f (x) = − .
x |x|
(5)

3.2.2 Calculus April 2014

Section A: Multiple choice questions

Question 7 [3]
We are given that f (x) is differentiable at x = a. Here are some statements:

f (x) − f (a)
i. f 0 (a) = lim .
x→a x−a
f (a + h) − f (a)
ii. f 0 (a) = lim .
h→0 h
f (x) − f (a)
iii. f+0 (a) = lim+ .
x→a x−a
Which of the following is correct?

A. (i) is true and (ii) and (iii) are false.


B. (ii) is true and (i) and (iii) are false.
C. (i) and (ii) are true and (iii) is false.
D. All the above are true.
E. Two statements must be false and one true.

Question 8 [3]
Which of the following is incorrect?

A. If f (x) is differentiable at x = a, then it is continuous at x = a.


B. If f (x) is NOT continuous at x = a, then it is NOT differentiable at x = a.
Z
xn+1
C. For all constants n, xn dx = + C.
n+1
f (x) − f (a) f (x) − f (a)
D. f (x) is differentiable at x = a, implies lim+ = lim− .
x→a x−a x→a x−a
E. f (x) is differentiable at x = a implies f (a) exists (i.e is defined).

Question 9 [3]
Use the chain rule to decide which of the following is correct.

d 
A. f (g(x)) = f 0 (x).g0 (x).

dx
d   f 0 (x)
B. ln f (x) = .
dx f (x)
Mathematics I (Major) 2012–2020 Past Test and Examination Questions Booklet 51

d  4
C. (x + a x )−1 = −(x4 + a x )−2 .(4x3 + a x ).

dx
d  2 1
D. (sin (ln(x)) = 2 sin(ln(x)). .

dx x
d  2
E. (sin (2x) = 2 sin(2x). cos(2x).

dx
Question 10 [3]
Which of the following is true?

d  d   d 
A. f (x).g(x) = f (x) . g(x) .
 
dx dx dx
d f (x) f (x)g0 (x) − f 0 (x)g(x)
 
B.  = .
dx g(x) [g(x)]2


d 
C. a f (x) − bg(x) = a f 0 (x) − bg0 (x) where (a and b are constants).

dx
d  2
D. f (x) = 2 f (x).
dx
d  2 1
E. ln x = 2 .
dx x
Question 11 [3]
You are given y as a function of u i.e y = f (u), and u is implicitly a function of x. Which of the following is
incorrect?

d  d   du
A. f (u) = f (u) . .

dx dx dx
d u2 2
e = eu .2u.
î ó
B.
dx
d
ïZ ò
C. f (x)dx = f (x).
dx
d du
D. [sin(π − u)] = − cos(π − u) .
dx dx
d  ln u  du
E. e = .
dx dx
Question 12 [3]
For differentiable functions f (x) and g(x), which of the following is false?
Z ïZ ò Z ïZ ò
A. f (x)g(x)dx = f (x)dx g(x) − f (x)dx g0 (x)dx.

Z b Z g(b)
B. f (g(x))g0 (x)dx = f (u)du.
a g(a)

d
ïZ ò
C. f (x)dx = f (x).
dx
Z
d 
D. f (x) dx = f (x) + C.

dx
52 2012–2020 Past Test and Examination Questions Booklet Mathematics I (Major)

Z
[ f (x)]3
ïZ ò
2
E. f (x) dx = f (x)dx .

3
Question 13 [3]
Which of the following is true?
Z
A. tan x dx = sec2 x + C.
Z
B. sec x tan x dx = tan x + C.
Z
C. cos x dx = − sin x + C.
Z
D. sin x dx = − cos x + C.
Z ïZ ò2
E. x2 dx = x dx .

Question 14 [3]
d
You are given that dx [F(x)] = f (x). Which of the following is true.
Z
A. F(x) dx = f (x) + C.
Z
B. f (x) dx = F(x) + C.
Z
f 0 (x)
C. dx = F(x) + C.
f (x)
Z
f 0 (x)
D. dx = ln |F(x)| + C.
f (x)
E. None of the above.

Section B

Full working must be shown.

Question 1 [10]

3

2x + 1


if x<1
f (x) =

 2
3x if x ≥ 1.

a) Which of the two formulae above is used to find f (1)? (1)


b) Find f+0 (1) and f−0 (1). (6)
c) Use (b) to determine whether f 0 (1) exists. (1)
0
d) If f (1) exists, what is it? (1)

Question 2 [6]
Mathematics I (Major) 2012–2020 Past Test and Examination Questions Booklet 53

Use integration by parts to find


Z
e x sin x dx. (6)

3.2.3 Calculus June 2014

Section A: Multiple choice questions

[12]
Question 5 [3]
Z
arctan x dx =

A. x arctan x + ln(1 + x2 ) + c.

B. x arctan x − ln(1 + x2 ) + c.

C. x arctan x + ln 1 + x2 + c.

D. x arctan x − ln 1 + x2 + c.
1
E. + c.
1 + x2

Question 6 [3]
Z
x+4
dx =
x2 + 4

1  x
A. ln(x2 + 4) + arctan + c.
2 2
1  x
B. ln(x2 + 4) + 2 arctan + c.
2 2
1 1  x
C. ln(x2 + 4) + arctan + c.
2 2 2
 x
D. ln(x2 + 4) + 2 arctan + c.
2
E. None of the above.

Question 7 [3]
(sin x) − x
lim =
x→0 x3
A. ∞ or −∞ .

B. 1.
1
C. − .
6
1
D. .
6
E. Does not exists.

Question 8 [3]
Let f (x) = (x − 3) ln(x − 1) be defined on the interval [2, 3]. Which of the following is false?
54 2012–2020 Past Test and Examination Questions Booklet Mathematics I (Major)

A. f (x) = 0 for some x ∈ [2, 3].

B. f (x) is continuous on [2, 3].

C. f (x) is differentiable on (2, 3).

D. There exists c in (2, 3) such that f 0 (c) = 0.

E. f 0 (x) , 0 for all x ∈ (2, 3).

Section B

In this section you are expected to show all your working to earn the marks allocated.
Question 1 [7]
dî ó 1
(a) Prove that arcsin x = √ . (5)
dx 1 − x2
dî ó
(b) Find arcsin ln x , where e−1 < x < e. (2)
dx

Question 2
In this question you do not need to simplify your answers.
Use the method of logarithmic differentiation to find y0 , where: [10]
√ 6
(a) y = 5 xe x (x4 + 10)100 . (5)
sin x cos x
(b) y = . (5)
e−x x x

Question 3 [11]
(a) State the Mean Value Theorem. (4)
ex + 1
(b) Does the function f (x) = x satisfy the Mean Value Theorem in the interval [−1, 2]? Justify your answer.
e −1
(2)
(c) Use l’Hôpital’s rule to evaluate lim+ (sin x) x (5)
x→0

3.2.4 Calculus September 2014

Section A: Multiple choice questions

Question 6 [3]
n
X 5
The sum given by g(xi∗ )
approximates the area between a curve and the positive x-axis on some interval
i=1
n
125
[a, b]. We are given that the maximum value of g in the ith subinterval is g(xi∗ ) = i3 3 . The function and the
n
interval specified in the sum are:

A. g(x) = 5x3 on [0, 53 ].

B. g(x) = 53 x on [0, 5]

C. g(x) = x3 on [0, 5]

D. g(x) = x3 on [0, 53 ]

E. g(x) = (5x)3 on [0, 5]


Mathematics I (Major) 2012–2020 Past Test and Examination Questions Booklet 55

Question 7 [3]
Z b
1
The average value of a function f on an interval [a, b] is given by fave = f (x) dx. Using this information,
b−a a
1
the average value of f (t) = − tan t on [0, 2] is
2
1
A. (sec 2 tan 2 − 1).
2
1
B. sec2 2 − 0 .

2
1
C. (sec 2 tan 2 − 0).
2
1Ä p ä
D. 1 − ln | sec 2| − 0 .
2
1 p
E. − ln | sec 2|.
2
Question 8 [3]
The area formed by a continuous, non-negative function f on an interval [a, b] and the x-axis is approximated by
partitioning this interval into n subintervals [x j−1 , x j ]. Points in this subinterval a j , b j and c j where chosen such
that f (a j ), f (b j ) and f (c j ) are the minimum, midpoint and maximum value of the function in each subinterval.
The most accurate calculation of the specified area is given by

n
X b−a
A. f (a j ) .
j=1
n

n
X b−a
B. f (b j ) .
j=1
n

n
X b−a
C. f (c j ) .
j=1
n
Z b
D. f (x) dx.
a

E. An option giving a limit tending to infinity is missing hence a correct answer is missing from the options
given.

Question 9 [3]
The area formulae of simple geometric figures can be used to evaluate integrals of functions. Let


 |2x| if x<1

f (x) =

2 if 1 ≤ x ≤ 2.

Z 2
The integral f (x) dx is given by one of the following areas
0

A. Two triangles both of base length of 1 and a height of 2.

B. Two rectangles both of base length of 1 and a height of 2.


56 2012–2020 Past Test and Examination Questions Booklet Mathematics I (Major)

C. A rectangle with an area of 2 and a triangle with an area of 1.

D. A rectangle with an area of 1 and a triangle with an area of 2.

E. A rectangle with base length of 1 and a triangle with an area of 1.

Question 10 [3]
Z v(x) Z 2x
d d dt
Use the expression of f (t) dt to select the option giving .
dx u(x) dx x 1 + t2

2x
A. .
1 + x2
x 2x
B. − .
1 + (2x)2 1 + x2
x 2x
C. 2
− .
1+x 1 + x2
2 1
D. − .
1 + 4x2 1 + x2
x 2x
E. 2
+ .
1+x 1 + x2

Section B

Full workings must be shown.

Question 1 [9]
a) Evaluate the following integral (4)
Z
dx
.
2x2 − 8x + 10
b) Find the area formed by the region bounded above by x2 + 4y2 = 4 and below by the x-axis. (5)

Question 2 [6]

Find the volume of the solid generated by revolving the region bounded by y = x and the lines y = 2 and x = 0
about the line x = 4 (6)

3.2.5 Calculus November 2014

Section A: Multiple choice questions [24]

Question 1 [3]
Which of the following is false?


X (−1)n
A. converges absolutely.
n=1
n2

X sin n
B. converges absolutely.
n=1
n2

X 1
C. converges.
n=1
n ln n
Mathematics I (Major) 2012–2020 Past Test and Examination Questions Booklet 57


X cos(nπ)
D. converges.
n=1
n

X 1
E. converges.
n=1
n(n + 1)

Question 2 [3] The derivative of y = x x is

A. x(x x−1 ).

B. x + ln x x .

C. (x + ln x) x .

D. x x + x ln x.

E. x x (1 + ln x).

Question 3 [3]
x −x x −x
e +e e −e 1 + cosh 2x
Recall that cosh x = and sinh x = · The function f (x) = can be represented by:
2 2 2

A. cosh2 x.

B. sinh2 x.

C. 1 − sinh2 x.

D. sinh 2x.

E. 2 cosh 2x.

Question 4 [3]
Z
Using integration by parts twice, the solution of the integral sin(ln x) dx is:

1
A. sin(ln x) + cos(ln x) + c.
x
1 Ä ä
B. x sin(ln x) − cos(ln x) + c.
2
1 1
C. sin(ln x) − cos(ln x) + c.
2 x
1 1
D. sin(ln x) + cos(ln x) + c.
x 2
E. None of the above.

Question 5 [3]
The following improper integral
Z 1
x p dx
0

A. diverges for p > −1.

B. converges for p < −1.


58 2012–2020 Past Test and Examination Questions Booklet Mathematics I (Major)

C. converges for p = −1.


1
D. is equal to if p > −1.
p+1
1
E. is equal to if p < −1.
p+1

Question 6 [3]

1 1
A. + ·
2(1 − sin x) 2(1 + sin x)
1 1
B. + ·
1 − sin x 1 + sin x
1 1
C. − ·
1 − sin x 1 + sin x
1 1
D. − ·
2(1 − sin x) 2(1 + sin x)
−1 1
E. + ·
2(1 − sin x) 2(1 + sin x)

Question 7 [3]
2 4 6 n n 2n 1
The power series 1 − 2x + 4x − 8x + · · · + (−1) 2 x + · · · which converges for |x| < √ is the representation
2
of
ä2
1 + x2 .
Ä
A.
Ä ä2
B. 1 + x−2 .
ä−1
1 + 2x2 .
Ä
C.
ä− 12
1 + 2x2
Ä
D. .
ä− 12
2 + x2
Ä
E. .

Question 8 [3]
dy 1 x
The differential equation = e y is
dx x2 + y2

A. separable.

B. linear.

C. exact.

D. homogeneous.

E. none of the above.

Section B
Mathematics I (Major) 2012–2020 Past Test and Examination Questions Booklet 59

[66]
Question 1 [12]
1) Show that the ordinary differential equation (ODE)

(4x + y4 )dx + (4xy3 − e3y )dy = 0 (E)

is exact. (4)
2) Give the general solution of the ODE (E). (6)
1
Å ã
3) Find the particular solution satisfying y − √ = 0. (2)
3
Question 2 [13]
R k
1) Let k be a real number not equal to −1, i. e. k ∈ R \ {−1}. Evaluate cos x sin x dx. (3)
Z
cos5 x
2) Evaluate √3 dx. If you wish to, you may use your answer in Question 2(1) . (8)
sin x
Z π
2 cos5 x
3) Calculate √3 dx. (2)
0 sin x
Question 3 [14]
4 4x + 1
Let f be the rational function defined by f (x) = 3 + − ·
x + 1 x2 − x + 1
1) Find the following integrals: (Complete the square, if necessary)
Z
4
a) dx. (1)
x+1
Z
4x − 2
b) dx. (3)
x2 − x + 1
Z
3
c) 2
dx. (4)
x −x+1
Z
3(x3 − 3x + 2)
2) Use your answers in the preceding question to find dx. (6)
x3 + 1

Question 4 [13]

X
1-a) When is a series an said to be absolutely convergent? (2)
n=1
X∞
sin(2n + 1) π2
b) Is the series convergent? Justify your answer. (3)
n=1
n
X∞
sin(2n + 1) π2
c) Is the series absolutely convergent? Justify your answer. (3)
n=1
n

Question 5 [14]

X (x − 5)n
1) For which values of x does the series converge? (6)
n=1
n
x2
2) Give with justification the power series representation of the function g(x) = about the point 0. (8)
(1 + x)3
60 2012–2020 Past Test and Examination Questions Booklet Mathematics I (Major)

3.3 Calculus 2015


3.3.1 Calculus March 2015

Question 1 - 4 marks
Fill in the answers to the limits in the table:

Expressions Answers
1: lim sinθ θ
θ→0
1
2: lim x
x→−∞ 5
x
3: lim e
x→1
4: lim |y|
y→0

Question 2 - 3 marks

Figure 1: This is a(n) discontinuity.

Figure 2: This is a(n) discontinuity.


Mathematics I (Major) 2012–2020 Past Test and Examination Questions Booklet 61

Figure 3: This is a(n) discontinuity.

Question 3 - 4 marks
(a) Write down the definition of continuity of the function g(x) at x = a. (2 marks)
(b) Write down the definition of the absolute value function (|x|) in mathematical notation (i.e. not in English
words). (2 marks)

Question 4 - 9 marks
Show that (a) lim |x| = 0 (3 marks)
x→0
2
(b) Hence use the sandwich (squeeze) theorem to prove that lim(x − 1) = −1.
x→0
SHOW ALL WORKING. (6 marks)

Question 5 - 6 marks
(a) The greatest integer function (also called the floor function) is defined as

bxc = greatest integer less than or equal to x.

Give a sketch of this function for −2 < x < 4 on the axes provided. (3 marks)

(b) Evaluate the following limit algebraically (if possible). SHOW ALL WORKING. (3 marks)

limbxc =
x→2

Question 6 - 6 marks

x2 +4x+4
Find any horizontal asymptotes of the function f (x) = 3x2 −5x+1
. SHOW ALL WORKING. (6 marks)
62 2012–2020 Past Test and Examination Questions Booklet Mathematics I (Major)

Question 7 - 8 marks
Write down the intervals over which the function f (x) is continuous if

x3


 if x ≤ 0,

f (x) = 5x if 0 < x < 2,
√

x + 3 if x ≥ 2.

Express your answer in interval notation and SHOW ALL WORKING. (8 marks)

Calculus Total 40 Marks

3.3.2 Calculus April 2015

Section A: Multiple choice questions

Instructions:
Circle the correct letter for your choice of answer in this section. There is only one correct answer to each question.
Question 1 [3]
2x
The derivative of ln 3 equals

2x
A.
ln 3
B. 2x.3

C. ln 2 ln 3

D. ln 9

E. ln 3

Question 2 [3]
Z
sec kx tan kx dx equals

1
A. sec kx + C
k
B. k sec kx + C
1
C. sec kx tan kx + C
k
D. sec kx + C

E. None of the above

Question 3 [3]
Z 1
4(2x + 1)3 dx equals
0

1
A. (2x + 1)4 + C
2
1
B. (2x + 1)2 + C
2
Mathematics I (Major) 2012–2020 Past Test and Examination Questions Booklet 63

C. 40

D. 20

E. None of the above

Total Section A: [9] marks

Section B

FULL WORKINGS MUST BE SHOWN.

Question 1 [8]
If f and g are two differentiable functions in x, then

a) State the product rule i.e. the derivative of the product of f and g. (2)
b) Prove the product rule i.e. the rule that gives the derivative of the product of f and g. (6)

Question 2 [7]
Find the derivative of the following functions;
a)
sin2 (3x + 1).

(3)
b)
5x − 7
.
x3 − 3x
Do not simplify your answer. (4)

Question 3 [6]
Let g(x) = x2 e x and h(x) = sin x.
a) Find
g0 (x).

(3)
b) Find
d
(h ◦ g) .
dx
(3)

Question 4 [6]
√3
Use linear approximation to estimate the value of 8.01.
Question 5 [5]
Find
Z
cos(3x) sin5 (3x) dx.

Total Section B: [25] marks + [2] bonus marks


64 2012–2020 Past Test and Examination Questions Booklet Mathematics I (Major)

3.3.3 Calculus June 2015

Section A: Multiple choice questions

Instructions:
Section A, Questions 1–4 are multiple choice questions. In each of these questions, circle the letter that corre-
sponds to the correct answer. There is only one correct answer to each question.
Question 1 [3]
Z π
x sin x dx =
0

A. 0
π
B.
2
C. π
π2
D.
2
E. π2

Question 2 [2]
Let f be an invertible function which is differentiable. Which of the following is true?

1
A. If f ( f −1 (a)) , 0, then ( f −1 )0 (a) = .
f 0 ( f −1 (a))
1
B. If f 0 ( f −1 (a)) , 0, then ( f −1 )0 (a) = .
f 0 ( f −1 (a))
C. If f ( f −1 (a)) , 0, then ( f −1 )0 (a) = − f 0 ( f −1 (a)).
1
D. If f ( f −1 (a)) , 0, then ( f −1 )0 (a) = − .
f 0 ( f −1 (a))
1
E. If f 0 ( f −1 (a)) , 0, then ( f −1 )0 (a) = − .
f 0 ( f −1 (a))

Question 3 [2]
What is the following theorem called?
Therorem. Let f : [a, b] → R be continuous and differentiable on (a, b). Then there is a c ∈ (a, b) such that
f (b) − f (a) = f 0 (c)(b − a).

A. Fermat’s Theorem,
B. Rolle’s Theorem,
C. Taylor’s Theorem,
D. Intermediate Value Theorem,
E. Mean Value Theorem.

Question 4 [3]
Which of the following functions does not have a maximum?

A. sin x on R.
Mathematics I (Major) 2012–2020 Past Test and Examination Questions Booklet 65

1
B. on R.
1 + x2

C. x5 − 7x4 on [0, 5].

D. 1 − x2 on [−1, 1).

E. 7 − x on (1, 3].

Total Section A: [10] marks

Section B

In this section you are expected to show all your working to earn the marks allocated.
Question 1 [5]
Using logarithmic differentiation, find the derivative of

(x2 − 4)9 √ x
f (x) = e .
(x − 1)2

Do not simplify your answer.


Question 2 [10]
e x sin x − x
(a) Find lim (5)
x→0 x2
(b) Find lim+ (sin x) x (5)
x→0

Question 3 [6]
Let R be a rectangle where the lengths of the sides change with constant speed of 2 cm/sec whereas the perimeter
of R remains constant. Originally, R is a square whose area is 1 m2 . What is the rate of change of the area of R
when the shorter sides are 80 cm long?
Question 4 [15]
Let
x2 − 3x − 4
f (x) = .
x2 − 4
Then
x2 + 4
f 0 (x) = 3
(x2 − 4)2
and
x2 + 12
f 00 (x) = −6x .
(x2 − 4)3

(a) Find the x-intercepts and the y-intercepts of f . (3)

(b) Find the horizontal and vertical asymptotes of f . (3)

(c) Find the local minima and maxima of f and determine where f is increasing and where f is decreasing. (3)

(d) Find the points of inflection of f and determine where f is concave up and where f is concave down. (3)

(e) Use your answers in parts (a)–(d) to decide which sketch below is the graph of the function f . (3)
66 2012–2020 Past Test and Examination Questions Booklet Mathematics I (Major)

A. y B. y

x x

C. y D. y

x x

E. None of the above.

3.3.4 Calculus September 2015

Section A: Multiple choice questions

Instructions:
Section A, questions 1 – 4 are multiple choice questions. Circle the letter corresponding to your choice of answer.
There is only one correct answer to each question.

Question 1 [2]
Which of the following equations/identities is NOT true:

A. cosh(x + y) = cosh(x) cosh(y) + sinh(x) sinh(y).

B. cosh(x) + sinh(x) = e x .

e x + e−x
C. tanh(x) = .
e x − e−x

D. cosh(2x) = cosh2 (x) + sinh2 (x).

E. cosh2 (x) = 1 + sinh2 (x).

Question 2 [3]
1
The average value of f (t) = − tan t on [0, 2] is
2
Mathematics I (Major) 2012–2020 Past Test and Examination Questions Booklet 67

1
A. (sec 2 tan 2 − 1).
2
1
B. sec2 2 − 0 .

2
1
C. (sec 2 tan 2 − 0).
2
1Ä p ä
D. 1 − ln | sec 2| − 0 .
2
1 p
E. − ln | sec 2|.
2
Question 3 [3]
n
X 4
The sum given by g(xi∗ ) approximates the area between a curve and the positive x-axis on some interval
i=1
n
4
[a, b]. We are given that the minimum value of g in the ith subinterval is g(xi∗ ) = 2 − i . The function and the
n
interval specified in the sum are:

A. g(x) = x on [2, 4].


B. g(x) = 2 − x on [2, 4].
C. g(x) = x − 2 on [0, 4].
D. g(x) = 2 − x on [0, 4].
E. g(x) = 2 − x on [−2, 2].

Question 4 [2]
The area formulae of simple geometric figures can be used to evaluate integrals of functions. If f (x) = |x| then
Z 1
integral f (x) dx is given by one of the following areas
−2

A. Two triangles both of base length of 1 and a height of 2.


B. Two rectangles both of base length of 1 and a height of 2.
C. A rectangle with an area of 2 and a triangle with an area of 1.
D. A rectangle with an area of 1 and a triangle with an area of 2.
1
E. Two triangles with areas 2 and .
2
Total Section A: [10] marks

Section B

Full workings must be shown.


Leave answers in exact form.

Question 1 [5]

You are asked to fence off a rectangular field with area 2400m2 and partition the field into two equal parts. What
is the smallest length of fence needed?
68 2012–2020 Past Test and Examination Questions Booklet Mathematics I (Major)

Question 2 Å ã [4]
x2 dy
If y = 5 cosh , find at the point (2, 0). Simplify your answer.
4 dx

Question 3 [5]
Find the area bounded by y = 2x2 and y = x4 − 2x2 .

Question 4 [6]
Find the volume of the solid generated by revolving the region in the first quadrant bounded above by the curve
y = x2 , below by the x-axis and on the right by the line x = 1, about the line x = −1.

Total Section B: [20] marks

3.3.5 Calculus November 2015

Section A: MULTIPLE CHOICE

Question 1 - 14 marks
Circle the correct answer in the following questions. Only one answer may be circled in each question.

R
(a) What is the most suitable substitution to use to solve the integral √ 1 dt?
t2 −81

A) t = 9 sec θ.
B) t = 9 sin θ.
C) t = 81 sec θ.
D) t = 81 sin θ.
E) None of the above.
(2 marks)
R x t2
(b) The derivative with respect to x of π e cosec(t)dt is
7

2 π 2
A) e x cosec(x) − e( 7 ) cosec( π7 ).
2
B) e x cosec(x).
2
C) 2xe x ln |cosec(x) − cot(x)|.
2 2π ( π7 )2
D) 2xe x ln |cosec(x) − cot(x)| − 7 e ln |cosec( π7 ) − cot( π7 )|.
3 π 3
ex e( 7 )
E) x2
ln |cosec(x) − cot(x)| − ( π7 )2
ln |cosec( π7 ) − cot( π7 )|.
(2 marks)

(c) Odd One Out: Which of the following expressions does not represent the same sequence as the others?
©∞
A) 2(2i−1)

2i
.
i=1
B) {1, 32 , 45 , 76 , . . . }.
©∞
C) 2n+1

2 n .
n=0
¶ m+ 1 ©∞
D) 2m−12 .
m=0
E) None of the above.
Mathematics I (Major) 2012–2020 Past Test and Examination Questions Booklet 69

(3 marks)


P (− cos(nπ))n n!
(d) Which test for convergence would you use on the series n3n
?
n=1
A) The Alternating Series Test.
B) The Ratio Test.
C) The Integral test.
D) The Comparison Test.
E) The Root Test.
(2 marks)

(e) If the power series of f at a is given by



X
f (x) = cn (x − a)n
n=0

for |x − a| < R, then


f (a)
A) cn = n! .
f n (a)
B) cn = a! .
f (n) (a)
C) cn = n! (z − a)n .
f (a) (n)
D) cn = a! .
f (n) (a)
E) cn = n! .
(2 marks)

x2 +2
(f) The partial fraction decomposition of x3 −3x2 +x−3
is
11 x+3
A) x−3 − x2 +1
.
11 x−3
B) x−3 + x2 +1
.
1 11x 1 1
C) 10 x−3 − x−1 + x+1 .
1 11 1 1
D) 10 x−3 + 10 (x+1)2 .
E) None of the above.
(3 marks)

Section B: SHOW ALL WORKING!!!

Question 2 - 6 marks
Ä R x2
d 1
ä
Find dx x 2 9+t2
dt . (6 marks)

Question 3 - 8 marks
R2 1
Evaluate 1 t+t2
dt by completing the square and using trigonometric substitution. Show all working. (8 marks)
70 2012–2020 Past Test and Examination Questions Booklet Mathematics I (Major)

Question 4 - 5 marks
R
Find sin2 θ cos2 θdθ. Show all your working. (5 marks)

Question 5 - 18 marks
(a) Tick any integrals in the table that are improper: (4 marks)

Integral
R −∞ 2 Improper?
1: 0 e x dx
R0 2
2: −∞ ew dw
R 1000000000 et2
3: R0 t dt
e
4: 0 sec α dα
R 3+e
(b) Determine whether the integral 3 ln(x − 3)dx is convergent or divergent. If convergent, give the value the
R
integral converges to. HINT 1: ln xdx = x ln x − x HINT 2: l’Hôpital’s rule may be useful to you.
(8 marks)

4
(c) The following questions make use of the function f (x) = x3 −2x2 −3x
.
i) Find partial fractions of f (x). (3 marks)

ii) If you integrated your answer in Part i) from x = 0 to x = 10, indicate which of your resulting integrals
would be improper. Do NOT evaluate the integrals! (3 marks)

Question 6 - 6 marks
¶ ©
−1
(a) Find the limit of the sequence (−1)n (if it exists). (2 marks)
¶ ©
−3n
(b) Determine whether the sequence n2 −1
is increasing/decreasing or neither. (4 marks)

Question 7 - 9 marks
(a) What is an n-th partial sum? (2 marks)

P
(b) Determine whether the series √1 converges or diverges. Give a reason for your answer. (3 marks)
n≥1 n3

P 2
(c) Find the sum of the series n2 −1
. (4 marks)
n≥2

Question 8 - 9 marks

P
(a) Prove that if the series an converges, then lim an = 0. (6 marks)
n=1 n→∞

(b) What is the contrapositive of the theorem above? (3 marks)


Mathematics I (Major) 2012–2020 Past Test and Examination Questions Booklet 71

Question 9 - 4 marks
Write down the 6th degree Maclaurin polynomial of f (x) = 2 sin(3x). (4 marks)

Question 10 - 9 marks
(a) Show that cos x is the integrating factor of the linear differential equation
dy
cos x dx = 1 + y sin x (where − π2 < x < π2 ). (4 marks)

(b) Solve the differential equation from Part (a) explicitly. (3 marks)

2
(c) Find the particular solution of the above differential equation if y = 9 when x = 0. (2 marks)

Question 11 - 1 (BONUS MARK) marks


In your course books, what does CFSSG stand for?
Calculus Total 90 (+ 1 BONUS) Marks

3.4 Calculus 2016


3.4.1 Calculus March 2016

Question 1 - 9 marks
(a) Complete the sentence in words or mathematical symbols:

For the limit lim f (x) to exist... (3 marks)


x→a

(b) Fill in the answers to the limits in the table:

Expressions Answers
1
1: lim(1 + x) x
x→0
2: lim + cosec θ
θ→−π
3: lim 29−x
x→∞
4: lim− byc
y→2

(4 marks)
(c) Write down a limit which shows that a function h(x) has a vertical asymptote at x = 1.5. (2 marks)

Question 2 - 8 marks
Evaluate (and SHOW ALL WORKING)

x2 − 4
(a) lim √ . (4 marks)
x→−2 x+3−1
x2 − 4
(b) lim . (4 marks)
x→∞ 3x2 (x + 1) + 10(x + 1)
72 2012–2020 Past Test and Examination Questions Booklet Mathematics I (Major)

Question 3 - 10 marks
1 − cos θ sin θ
(a) Prove that lim = 0. You may assume that lim = 1. (5 marks)
θ→0 θ θ→0 θ

(b) Use the Sandwich (Squeeze) Theorem to determine lim sinx2 x . Clearly define your functions f (x), g(x) and
x→−∞
h(x) and explain your working. (5 marks)

Question 4 - 13 marks
(a) A function f is continuous at the point a if and only if ... (2 marks)
(b) Indicate with ‘Y’ or ‘N’ whether the functions in the following table are continuous at x = −1:

Function Y/N
1
1: f (x) = x−1
2: g(x) = |x + 1|
3: h(x) = sec(x + 1)
4: j(x) = b2xc

(4 marks)

(c) Find the value(s) of a which make(s) the function g(x) continuous if

|x − 2| + 1 if x ≤ 1
(
g(x) =
2
−x + ax − 3 if x > 1.

Show all your working. (5 marks)

(d) Draw a function with a removable discontinuity at x = 2 on the Cartesian plane below. (2 marks)

(Calculus Total 40)

3.4.2 Calculus April 2016

Question 5 - 13 marks

d
(a) Evaluate sin 4x2 + 3x . ( 2 marks)

dx

(t2 + 1)5
Å ã
(b) Let y = ln √ .
1−t
dy
Find .
dt

Hint: You may use logarithm rules. ( 4 marks)

Question 6 - 13 marks
Let f be a function. Complete the following in words or mathematical symbols:
Mathematics I (Major) 2012–2020 Past Test and Examination Questions Booklet 73

(a) (i) Left-hand derivative of f at x = a,

f−0 (a) =
( 1 mark)

(ii) Right-hand derivative of f at x = a,

f+0 (a) =
( 1 mark)

(iii) f is differentiable at a provided that · · · ( 2 marks)

(b) Consider

3x2 if x ≤ 1,
f (x) =
2x + 1 if x > 1.

Verify whether f 0 (x) exists at x = 1. ( 6 marks)

1
(c) Find an equation of the straight line having slope that is tangent to the curve
√ 4
y = x.
( 4 marks)

2
(d) Show that the graph of f (x) = x 3 has a vertical tangent at x = 0. ( 4 marks)
74 2012–2020 Past Test and Examination Questions Booklet Mathematics I (Major)

Question 7 - 13 marks

(a) Consider the equation 2xy + π sin y = 2e.

dy
(i) Find .
dx
dy  π
(ii) Find the value of at 1, .
dx 2
( 4 marks)
√ √3
(b) Find the linearization of the function f (x) = 3 x at x near 8 and use this linearization to approximate 8.12
. ( 6 marks)

Question 8 - 13 marks
Z
4 + x2
(a) Find √ dx.
x

Simplify your answer. ( 3 marks)

Z π
4
(b) Evaluate tan2 x dx. ( 3 marks)
0

3.4.3 Calculus June 2016

Section A: Multiple choice questions

Question 1 [11]
Z π
(a) esin x cos x dx = (3)
0

A. −e
B. −1
C. 0
D. 1
E. e

(b) The function f (x) = x3 + x5 − 2 is invertible. (3)


Which of the following is true?

A. f −1 is not differentiable at 0
B. ( f −1 )0 (0) = 0
C. ( f −1 )0 (0) = 8
D. ( f −1 )0 (0) = −8
1
E. ( f −1 )0 (0) =
8
Mathematics I (Major) 2012–2020 Past Test and Examination Questions Booklet 75

Z
2
(c) Evaluate x3 e x dx (5)

Section B

Question 2 [10]
Find the limits
e x − sin x − 1
(a) lim (5)
x→0 x2 cos x
(b) limπ (sin x)sec x (5)
x→ 2

Question 3 [10]
Thandi rides on her bicycle at a constant speed of 4 meters per second (14.4 km/h) towards a water tower, which
has a diameter of 20 meters. How fast does her horizontal angle of sight of the water tower increase when Thandi
is 30 meters away from the water tower?
(a) In the sketch of the aerial view below, label the quantities which you will use for your solution. (2)

(b) If 2θ is Thandi’s horizontal angle of sight of the water tower, find a relation between θ and Thandi’s distance
from the water tower. (2)
(c) Solve the problem. (6)
Question 4 [8]
(a) What is the following theorem called? (2)
Theorem. If f has a local maximum or minimum at an interior point c of dom( f ) and if f is differentiable at c,
then f 0 (c) = 0.

A. Fermat’s Theorem,

B. Intermediate Value Theorem,

C. Mean Value Theorem,

D. Rolle’s Theorem,

E. Taylor’s Theorem.

(b) Prove the theorem in case of a local maximum. (6)


Question 5 [6]
2
Find the global maximum and the global minimum of 2x − 3x 3 on [−1, 8].

3.4.4 Calculus August 2016

Full workings must be shown.


Leave answers in exact form.
76 2012–2020 Past Test and Examination Questions Booklet Mathematics I (Major)

Question 1 [5]
1
What is the lower sum approximation of the area bounded by g(x) = between x = 1 and x = 5 using 4 partitions?
x
Question 2
What are the dimensions of the rectangle with the largest area that be inscribed in the ellipse below? [6]
y axis

(x,y)

−5 5 x axis

−3

Question 3

1 1+x
Å ã
−1
Prove the following formula tanh x = ln . [5]
2 1−x
Question 4 [5]

Prove that if f is continuous on a closed interval [a, b], then there exist a c in the closed interval such that
Zb
f (x) dx = f (c)(b − a).
a

Question 5 [3]
Using areas of known geometries and other properties of integrals evaluate the integral
Z2
(2 − |x|) dx.
−2

Question 6 [10]

a. What is the total area of the region bounded by x = 4y2 − 2y and x = 12y2 ? (5)

b. What is the volume of a solid generated by rotating h(x) = x + 4 about the line y = 2 on [1, 4]? (5)

Total :[30] marks


Bonus : [4] marks
Mathematics I (Major) 2012–2020 Past Test and Examination Questions Booklet 77

3.4.5 Calculus November 2016

Section A

[10]
Z 0
dy
[MCQ1] Let y = sin (t2 ) dt. Then =

x dx

1
(A) √ sin (x2 ).
2 x
1
(B) √ sin (x).
2 x
1
(C) − √ sin (x2 ).
2 x
1
(D) − √ sin (x).
2 x

(E) 0.

( 2 marks)

Z ∞
1


1
[MCQ2] Given that the integral dx = p−1 if p > 1,
1 xp
diverges
if p ≤ 1.


Z ∞
1
Which of the following statements is true for the integral dx.
1 x−p
(A) The integral converges if p > −1.

(B) The integral diverges if p ≤ −1.

−1
(C) The integral is equal to p+1 if p < −1.

1
(D) The integral is equal to −p+1 if p < −1.

1
(E) The integral is equal to p−1 if p ≥ −1.

( 2 marks)

X x2n
[MCQ3] If cos(x) = (−1)n then
n=0
(2n)!
cos(−3x2 ) =

X (3x2 )2n
(A) (−1)n .
n=0
(2n)!

X (−3x2 )2n+1
(B) (−1)n+1 .
n=0
(2n + 1)!

X 32n x2n
(C) (−1)n .
n=0
(2n)!
78 2012–2020 Past Test and Examination Questions Booklet Mathematics I (Major)


X (−3)2n x4n
(D) (−1)n .
n=0
2(n!)

(E) None of the above.


( 3 marks)
0 0 2
[MCQ4] Consider the differential equation y − xy = 1 − y x .

It may be written in the form y0 + yP(x) = Q(x).

Which of the following statements is true?

(A) The differential equation is variable separable.


−x
(B) Q(x) = 2 .
x +1
1
(C) P(x) = .
x2 + 1

(D) The differential equation is linear.


R 1
dx
(E) Its integrating factor is IF = e x2 +1 .

( 3 marks)
Mathematics I (Major) 2012–2020 Past Test and Examination Questions Booklet 79

Section B

Question 1 - 13 marks
Evaluate the following integrals.
Z
3x2 − x + 1
(a) dx. ( 8 marks)
x(x2 + 1)

Z
x
(b) √ dx. ( 10 marks)
3 − 2x − x2

[18]

Question 2 - 13 marks

2x2 4x 4 + 12x2
Let f (x) = 2
, with f 0 (x) = 2 2
and f 00 (x) = .
1−x (1 − x ) (1 − x2 )3

(a) Find the domain of f . (2 marks)

(b) Find the x and y intercepts (if any). (1 mark)

(c) Describe the behaviour of f as x tends to ∞ and −∞. (2 marks)

(d) Describe the behaviour of f as x tends to 1+ , 1− , −1+ and −1− . (2 marks)

(e) Find the equation(s) of the asymptote(s) (if any). (2 marks)

(f) Find the coordinates of the local extrema. (2 marks)

(g) Find intervals of increase and decrease of f . (4 marks)

(h) Determine local maxima and local minima. (1 mark)

(i) Determine concavity of the curve. (3 marks)

2x2
(j) Sketch the curve of f (x) =
1 − x2
indicating point(s) of intersection (if any), asymptotes and local extrema. (4 marks)

[23]
80 2012–2020 Past Test and Examination Questions Booklet Mathematics I (Major)

Question 3 - 13 marks
Determine whether the following improper integrals converge or diverge. Show all your working.
Z ∞
4
(a) dx. (3)
0 x
Z ∞
4
(b) 2+2
dx. (5)
0 x

[8]

Question 4 - 13 marks
∞ Ä
X 4n − 3 än
(a) Determine whether the series converge or diverge. (5)
n=1
3n − 4

X (x − 3)n
(b) For which value of x does the series (−1)n converge? (7)
n=1
n!

[12]

Question 5 - 13 marks
Consider the differential equation (x2 − y2 )dx + x(x + 2y)dy = 0.

(a) Show that the differential equation is homogeneous. (2)

(b) Find the general solution of the differential equation. (15)

(c) Find the particular solution for x = 1 and y = −1. (2)


[19]

3.5 Calculus 2017


3.5.1 Calculus March 2017

3.5.2 Calculus April 2017

Section A [8 marks]

Instructions:
For each multiple choice question, select the letter that corresponds to the correct answer.

[A1] The normal to the curve y = 3 tan x + x2 + 1 at the point with coordinates (0, 1) is
(A) y = −3x + 1.
(B) y = 3x − 1.
1
(C) y = x − 1.
3
1
(D) y = − x + 1.
3
Mathematics I (Major) 2012–2020 Past Test and Examination Questions Booklet 81

1
(E) y = x + 1. 2 marks
3
dy
[A2] If y = log10 (sin x) ; 0 < x < π, then =
dx
1
(A) ln 10 .
sin x
1
(B) .
ln 10 sin x
1
(C) cot x.
ln 10
1
(D) .
ln 10 cot x
1
(E) . 2 marks
ln 10 cos x
[A3] Let f and g be differentiable functions. Which of the following is equal to

d
( f (x + g(x)))?
dx
(A) f 0 (x) + f 0 (g(x))g0 (x).

(B) f 0 (x + g(x))g0 (x).

(C) f 0 (x + g(x))(x + g0 (x)).

(D) f 0 (x) + g0 (x)(1 + g0 (x)).

(E) f 0 (x + g(x))(1 + g0 (x)). 2 marks


Z Å
1
ã
[A4] Which of the following is equal to + eπ dx?
7x
2
(A) − + C.
7x2
1
(B) ln |7x| + eπ+1 x + C.
π+1
ln |x|
(C) + π eπ x + C.
7
ln |x|
(D) + eπ x + C.
7
x
(E) ln + eπ x + C.
7
2 marks

Section B [32 marks]

Instructions:
In this section you are expected to show all your working to earn the marks allocated.
Simplify your answers.
82 2012–2020 Past Test and Examination Questions Booklet Mathematics I (Major)

Question 1 - 13 marks
(a) Let y = xr for r ∈ R.

dy
Prove that = rxr−1 . 4 marks
dx
Ç √
x
√ å
d 5 +x−4 x
(b) Find . 3 marks
dx x

Question 2 - 13 marks

2
(a) Consider the equation x4 + cos y + e x y = x2 + y2 .

dy
Find . Show all your working. 6 marks
dx

(b) Find the differential dy if y = πt tπ . 2 marks

Question 3 - 13 marks

(a) Find the equation of the tangent line to the curve of


x
f (x) = at x = 1. 4 marks
x+1
(b) For what value(s) of a does the tangent line to the curve pass through
the point (−5, a ) ? 3 marks

(c) Use linearization to approximate the value of f (1.2). 2 marks

Question 4 - 13 marks
Z 2
x−1
Evaluate √ dx. 8 marks
−1 x+2

3.5.3 Calculus June 2017

Question 1 - 4 marks
TRUE or FALSE:

Statement True/False
R R
If s0 (t) and r0 (t) are continuous then s0 (t)r(t)dt = r(t)s(t) − s(t)r0 (t)dt.

If f is continuous on an interval I, then f −1 exists and is also continuous on I.

The function g has an absolute minimum at c if g(c) ≥ g(x) ∀x in the domain of g.

r0 (s) = 0 ⇒ the point (s, r(s)) is a local extremum of the function r(s).
Mathematics I (Major) 2012–2020 Past Test and Examination Questions Booklet 83

Question 2 - 4 marks
R
(a) Write in the block how many times you need to use integration by parts in order to integrate eu cos u du?
DO NOT do the integration! (1 mark)

(b) Evaluate, showing all working: (3 marks)

Z 1
1

3
dt.
0 e − t2

Question 3 - 7 marks
d x
(a) Show all working to find dx x . 4 marks
2t3 −t2 −t+1
(b) Show all working to find the slant asymptote of h(t) = t2 −1
. (3 marks)

Question 4 - 4 marks
(a) Find the 3rd degree Taylor polynomial of sin x at x = 0. SHOW ALL YOUR WORKING. 3 marks

(b) Differentiate your answer above to write down the 3rd degree Taylor polynomial of cos x at x = 0. (1 mark)

Question 5 - 9 marks
(a) Prove the following Theorem: 5 marks
If f 0 (x) = g0 (x) for all x in an interval (a, b), then f − g is a constant on (a, b), that is, f (x) = g(x) + C,
where C is a constant.

(b) State l’Hôpital’s Rule in full. Hint: Start with “Let f and g be differentiable functions...”. 4 marks

Question 6 - 6 marks
Sketch a graph on the axes provided which satisfies the following criteria:

(a) f (x) is continuous for all positive real numbers.

(b) f (x) < 0 for x ∈ (0, 2) and f (x) > 0 for x ∈ (2, ∞)

(c) f 0 (x) > 0 for x < 5 and f 0 (x) < 0 for x > 5

(d) f 0 (x) = 0 when x = 5

(e) f 00 (x) < 0 for 0 < x < 7 and f 00 (x) > 0 for x > 7

(f) lim+ = −∞
x→0
84 2012–2020 Past Test and Examination Questions Booklet Mathematics I (Major)

Question 7 - 6 marks

You have 40cm of wire to form a square and a circle. How much of the wire should be used for the square and
how much for the circle to enclose the maximum total area?
Calculus Total 40

3.5.4 Calculus August 2017

Question 1 - 6 marks

TRUE or FALSE:
Mathematics I (Major) 2012–2020 Past Test and Examination Questions Booklet 85

Statement True/False

The derivative of cosh x is − sinh x.

For a decreasing function, the right-most endpoint will give the height for the

lower Riemann sum.

d
R x2 2
dx −5 e2t dt = 2xe2x

Question 2 - 7 marks
(a) State and prove the Fundamental Theorem of Calculus 2. (7 marks)

Question 3 - 7 marks
(a) Find the area of the region bounded by the graphs y = x2 and y = c in terms of the positive constant c. (4
marks)
(b) Write down an expression for the area enclosed by x = cos y, x = 12 , y = 0 and y = 2π. DO NOT evaluate
your integral. (3 marks)

Question 4 - 7 marks
(a) Write down an expression for the volume of the solid generated by rotating the region bounded by y = 1x + 2
and y = −x + 10 about the line y = 0. DO NOT evaluate your integral. (5 marks)
(b) Write down an expression for the volume of the solid generated by rotating the region bounded by y = 1x + 2
and y = −x + 10 about the line y = 10. DO NOT evaluate your integral. (2 marks)

Question 5 - 3 marks
Find the cross-sectional area of a pyramid with a height of h and a rectangular base with dimensions w and b,
sliced perpendicular to the y-axis.
Calculus Total 30 Marks

3.5.5 Calculus November 2017

Section A:
Multiple choice questions [18 marks]

Instructions:
Questions MCQ1–MCQ6 are multiple choice questions.

In each of these questions, select the letter that corresponds to the correct answer.
86 2012–2020 Past Test and Examination Questions Booklet Mathematics I (Major)

t4 + 9
(MCQ1) Which of the following is the appropriate partial fraction decomposition of :
t2 (t2 + 9)
t4 + 9 A Bt + C
(A) 2 2
≡1+ 2 + 2
t (t + 9) t t +9
t4 + 9 At + B C D
(B) 2 2
≡ 2
+ +
t (t + 9) t t+3 t−3
t4 + 9 A B Ct + D
(C) 2 2
≡1+ + 2 + 2
t (t + 9) t t t +9
t4 + 9 A B C D
(D) 2 2
≡ + 2+ +
t (t + 9) t t t+3 t−3
t4 + 9 A B Ct + D
(E) 2 2
≡1+ + 2 + 2 . [3]
t (t + 9) t t t +9
(MCQ2) Which of the following statements is NOT TRUE?
Z 3
1 3
(A) 2
dx = .
0 (x − 4) 4
Z 3 Z 2 Z 3
1 1 1
(B) 2
dx = 2
dx + 2
dx.
0 (x − 4) 0 (x − 4) 2 (x − 4)
Z 3
1
(C) 2
dx has an interior infinite discontinuity.
0 (x − 4)
Z 3
1
(D) 2
dx converges.
0 (x − 4)

1
(E) The integrand is defined on (0 , 3) . [3]
(x − 4)2
Mathematics I (Major) 2012–2020 Past Test and Examination Questions Booklet 87


X cos (nπ)
(MCQ3) Consider the series .
n=1
n
Which of the following statements is TRUE?

(A) The series is not conditionally convergent.

(B) The series does not converge.

(C) The series is absolutely convergent.

cos (nπ)
ß ™
(D) The sequence diverges.
n

X X∞
cos (nπ) 1
(E) = (−1)n . [3]
n=1
n n=1
n

(MCQ4) Which of the following statements is TRUE about the series



X
ln x + (ln x)2 + (ln x)3 + · · · = (ln x)n .
n=1

1
(A) The series converges for < x < e.
e
1
(B) The series converges for ≤ x ≤ e.
e

(C) The series does not converge for any value of x.

an+1 1
(D) = 1 + → 1 as n → ∞.
an n

(E) The series is convergent. [3]

(MCQ5) Let g(x, y) = x2 y + cos y + y sin x.

Which of the following statements is TRUE?


(A) g x = 2xy + cos y + ycos x.

(B) gy = 2x − sin y + sin x.

(C) g x x = 2y − sin y + sin x.

(D) g x y = gy x .

(E) gy y = 2x − cos y. [3]


88 2012–2020 Past Test and Examination Questions Booklet Mathematics I (Major)

(MCQ6) Consider the differential equation (2xy − yn ) dx + x2 dy = 0.

Which of the following statements is NOT TRUE?

(A) For n = 1 the differential equation is variable separable.

(B) For n = 2 the differential equation is homogeneous of degree 2.

(C) For n = 0 the differential equation is exact.

(D) For n = 0 the differential equation is variable separable.

(E) For n = 1 the differential equation is first order linear. [3]

Section B [72 marks]

In this section you are expected to SHOW ALL YOUR WORKING on the provided space to earn the
allocated marks.

Question 1 - 3 marks

1
(a) Find the partial fraction decomposition of . [3]
z (1 + z)
Z
ln z
(b) Evaluate dz. [7]
(1 + z)2

Question 2 - 3 marks
Evaluate the following integrals:
Z
t
(a) √ dt.
t4 −1
Z
Hint : You may use the fact that sec θ dθ = ln |sec θ + tan θ| + C. [9]

Z 2
2s − 2
(b) √ ds. [5]
1 −s2 + 2s + 3

Question 3 - 3 marks
Z
1
(a) Evaluate dx. [4]
(1 + x2 )arctan x
Z 1
1
(b) Hence or otherwise determine whether the integral dx
0 (1 + x2 )arctan x
is convergent or divergent. [4]
Mathematics I (Major) 2012–2020 Past Test and Examination Questions Booklet 89

Question 4 - 3 marks
X∞ Å
1 2 1
ã
Consider the telescoping series − + .
r=1
r+2 r+1 r

Xn Å
1 2 1
ã
(a) Find the partial sum S n = − + . [5]
r=1
r+2 r+1 r

X∞ Å
1 2 1
ã
(b) Hence or otherwise show that the series − +
r=1
r+2 r+1 r
converges and find its sum. [3]

Question 5 - 3 marks

n3 − 1 + n2 sin n
ß ™
(a) Determine, giving reasons, whether the sequence {an } =
1 + 3n3
is convergent or divergent.

If it is convergent, find the value to which it converges. [4]

(b) Determine whether the following series converge or diverge



X n3 − 1 + n2 sin n
(i) . [2]
n=1
1 + 3n3
X∞ Å 3 2 ãn
n n − 1 + n sin n
(ii) (−1) . [4]
n=1
1 + 3n3

Question 6 - 3 marks
X ∞
1
Given that = 1 + x + x2 + x3 + · · · = xn .
1−x n=0

1
(a) Write down the power series of . [4]
1 + x2

(b) Hence or otherwise find a power series representation of f (x) = arctan x. [4]

π
(c) Write as a series. [2]
4

Question 7 - 3 marks
Ä ä2 Ä ä2
x x
dx y2 + y2 e y + 2x2 e y
Consider the differential equation = .
dy
Ä ä2
x
2x y e y
(a) Using the substitution x = vy or otherwise, find the general solution of the
differential equation. [10]
90 2012–2020 Past Test and Examination Questions Booklet Mathematics I (Major)

(b) Find the particular solution of the differential equation given that y = 1 when x = 0. [2]

3.5.6 Calculus Diagonal D March 2018

Question 1 - 4 marks
TRUE or FALSE:
Let c ∈ R and a ∈ R.

Statement True/False

lim |x| = 1.
x→1

lim bxc = 1
x→1−

lim c = c
x→c

Let lim f (x) = L and lim g(x) = M both exist, for L, M ∈ R.


x→a x→a

f (x) L
Then lim = M
x→a g(x)

Question 2 - 3 marks
−1
(a) Determine whether the function f (x) = x2
has a vertical asymptote at x = 0. Show all your working. (3
marks)

Question 3 - 8 marks
Prove that
1 − cos θ
lim = 0.
θ→0 θ
You may assume that lim sinθ θ = 1. (8 marks)
θ→0

Question 4 - 9 marks
(a) Complete the sentence:
The function g(x) is continuous on the interval [−1, 3) if and only if.... (2 marks)
Mathematics I (Major) 2012–2020 Past Test and Examination Questions Booklet 91

(b) For which value(s) of a ∈ R is the function


 2
x + a


for x<2
f (x) = 8 for x=2

 x3

for x>2

continuous on its entire domain?


Show your working and explain your reasoning. (6 marks)
(c) If a = 5 in part (b) above, then what kind of discontinuity exists at x = 2? (1 mark)

Question 5 - 7 marks

Use first principles to find f 0 (2) if f (x) = x2 + 3.

Question 6 - 4 marks
Determine whether the function f (x) = |2x − 5| has a vertical tangent at 25 . Explain why you came to that
conclusion.

Question 7 - 2 marks
(a) Is the function g(x) = tan x + 3 continuous at x = π2 ?
(b) Using part (a) above, what can you say about the differentiability of tan x + 3 at x = π2 ?

Question 8 - 3 marks
y
6
-x

The function g(x)

Above you are given the graph of the function g(x). Circle the letter of ONE of the graphs below which gives the
most accurate sketch of the derivative of g(x).
y y
6 6
-x -x

A: The function g0 (x)B: The function g0 (x)


y y
6 6
-x -x

C: The function g0 (x)D: The function g0 (x)

Calculus Total 40 Marks


92 2012–2020 Past Test and Examination Questions Booklet Mathematics I (Major)

3.5.7 Calculus Diagonal E March 2018

Question 1 - 4 marks
TRUE or FALSE:

Let c ∈ R.

Statement True/False

If lim− f (x) = 5 and lim+ f (x) = −5 then lim f (x) = |5|.


x→1 x→1 x→1

limbxc = 4
x→4

lim 0 = 0
x→c

If n ∈ N then lim xn = cn
x→c

Question 2 - 4 marks
Evaluate the limit
3x
lim √ .
x→0 1 − 2x − 1

Question 3 - 3 marks
1
Determine whether the function f (x) = x has a vertical asymptote at x = 0. Show all your working.

Question 4 - 9 marks
(a) Complete the sentence:
The function g(x) is continuous from the left at c if.... (2 marks)
(b) For which value(s) of a ∈ R is the function
 3

 x for x<1

f (x) = a for x=1

 x2

for x>1

continuous on its entire domain?


Show your working and explain your reasoning. (6 marks)
(c) If a = 5 in part (b) above then what kind of discontinuity exists at x = 1? (1 mark)
Mathematics I (Major) 2012–2020 Past Test and Examination Questions Booklet 93

Question 5 - 7 marks
2
Use first principles to find f 0 (2) if f (x) = x2 +3
.

Question 6 - 4 marks
A function f (x) has a vertical tangent at a provided that which three conditions hold?

Question 7 - 6 marks
Prove that if a function f (x) is differentiable at x = a, then f (x) must be continuous at x = a.
HINT: begin as follows:
If x is in the domain of f and x , a, then f (x) may be written as
f (x) − f (a)
Å ã
f (x) = f (a) + (x − a).
x−a

Question 8 - 3 marks
y
6

-x

The function g(x)

Above you are given the graph of the function g(x). Circle the letter of ONE of the graphs below which gives the
most accurate sketch of the derivative of g(x).
y y
6 6
-x -x

A: The function g0 (x)B: The function g0 (x)


y y
6 6
-x -x

C: The function g0 (x)D: The function g0 (x)

Calculus Total 40 Marks

3.5.8 Calculus Diagonal D April 2018

Instructions:
You are expected to show all your working to earn the marks allocated.

Total marks [40 marks]

Question 1 - 3 marks
Find the linearization of the function f (x) = arctan(x) near 1 and use this linearization to approximate arctan(1.01)
94 2012–2020 Past Test and Examination Questions Booklet Mathematics I (Major)

Leave your final answer as a fraction. 6 marks

Question 2 - 3 marks
Let x2 y = 1 + xy2 . Find y0 . 5 marks

Question 3 - 3 marks
√ 1
Let f (x) = c x − log10 x. For what value(s) of c is f 0 = 10? 4 marks

2

Question 4 - 3 marks
x

Let g(x) = x arcsin 16 − x2 .

4 +

Find g0 (2).

Simplify your answer. 7 marks

Question 5 - 3 marks
Z
1
Show that dx = ln |x| + C. 5 marks
x

Question 6 - 3 marks
Evaluate the following integrals:
Z
2t − 2
(a) √ dt. 4 marks
−t2 + 2t + 3
Z
4 + z2
(b) √ dz. 4 marks
z
Z 2
(c) (x − 1)e(x−1) dx. 5 marks
0

3.5.9 Calculus Diagonal E April 2018

Total marks [40 marks]

Question 1 - 3 marks
d x
Show that d x (e ) = ex . 5 marks

e x −1
Hint: You may use the fact that lim = 1.
x→0 x

Question 2 - 3 marks
Let f (x) = logb (3x2 − 2) where b is a positive real number different from 1.

For what value(s) of b is f 0 (1) = 3? 4 marks


Mathematics I (Major) 2012–2020 Past Test and Examination Questions Booklet 95

Question 3 - 3 marks
π x2
Å ã
Find the equation of the tangent line to the curve y = 2tan at the point (1, 2). 6 marks
4

Question 4 - 3 marks
dy
Find if arctan(x + y) = xy2 . 6 marks
dx

Question 5 - 3 marks
Let f (x) = cos x.

π
Find the third degree Taylor polynomial of f (x) at x = . 6 marks
3

Question 6 - 3 marks
Evaluate the following integrals:
Z 2
x+2
(a) √ dx. 6 marks
1 x2 + 4x
Z

(b) x ln x dx. 7 marks


3.5.10 Calculus June 2018

Full workings must be shown.


Leave answers in exact form.
96 2012–2020 Past Test and Examination Questions Booklet Mathematics I (Major)

Question 1 [4]

x
Differentiate y = x .
Question 2 [3]

x ∂f ∂f
Å ã
If f (x, y) = sin , calculate and .
1+y ∂x ∂y
Question 3 [4]

Evaluate lim (x − ln x).


x→∞
Question 4 [16]

Let f (x) = (4 − x2 )5 .
(a) Where does f cut the x-axis? (1)
(b) Where does f cut the y-axis? (1)
(c) What are the critical number(s) of f ? (3)
(d) State the open interval(s) where f is increasing, decreasing or constant. (3)
(e) If f has local extrema, give point/s at which they occur and specify if the local extrema is a minimum or
maximum. Otherwise, state if it has none. (2)
(f) State the open interval(s) where f is concave up or concave down and point(s) of inflection. (6)
Question 5 [6]

State and prove Fermat’s theorem.


Question 6 [7]

A conical tank 8 meters across the top and 6 meters deep is full of water. The tank springs a leak at the bot-
tom and loses water at a rate of 2 cubic meters per min. How fast is the water level dropping at the instance when
the water is exactly 3 meters deep?

Total :[40] marks


Mathematics I (Major) 2012–2020 Past Test and Examination Questions Booklet 97

3.5.11 Calculus Diagonal D August 2018

Section A: 8 marks.
MULTIPLE CHOICE QUESTIONS

Circle the correct answer of your choice.

Question 1. sinh x = [2]

A) e x + e−x
B) sin x
C) e x − e−x
e x +e−x
D) e x −e−x
E) none of the above

Question 2. Which of the following statements is true? [2]

A) sinh(−x) = sinh x
B) cosh(−x) = cosh x
C) sinh2 x + cosh2 x = 1
sin x
D) tanh x = cos x
E) tanh(−x) = tanh x

Question 3. The area of the region bounded by x = 1, y = 27, and y = x3 equals [2]

A) 12
B) 20
C) 32
D) 34
E) none of the above

Question 4. Which of the following statements is true? [2]


Rb Ra
A) a
f (x)dx = f (x)dx
b
Rb Rb
B) a
f (−x)dx = − a f (x)dx
Rb Ra Rb
C) a
f (x)dx = a f (x)dx + b f (x)dx
Rb Rb
D) | a f (x)dx| = a | f (x)|dx
E) none of the above
98 2012–2020 Past Test and Examination Questions Booklet Mathematics I (Major)

Section B: 22 marks.

Question 1. State the definition of a definite integral. [4]

Question 2. Compute the Lower Riemann Sum for the function f (x) = 2x2 − 1 on [1, 4], n = 3. [4]
R4
Question 3. Compute 1 (2x + 1)dx using Riemann Sums. [7]

Question 4. Find the volume of the solid generated by revolving the region bounded by the y-axis and x + y2 − y = 0
about the y-axis. [7]
Mathematics I (Major) 2012–2020 Past Test and Examination Questions Booklet 99

3.5.12 Calculus Diagonal E August 2018

Section A: 8 marks.
MULTIPLE CHOICE QUESTIONS

Circle the correct answer of your choice.

d
Question 1. dx (sinh x) = [2]

A) e x + e−x
B) cos x
e x −e−x
C) 2
e x +e−x
D) 2
E) none of the above

Question 2. Which of the following statements is true? [2]

A) sinh(−x) = −sinh x
B) cosh(−x) = −cosh x
C) sinh2 x − cosh2 x = 1
sin x
D) tanh x = cos x
E) tanh(−x) = tanh x

Question 3. The area of the region bounded by x = 1, y = 2, x = −1, and y = 3x2 + 4 equals [2]

A) 2
B) 4
C) 6
D) 8
E) none of the above

Question 4. Which of the following statements is true? [2]


Rb Ra
A) a
f (x)dx = f (x)dx
b
Rb Rb
B) a
f (−x)dx = − a
f (x)dx
Rb Ra Rb
C) a
f (x)dx = a
f (x)dx + b
f (x)dx
Rb Ra
D) a
f (x)dx + b f (x)dx = 0
Rb Rb
E) | a f (x)dx| ≥ a | f (x)|dx
Mathematics I (Major) 2012–2020 Past Test and Examination Questions Booklet 1

Section B: 22 marks.

Question 1. State the definition of a Riemann Sum. [4]

Question 2. Compute the Upper Riemann Sum for the function f (x) = − 1x on [−4, −1], n = 3. [4]
R4
Question 3. Compute −1 (3x2 + 1)dx using Riemann Sums. [7]

Question 4. Find the volume of the solid generated by revolving the region bounded by x2 + 1 − y = 0, x = 0, x = 1, and
−2x + 2y = 1 about the line y = 2. [7]

3.5.13 Calculus November 2018


MATH1036 NOVEMBER EXAM 2018
Section A:
[12 marks]

Instructions:
Answer Questions A1–A6 with True T or False F.
In each of the questions, select one choice by encircling the letter T or F.
Each question is worth 2 marks.

Z 0
1
(A1) Let g be a function defined by g(x) = dt.
−x 1 + t2
0 1
Then g (x) = − . T F
1 + x2

Z 0
2x
(A2) The integral dx is an improper integral. T F
−5 1 − x2


X ∞
X
cos (nπ) 1
(A3) = (−1)n . T F
n=1
n n=1
n

X∞
1
(A4) If = 1 + x + x2 + x3 + · · · = xn , then
1−x n=0
X∞
1 2 4 6
= 1 + x + x + x + · · · = x2n . T F
1 + x2 n=0

(A5) Let g(x, y) = x2 y + cos y + y sin x.

Then gy = 2x − sin y + sin x. T F


2 2012–2020 Past Test and Examination Questions Booklet Mathematics I (Major)

Z
sin 18 t
(A6) dt = 2sin 9t + C. T F
sin 9t

Section B [78 marks]

In this section you are expected to SHOW ALL YOUR WORKING on the
provided space to earn the allocated marks.

Question 1 - 3 marks
Z t2
2
Consider the function g(t) = (x5 − 4 · 3 x ) dx.
−t

Find g0 (t). [4]

Question 2 - 3 marks
Evaluate the following integrals:
Z
2t − 3
(a) √ dt. [7]
4 − t2
Z
sin3 x cos4 x
(b) √ dx. [6]
cos x

Question 3 - 3 marks
2u2
(a) Express in its partial fraction decomposition. [8]
(u − 1)2 (u + 1)2
Z √
x+1
(b) Hence, or otherwise, evaluate dx.
x2
[7]

Question 4 - 3 marks
Z ∞
1
Determine whether the improper integral dx converges or diverges. [4]
1 x3

Question 5 - 3 marks


X
(a) Prove that if the series an converges, then lim an = 0. [6]
n→∞
n=1

3n
(b) Consider the sequence {an }∞
n=1 where an = .
4n − 1

X
Does the series an converge?
n=1
Justify your answer. [2]
Mathematics I (Major) 2012–2020 Past Test and Examination Questions Booklet 3

1
(c) Consider the sequence {an }∞
n=1 where an = 3
.
n +1
2

X
Does the series an converge?
n=1
Justify your answer. [4]

Question 6 - 3 marks


X 1
(a) Show that the series converges to 1. [8]
r=1
r(r + 1)

X
(b) When is a series an said to be absolutely convergent? [2]
n=1

X∞
sin((2n + 1) π2 )
(c) Show that the series is absolutely convergent.
n=1
n2
Justify your answer. [4]

Question 7 - 3 marks

X (x − 2)n
For which values of x does the series converge?
n=1
n 2n
Justify your answer. [8]

Question 8 - 3 marks
Find the general solution of the differential equation

dy π π
= y tan x + sec x, − <x< . [8]
dx 2 2
4 2012–2020 Past Test and Examination Questions Booklet Mathematics I (Major)

4 Calculus Solutions
4.1 Calculus Solutions 2013
4.1.1 Calculus Solutions April 2013

Section B

Question 2
2)
(a)
(e x + e−x )5 x ln 5 − 5 x (e x − e−x )
f 0 (x) =
1 2
Å ã
ex + x
e
(b)
3
g0 (x) = 3x2 e x +ln 9

3
= 3x2 eln 9 e x
3
= 3x2 × 9e x
3
= 27x2 e x

Question 3
(a)

d  xy d
e + y2 cos x + e x = [6]

ïdx dx
d dy
ò
⇔ e xy . [xy] + 2y (cos x) − y2 sin x + e x = 0
dx dx
dy dy
ï ò ï ò
xy 2 x
⇔e . y+x + 2y (cos x) − y sin x + e = 0
dx dx
xy xy dy dy
⇔ ye + xe + 2y (cos x) − y2 sin x + e x = 0
dx dx
dy  xy
⇔ xe + 2y cos x = y2 sin x − e x − ye xy

dx
(4.1)

Hence,
dy y2 sin x − e x − ye xy
= .
dx xe xy + 2y cos x
(b)
dy 22 sin 0 − e0 − 2e0×2 0 − 1 − 2 3
|(0,2) = 0×2
= =− .
dx 0e + 2 × 2 cos 0 0+4 4
(c) Let mnormal be the gradient of the normal to the curve at (0, 2). Then

dy 3
mnormal . |(0,2) = −1 ⇔ − .mnormal = −1
dx 4
4
⇔ mnormal = .
3
Mathematics I (Major) 2012–2020 Past Test and Examination Questions Booklet 5

Hence the equation of the normal line to the curve at (0, 2) is


4 4
y−2= (x − 0) ⇔ y = x + 2.
3 3

Question 4
1 1 1
(b) f (1) = 1. f 0 (x) =
√3 . Thus f 0 (1) = .
3 x2 3
Hence the linear approximation of the function f (x) at x = 1 is:

L(x) = f (1) + f 0 (1)(x − 1)


1
= 1 + (x − 1).
3

(c) It follows from (b) that


1
L(1.0006) = 1 + (1.0006 − 1)
3
0.0006
=1+ = 1 + 0.0002
3
= 1.0002.
√3
Hence 1.0006 ≈ 1.0002.

4.1.2 Calculus Solutions June 2013

Section B

Question 2
(b) The function f (x) is differentiable on the open intervals (0, 1) and (1, 2) and is not differentiable at x = 1.
Hence for x ∈ (0, 1) ∪ (1, 2) we have:
2 1 1 2
f 0 (x) =x(x − 1) 3 + x2 (x − 1)− 3
3 ñ 9
√3
ô
1 x
= x 6 x−1+ p
9 3
(x − 1)2
1 x
= (6(x − 1) + x)
9 (x − 1)2
3
p

1 x
= p (7x − 6)
9 (x − 1)2
3

∴ f 0 (x) = 0 ⇔ x = 0 or x = 67 .

6 12 2
Now, f (0) = 0 and f ( ) = − √3 . Also f (1) = 0 and f (2) = .
7 49 7 3
Hence the absolute minimum and absolute maximum values of f (x) on the interval [0, 2] are
6 12 2
f ( ) = − √3 and f (2) = respectively.
7 49 7 3

4.1.3 Calculus Solutions June 2013

Multiple Choice: D;C;E;D;E


6 2012–2020 Past Test and Examination Questions Booklet Mathematics I (Major)
Mathematics I (Major) 2012–2020 Past Test and Examination Questions Booklet 7
8 2012–2020 Past Test and Examination Questions Booklet Mathematics I (Major)
Mathematics I (Major) 2012–2020 Past Test and Examination Questions Booklet 9
10 2012–2020 Past Test and Examination Questions Booklet Mathematics I (Major)

4.1.4 Calculus Solutions August 2013

Multiple Choice: B;C;D;B


Mathematics I (Major) 2012–2020 Past Test and Examination Questions Booklet 11
12 2012–2020 Past Test and Examination Questions Booklet Mathematics I (Major)

4.2 Calculus Solutions 2014


4.2.1 Calculus Solutions March 2014
Mathematics I (Major) 2012–2020 Past Test and Examination Questions Booklet 13
14 2012–2020 Past Test and Examination Questions Booklet Mathematics I (Major)
Mathematics I (Major) 2012–2020 Past Test and Examination Questions Booklet 15
16 2012–2020 Past Test and Examination Questions Booklet Mathematics I (Major)
Mathematics I (Major) 2012–2020 Past Test and Examination Questions Booklet 17

4.2.2 Calculus Solutions April 2014


18 2012–2020 Past Test and Examination Questions Booklet Mathematics I (Major)
Mathematics I (Major) 2012–2020 Past Test and Examination Questions Booklet 19

4.2.3 Calculus Solutions June 2014

Multiple choice: Q5) D, Q6) B, Q7) C, Q8) E

MATH1036/7 CALCULUS Page 3 of 5 June 2014

Section B

In this section you are expected to show all your working to earn the marks allocated.
Question 1 [7]
dh i 1
(a) Prove that arcsin x = √ . (5)
dx 1 − x2
✓✏ ✓✏ ✓✏
Solution

✒✑ ✒✑ ✒✑
Let y = arcsin x, 0.5 −1 < x < 1. 0.5 Then sin y = sin(arcsin x) = x 0.5
✓✏
d(sin y)
✒✑
and = 1. 0.5
✓✏ ✓✏
dx
q
dy dy 1
dx cos y ✒✑ ✒✑
Thus cos y =1⇔ = . 0.5 However cos y = ± 1 − sin2 y. 0.5
dx
✓✏ q √ ✓✏
π π
2 ✒✑ ✒✑
2
Since −1 < x < 1, then − < y < . 0.5 Thus cos y = 1 − sin y = 1 − x . 0.5 2
2
· 1❥
dy 1 d h i 1
Hence = √ and it follows that arcsin x = √
dx 1−x 2 dx 1−x 2

dh i
(b) Find arcsin ln x , where e−1 < x < e. (2)
dx
Solution

i ✓✏ ✓✏
d(ln x)
1❥ .
dh 1
0.5 = p dx
✒✑ ✒✑
arcsin ln x 0.5 = p
dx 1 − (ln x)2 x 1 − (ln x)2

Question 2
In this question you do not need to simplify your answers.
Use the method of logarithmic differentiation to find y′ , where: [10]
√ 6
(a) y = 5 xe x (x4 + 10)100 . (5)
Solution

 ✓✏ 1
0.5 = ln x + ln e x + 100 ln(x4 + 10) 1❥
 √5
✒✑ 5
x6 4 100 6
ln y = ln xe (x + 10)
✓✏
1
✒✑
6 4
= ln x + x + 100 ln(x + 10). 0.5
5
It follows that
" # ✓✏ ′ ✓✏
1❥
d(ln y) d 1 y 1 100 × 4x3
✒✑ y ✒✑ 5x
6 4
= ln x + x + 100 ln(x + 10) 0.5 ⇔ 0.5 = + 6x5 + 4 .
dx dx 5 x + 10
20 2012–2020 Past Test and Examination Questions Booklet Mathematics I (Major)

MATH1036/7 CALCULUS Page 4 of 5 June 2014

Hence
! !
1❥
1 5 100 × 4x3 1 5 100 × 4x3 √5 x6 4

y = + 6x + 4 ×y= + 6x + 4 xe (x + 10)100 .
5x x + 10 5x x + 10

sin x cos x
(b) y = . (5)
e−x x x
Solution

! ✓✏ ✓✏
sin x cos x
✒✑ ✒✑
−x x
ln y = ln 0.5 = ln sin x + ln cos x − ln e − ln x 0.5
e−x x x ✓✏
✒✑
= ln sin x + ln cos x + x − x ln x. 0.5

It follows that
y′ ✓✏ d   ✓✏
y ✒✑ dx ✒✑
0.5 = ln sin x + ln cos x + x − x ln x 0.5
′ ✓✏ ✓✏ ✓✏
y cos x sin x
sin x ✒✑ cos x ✒✑ ✒✑
⇔ = 0.5 − 0.5 + 1 − ln x − 1 · 0.5
y
Hence
! !
1❥
′ cos x sin x cos x sin x sin x cos x
y = − − ln x y = − − ln x
sin x cos x sin x cos x e−x x x

Question 3 [11]
(a) State the Mean Value Theorem. (4)
The Mean Value Theorem’s statement
Let f be a function that satisfies the following conditions:
1) f is continuoous on the closed interval [a, b], 1❥
2) f is differntiable on the open interval (a, b). 1❥
Then there is a number c ∈ (a, b) 1❥ such that

1❥
f (b) − f (a)
f ′ (c) = ,
b−a
or equivalently
f (b) − f (a) = f ′ (c)(b − a).

Note Give 1❥ to any of the above two equations.


ex + 1
(b) Does the function f (x) = x satisfy the Mean Value Theorem in the interval
e −1
[−1, 2]? Justify your answer. (2)
Mathematics I (Major) 2012–2020 Past Test and Examination Questions Booklet 21

MATH1036/7 CALCULUS Page 5 of 5 June 2014

Solution
1❥
ex + 1
No f (x) = does not satisfy the Mean Value Theorem in the interval [−1, 2].
ex − 1
ex + 1
Because f (x) = x is not defined at x = 0, so it is not defined on the interval [−1, 3]
1❥
e −1
and therefore it is not continuous on the interval [−1, 2].
1❥ if the student writes that the function is either not defined or not contin-
Note Give ✓✏
✒✑
uous, but 0.5 if the student only says that the function is not differentiable.
(c) Use l’Hôpital’s rule to evaluate lim+ (sin x) x (5)
x→0

✓✏ ✓✏ ✓✏
Solution
ln sin x
✒✑ ✒✑ ✒✑
Let y = (sin x) x . Then ln y = ln(sin x) x 0.5 = x ln sin x 0.5 = · 0.5
1

✓✏
x
ln sin x ∞
✒✑
And lim+ ln y = lim+ = − · 0.5 Hence arrording to l’Hôpital rule,
x→0 x→0 1 ∞
x
✓✏
cos x
ln sin x
= lim+ sin x · 0.5
✒✑
lim+ ln y = lim+
x→0 x→0 1 x→0 −1
x x 2
 
 x    ✓✏
 1 
 sin x  ✒✑
Thus lim+ ln y = lim+ −x cos x = lim+ −x cos x   · 0.5
x→0 x→0 sin x x→0

✓✏ ✓✏
x
sin x
✒✑ x→0 ✒✑
Since lim+ −x cos x = 0 0.5 and lim+ = 1, 0.5
x→0 x
 

 1 
 ✓✏
then lim+ ln y = lim+ −x cos x    = 0· ✒✑
 0.5
x→0 x→0  sin x 
x ✓✏
✒✑
Hence lim+ y = 1 and lim+ (sin x) x = 1. 0.5
x→0 x→0

Total Section B: [28] marks


22 2012–2020 Past Test and Examination Questions Booklet Mathematics I (Major)

4.2.4 Calculus Solutions September 2014

Multiple choice: C;E;D;C;D


Mathematics I (Major) 2012–2020 Past Test and Examination Questions Booklet 23
24 2012–2020 Past Test and Examination Questions Booklet Mathematics I (Major)
Mathematics I (Major) 2012–2020 Past Test and Examination Questions Booklet 25

4.2.5 Calculus Solutions November 2014

Multiple choice: Q1) C, Q2) E, Q3) A, Q4) B, Q5) D, Q6) A, Q7) C, Q8) D

MATH1036 CALCULUS Page 5 of 10 November 2014

Section B

[66]
In this section you are expected to show all your working to earn the marks allocated.
Question 1 [12]
1) Show that the ordinary differential equation (ODE)

(4x + y4 )dx + (4xy3 − e3y )dy = 0 (E)

is exact. (4)
Solution

1❥ and 1❥
∂(4x + y4 ) ∂(4xy3 − e3y )
= 4y3 = 4y3 .
∂y ∂y

= 4y3 . 1❥
∂(4x + y4 ) ∂(4xy3 − e3y )
So =
∂y ∂y
Hence the differential equation (E) is exact. 1❥
2) Give the general solution of the ODE (E). (6)
Solution
Z
Let f (x, y) = (4x + y4 )dx.

Then f (x, y) = 2x2 + xy4 + g(y). 1❥

= 4xy3 + g′ (y) = 4xy3 − e3y . 1❥


∂ f (x, y)
∂y
Z

So g (y) = −e 3y ❥
1 and g(y) =
1
−e3y dy = − e3y . 1❥
3

Hence the general solution of the differential equation (E) is 2x2 + xy4 − e3y = C. 2❥
1
3
!
1
3) Find the particular solution satisfying y − √ = 0. (2)
3
Solution
✓✏
2 1 1 ✓✏
!2
1 1 1
✒✑ 3 3 3 ✒✑
We have 2 − √ − √ × 0 − = C. 0.5 So C = − = . 0.5 It follows
3 3 3 !
that the particular solution satisfying y − √ = 0 is 2x2 + xy4 − e3y = C. 1❥
1 1
3 3
26 2012–2020 Past Test and Examination Questions Booklet Mathematics I (Major)

MATH1036 CALCULUS Page 6 of 10 November 2014

Question 2 [13]
R k
1) Let k be a real number not equal to −1, i. e. k ∈ R \ {−1}. Evaluate cos x sin x dx. (3)
Solution
Z
1❥sink+1 x 1❥ + C. 1❥
1
cos x sink x dx =
k+1
Z 5
cos x
2) Evaluate √3 dx. If you wish to, you may use your answer in Question 2(1). (8)
sin x
Solution

✓✏
✒✑ ✓✏
cos5 x = (cos2 x)2 0.5

✒✑
= (1 − sin2 x)2 cos x 0.5
= (1 − 2 sin2 x + sin4 x) cos x. 1❥

= (1 − 2 sin2 x + sin4 x) cos x × sin− 3 x 1❥


cos5 x 1
√3
sin x
= (sin− 3 x − 2 sin 3 x + sin 3 x) cos x 1❥
1 5 11

Z Z ✓✏
cos5 x
✒✑
1 5 11
√3 dx = cos x(sin− 3 x − 2 sin 3 x + sin 3 x) dx 0.5
✓✏ ✓✏ ✓✏
sin x
1 2 1
✒✑5 ✒✑11 ✒✑
2 8 14
= sin x 0.5 −
3 sin x 0.5 +
3 sin x 0.5 + C
3
1
− +1 +1 +1
✓✏ ✓✏ ✓✏ ✓✏
3 3 3
3 3 3
✒✑4 ✒✑14 ✒✑ ✒✑
2 8 14
= sin 3 x 0.5 − sin 3 x 0.5 + sin 3 x 0.5 + C. 0.5
2
Z π
2 cos5 x
3) Calculate √3 dx. (2)
0 sin x
Solution

Z π " # π2 ✓✏
2 cos5 x 3 3 3
✒✑
2 8 14
√3 dx = sin 3 x − sin 3 x + sin 3 x 0.5
2 4 14
3 ✓✏
0 sin x 0
3 3
2 4 14 ✒✑
= − + 0.5
6−3 3
= +
4 14
3 3
= +
4 14
21 + 6 27 ❥
= = 1
28 28
Mathematics I (Major) 2012–2020 Past Test and Examination Questions Booklet 27

MATH1036 CALCULUS Page 7 of 10 November 2014

Question 3 [14]
4 4x + 1
Let f be the rational function defined by f (x) = 3 + − ·
x + 1 x2 − x + 1
1) Find the following integrals: (Complete the square, if necessary)
Z
4
a) dx. (1)
x+1
Solution
Z
dx = 4 ln |x + 1| + c1 . 1❥
4
x+1
Z
4x − 2
b) dx. (3)
x2 − x + 1

✓✏ ✓✏
Solution
Z Z
4x − 2 2x − 1 ❥ ❥
✒✑ ✒✑
2
dx = 2 dx 1 = 2 0.5 ln |x − x + 1| 1 + c2 . 0.5
x2 − x + 1 x2 − x + 1
Z
3
c) dx. (4)
x −x+1
2

Solution
Z Z Z ✓✏
3 dx ❥ dx
(x − 12 )2 + 34 ✒✑
dx = 3 1 =3 0.5
x2 − x + 1 (x − 12 )2 − 14 + 1
Z ✓✏
dx
(x − 12 )2 + ( 23 )2 ✒✑
=3 √ 0.5
  
= 3  √ arctan  √  + c3 1❥
 1  x − 12 
3 3
2 2
!
+ c3 . 1❥
√ 2x − 1
= 2 3 arctan √
3
Z
3(x3 − 3x + 2)
2) Use your answers in the preceding question to find dx. (6)
x3 + 1
Solution

4 4x + 1
f (x) = 3 + − 2
3(x3 + 1) + 4(x2 − x + 1) − (4x + 1)(x + 1) ✓✏
x+1 x −x+1

✒✑
= 0.5
3x3 + 3 + 4x2 − 4x + 4 − (4x2 + 4x + x + 1) ✓✏
x3 + 1

✒✑
= 0.5
3x3 + 4x2 − 4x + 7 − 4x2 − 5x − 1 ✓✏
x3 + 1

✒✑
= 0.5
3x3 − 9x + 6 3(x3 − 3x + 2) ✓✏
x3 + 1

✒✑
= = 0.5
x3 + 1 x3 + 1
28 2012–2020 Past Test and Examination Questions Booklet Mathematics I (Major)

MATH1036 CALCULUS Page 8 of 10 November 2014


Z Z ! ✓✏
3(3x3 − 3x + 2) 4 4x + 1
✒✑
dx = 3+ − 2 dx 0.5
x3 + 1 x+1 x −x+1
Z
dx 1❥
4x − 2 + 3
= 3x + 4 ln |x + 1| −
x2 − x + 1
But
Z Z Z ✓✏
4x − 2 + 3 2(2x − 1) dx
✒✑
dx = dx + 3 dx 0.5
x2 − x + 1 x2 − x + 1 x2 − x + 1
!
+ K. 1❥
2
√ 2x − 1
= 2 ln |x − x + 1| + 2 3 arctan √
3
Hence
Z !
+ C. 1❥
3(x3 − 3x + 2) 2
√ 2x − 1
dx = 3x + 4 ln |x + 1| − 2 ln |x − x + 1| − 2 3 arctan √
x3 + 1 3

Question 4 [13]

X
1-a) When is a series an said to be absolutely convergent? (2)
n=1
Solution

X
A series is said to be absolutely convergent if |an | converges. (2)
n=1
X∞
sin(2n + 1) π2
b) Is the series convergent? Justify your answer. (3)
n=1
n
Solution

sin(2n + 1) = (−1)n , 1❥ where n = 1, 2, · · · .



X ∞
X
π sin(2n + 1) π 2 (−1)n
So = ·
2 n=1
n n=1
n

1❥ Hence the

X (−1)n
The series · converges according to the alternating series test.
n=1
n

converges. 1❥
X∞
sin(2n + 1) π2
series
n=1
n
X∞
sin(2n + 1) π2
c) Is the series absolutely convergent? Justify your anwser. (3)
n=1
n
Solution
1 ❥
diverges. 1❥ Therefore
X∞ X ∞ X∞ X∞
sin(2n + 1) π2 (−1)n 1
= = · 1 The series
n=1
n n=1
n n=1
n n=1
n

does not converge absolutely. 1❥


X∞
sin(2n + 1) π2
the series
n=1
n
Mathematics I (Major) 2012–2020 Past Test and Examination Questions Booklet 29

MATH1036 CALCULUS Page 9 of 10 November 2014

X∞ !n
3n − 4
2) Prove that the series converges. (5)
n=1
4n − 3
Solution

s !n r
1❥ = lim
n 3n − 4 n 3n − 4 n
3n − 4 ❥
lim = lim 1
n→∞ 4n − 3 n→∞ 4n − 3 n→∞ 4n − 3
3 ❥
1 < 1. 1❥
4
3− n
= lim =
n→∞ 4− 3
n
4
!n
converges (absolutely). 1❥
X∞
3n − 4
Hence the series
n=1
4n − 3
Question 5 [14]

X (x − 5)n
1) For which values of x does the series converge? (6)
n=1
n
Solution

|x − 5| 1❥ =
an+1 (x − 5)n+1 n n 1
= × = |x − 5|.
an n+1 (x − 5) n n+1 1 + 1n

|x − 5| = |x − 5|. 1❥
an+1 1
lim = lim
n→∞ an n→∞ 1 + 1
n

converges if |x − 5| < 1. 1❥

X (x − 5)n
The series
n=1
n
|x − 5| < 1 ⇔ −1 < x − 5 < 1 ⇔ 4 < x < 6.

converges according to the alternating series test. 1❥


X∞
(−1)n (−1)n
If x = 4, an = and
n n=1
n

diverges. 1❥
X1 ∞
(1)n 1
If x = 6, an = = and the series
n n n=1
n

1❥

X (x − 5)n
Hence the series converges if and only if x ∈ [4, 6).
n=1
n
30 2012–2020 Past Test and Examination Questions Booklet Mathematics I (Major)

MATH1036 CALCULUS Page 10 of 10 November 2014

x2
2) Give with justification the power series representation of the function g(x) =
(1 + x)3
about the point 0. (8)
Solution

1❥

X ∞
X
1 1 n
= = (−x) = (−1)n xn .
1 + x 1 − (−x) n=0 0

(−1)n xn is continuous and differentiable in the interval (−1, 1), 1❥



X
The function f (x) =

✓✏
n=0

X
1
✒✑
n n−1
and f ′ (x) = − = (−1) nx . 1.5
(1 + x)2 n=0

X ✓✏
2
✒✑
′′ n n−2
f (x) = = (−1) n(n − 1)x . 1.5
(1 + x)3 n=0

(−1)n n(n − 1)xn−2 , 1♠



1 1X
So =
(1 + x)3 2 n=0

2 ♠ 1
∞ ∞
x2 1X n n−2
X
and = (−1) n(n − 1)x × x 1 = (−1)n n(n − 1)xn .
(1 + x)3 2 n=0 2 n=0

(−1)n n(n − 1)xn . 1♠



1X
Hence g(x) =
2 n=0

1X
Or g(x) = (−1)n n(n − 1)xn .
2 n=2

Total Section B: [66] marks


Mathematics I (Major) 2012–2020 Past Test and Examination Questions Booklet 31

4.3 Calculus Solutions 2015


4.3.1 Calculus Solutions March 2015
32 2012–2020 Past Test and Examination Questions Booklet Mathematics I (Major)
Mathematics I (Major) 2012–2020 Past Test and Examination Questions Booklet 33
34 2012–2020 Past Test and Examination Questions Booklet Mathematics I (Major)
Mathematics I (Major) 2012–2020 Past Test and Examination Questions Booklet 35
36 2012–2020 Past Test and Examination Questions Booklet Mathematics I (Major)

4.3.2 Calculus Solutions April 2015


Mathematics I (Major) 2012–2020 Past Test and Examination Questions Booklet 37
38 2012–2020 Past Test and Examination Questions Booklet Mathematics I (Major)

4.3.3 Calculus Solutions June 2015

Section A: Multiple choice questions

Question 1: C; Question 2: B; Question 3: E; Question 4: E;

Section B
(Note: X is one mark, (X) is 1/2 mark.)

Question 1 [5]
Let
(x2 − 4)9 √ x
y= e (X)
(x − 1)2
Then Å 2
(X) (x − 4)9 √ x X √
ã
ln y = ln 2
e = 9 ln(x2 − 4) + x − 2 ln(x − 1)
(x − 1)
Implicit differentiation gives
y0 9(2x) 1 2
= 2 + √ − XX
y x −4 2 x x−1
Therefore
(x2 − 4)9 √ x 18) 1 2
Å ã
f 0 (x) = y0 = e + √ − X
(x − 1)2 x2 − 4 2 x x − 1
Question 2 [10]
(a)
e x sin x − x 0
Å ã
lim X
x→0 x2 0
x x
l’H e cos x + e sin x − 1 0
Å ã
= lim XX
x→0 2x 0
x
l’H e (− sin x) + e x cos x + e x cos x + e x sin x
= lim X
x→0 2
2
= =1 X
2
(b)
lim (sin x) x is a limit of the form (00 ). (X)
x→0+
x
Put y = lim+ (sin x) . (X)
x→0
Then
ln y = lim x ln(sin x) (0 · ∞) X
x→0+
ln(sin x) ∞
= lim (X)
x→0+ 1 ∞
x
− cos x
l’H
= lim sin x X
x→0 −1
x2
x
= lim x cos x (X)
x→0 sin x
=0 (X)
Mathematics I (Major) 2012–2020 Past Test and Examination Questions Booklet 39

Hence
lim x ln(sin x) = y = eln y = e0 = 1. (X)
x→0+

Question 3 [6]
Let x and y be the length and the width of the rectangle. The following sketch may be used: (X)

x
We know that the perimeter is constant,
p = 2x + 2y (X)
so that
dx dy
+ = 0. (X)
dt dt
Since the side lengths change with constant speed 2cm/sec, and assuming that x increases, we find

dx dy
= 2 and = −2. X
dt dt
In cm2 , the area of the rectangle is
A(x) = xy. (X)
Then
dA dx dy
=y +x = 2y − 2x. X
dt dt dt
Originally we have that x = y are 1 m, and therefore x + y = 200. Hence, when y = 80, then x = 120, so that X

dA
= 2(80) − 2(120) = −80. (X)
dt
Therefore the area of R decreases at a rate of 80 cm2 /sec when the shorter sides are 80 cm long. (X)
Question 4 [15]
(a) We have f (x) = 0 if and only if
0 = x2 − 3x − 4 = (x − 4)(x + 1) X
Hence the x-intercepts are at x = 4 and x = −1 [alternatively: The x-intercepts are (4, 0) and (−1, 0)] X
Since f (0) = 1, the y-intercept is at 1 [alternatively: The y-intercept is (0, 1)] X
(b)

3 4
x2 − 3x − 4 1− − 2
lim = lim x x =1
x→±∞ x2 − 4 x→±∞ 4
1− 2
x
Hence f f has one horizontal asymptote y = 1. X
2
f is undefined when x − 4 = 0, i. e., when x = ±2. Then

f (x) → ∞ as x → −2− or x → 2− X
f (x) → −∞ as x → −2 or x → 2
+ +
X

(c) Since
x2 + 4
f 0 (x) = 3
(x2 − 4)2
40 2012–2020 Past Test and Examination Questions Booklet Mathematics I (Major)

one sees that f 0 (x) > 0 for all x in the domain of f . X


Hence f increases on (−∞, −2), −2, 2), and (2, ∞), X
and f has no global minima and maxima. X
(d)
Since
x2 + 12
f 00 (x) = −6x
(x2 − 4)3
it follows that f 00 is positive on (−∞, −2) and on (0, 2), and f 00 is negative on (−2, 0) and on (2, ∞). [Students may
use a second derivative table.] X
Hence f is convex up on (−∞, −2) and on (0, 2), and f is convex down on (−2, 0) and on (2, ∞). X
Since convexity changes at 0, f has a point of inflection at x = 0 [or (0, 1) is a point of inflection] X
(e) B

4.3.4 Calculus Solutions September 2015

Section A: Multiple choice answers: C;E;D;E

Section B

Question 1 [5]

You are asked to fence off a rectangular field with area 2400m2 and partition the field into two equal parts. What
is the smallest length of fence needed?
Solution

Let x and y be the length and width of the field, respectively.


2400
Area of the field is given by A = xy = 2400m2 =⇒ y = m.X
x
The length of fence required and partition the field is given by the perimeter in meters as

2400
Å ã
P = 3y + 2x = 3 + 2x.X
x

The perimeter is function of x. To minimise the length of fence required we differentiate P(x) and find critical
d
values at P(x) = 0.
dx
d 1
Å ã
P(x) = 7200 − 2 + 2 = 0X ⇔ x2 = 3600.
dx x

Thus x = ±60X but it is a length x > 0 hence x = 60. Since 0 ≤ x ≤ 2400


Mathematics I (Major) 2012–2020 Past Test and Examination Questions Booklet 41

P(0) is undefined
P(60) = 120
P(2400) = 4803

The smallest length of fence occurs at the minimum value of x = 60m and y = 40m giving total length of
P = 240mX.
Question 2 Å ã [4]
x2 dy
If y = 5 cosh , find at the point (2, 0). Simplify your answer.
4 dx
Solution

d x2 x2 2x
Å ã Å ã
5 cosh = 5 sinh .XX
dx 4 4 4
At the point (2, 0),
Å 2ã
dy 2 2.2
= 5 sinh X
dx 4 4
= 5 sinh(1)X

Question 3 [5]
Find the area bounded by y = 2x2 and y = x4 − 2x2 .
Solution
We find the bounds of the enclosed area by equating the two functions,

2x2 =x4 − 2x2 X


0 =x2 (4 − x2 ) ⇔ x = 0 or x = −2 or x = 2

Z 2
Area = 2x2 − (x4 − 2x2 ) dxXX

−2
Z 2
=2 4x2 − x4 dx

0
ï 3 ò2
x x5
=2 4 − X
3 5 0
ï 3
2 25
ò
=2 4 − − (0 − 0)
3 5
27 128
= = X
15 15

Question 4 [6]
Find the volume of the solid generated by revolving the region in the first quadrant bounded above by the curve
y = x2 , below by the x-axis and on the right by the line x = 1, about the line x = −1.
Solution
We find the bounds of the enclosure by equating functions. In the first quadrant,

x2 = 1 ⇔ x = 1 =⇒ y = 1X

Or

y = 1 ⇔ y = 1.X
42 2012–2020 Past Test and Examination Questions Booklet Mathematics I (Major)

Z 1 √ 2 ©
π [1 − (−1)]2 −

Volume = y − (−1) dyXXX
0
Z 1

[4] − y + 2 y + 1 dy
  

0
p ô1
y2 y3
ñ
= π 3y − −4 X
2 3 0

12 13
ñ ô
= π 3(1) − −4 − (0 − 0 − 0)
2 3
7
= π X.
6

Total Section B: [20] marks


Mathematics I (Major) 2012–2020 Past Test and Examination Questions Booklet 43

4.3.5 Calculus Solutions November 2015


44 2012–2020 Past Test and Examination Questions Booklet Mathematics I (Major)
Mathematics I (Major) 2012–2020 Past Test and Examination Questions Booklet 45
46 2012–2020 Past Test and Examination Questions Booklet Mathematics I (Major)
Mathematics I (Major) 2012–2020 Past Test and Examination Questions Booklet 47
48 2012–2020 Past Test and Examination Questions Booklet Mathematics I (Major)
Mathematics I (Major) 2012–2020 Past Test and Examination Questions Booklet 49
50 2012–2020 Past Test and Examination Questions Booklet Mathematics I (Major)
Mathematics I (Major) 2012–2020 Past Test and Examination Questions Booklet 51
52 2012–2020 Past Test and Examination Questions Booklet Mathematics I (Major)

4.4 Calculus Solutions 2016


4.4.1 Calculus Solutions March 2016

Question 9 - 9 marks
(a) Complete the sentence in words or mathematical symbols:

For the limit lim f (x) to exist... (3 marks) Both the left-hand limit lim− f (x) and the right-hand limit
x→a x→a

lim f (x) of the function exist and are equal to each other.
x→a+

or
lim f (x) = L ⇔ lim− f (x) = L = lim+ f (x).
x→a x→a x→a

(b) Fill in the answers to the limits in the table:

Expressions Answers
1
1: lim(1 + x) x e
x→0
2: lim + cosec θ −∞ or D.N.E.
θ→−π
3: lim 29−x 0
x→∞
4: lim− byc 1
y→2

(4 marks)

(c) Write down a limit which shows that a function h(x) has a vertical asymptote at x = 1.5. (2 marks)

lim h(x) = ∞
x→1.5−

or
lim h(x) = −∞
x→1.5+
or
lim h(x) = ∞
x→1.5

etc. (They can write D.N.E. too, but it is not sufficient on its own, they must show that at least one of the
limits is infinite.)
Mathematics I (Major) 2012–2020 Past Test and Examination Questions Booklet 53

Question 10 - 8 marks
Evaluate (and SHOW ALL WORKING)

x2 − 4
(a) lim √ . (4 marks)
x→−2 x+3−1


x2 − 4 (x2 − 4)( x + 3 + 1)
lim √ = lim √ √
x→−2 x + 3 − 1 x→−2 ( x + 3 − 1)( x + 3 + 1)

(x2 − 4)( x + 3 + 1)
= lim
x→−2 x+3−1
2

(x − 4)( x + 3 + 1)
= lim
x→−2 x+2

(x + 2)(x − 2)( x + 3 + 1)
= lim
x→−2 x+2

= lim (x − 2)( x + 3 + 1)
x→−2

= (−2 − 2)( −2 + 3 + 1)
= (−4)(2)
= −8

x2 − 4
(b) lim . (4 marks)
x→∞ 3x2 (x + 1) + 10(x + 1)

x2 − 4
lim
x→∞ 3x2 (x + 1) + 10(x + 1)
(x2 − 4)/x3
= lim
x→∞ (3x + 3x2 + 10x + 10)/x3
3

(divide top and bottom by x to the highest exponent in the denominator)


1/x − 4/x3
= lim
x→∞ 3 + 3/x + 10/x2 + 10/x3
0
= since all terms except 3 go to zero in the limit.
3
= 0.
54 2012–2020 Past Test and Examination Questions Booklet Mathematics I (Major)

Question 11 - 10 marks
1 − cos θ sin θ
(a) Prove that lim = 0. You may assume that lim = 1. (5 marks)
θ→0 θ θ→0 θ

1 − cos θ 1 − cos θ 1 + cos θ


Å ãÅ ã
lim = lim
θ→0 θ θ→0 θ 1 + cos θ
2
1 − cos θ
= lim
θ→0 θ(1 + cos θ)

sin2 θ
= lim
θ→0 θ(1 + cos θ)
sin θ sin θ
Å ãÅ ã
= lim
θ→0 θ 1 + cos θ
sin θ sin θ
Å ãÅ ã
= lim lim
θ→0 θ θ→0 1 + cos θ
ã lim sin θ
sin θ
Å
θ→0
= lim
θ→0 θ lim(1 + cos θ)
θ→0
lim sin θ
sin θ
Å ã
θ→0
= lim
θ→0 θ 1 + lim cos θ
θ→0
0
= (1)
1+1
= 0.

(b) Use the Sandwich (Squeeze) Theorem to determine lim sinx2 x . Clearly define your functions f (x), g(x) and
x→−∞
h(x) and explain your working. (5 marks)

Since
−1 ≤ sin x ≤ 1
we have (recall x2 is always positive or zero, and we do not need to consider what happens at zero)
1 sin x 1
− ≤ 2 ≤ 2
x2 x x
Let
1 sin x 1
f (x) := − ; g(x) := ; h(x) := .
x2 x2 x2
Now,
lim f (x) = lim h(x) = 0
x→−∞ x→−∞

and thus by the Sandwich Theorem


sin x
lim g(x) = lim = 0.
x→−∞ x→−∞ x2
Mathematics I (Major) 2012–2020 Past Test and Examination Questions Booklet 55

Question 12 - 13 marks
(a) A function f is continuous at the point a if and only if ... (2 marks)

f (a) = lim f (x).


x→a

(b) Indicate with ‘Y’ or ‘N’ whether the functions in the following table are continuous at x = −1:

Function Y/N
1
1: f (x) = x−1 Y
2: g(x) = |x + 1| Y
3: h(x) = sec(x + 1) Y
4: j(x) = b2xc N

(4 marks)

(c) Find the value(s) of a which make(s) the function g(x) continuous if

|x − 2| + 1 if x ≤ 1
(
g(x) =
2
−x + ax − 3 if x > 1.

Show all your working. (5 marks) The value of the function at 1 is

g(1) = |1 − 2| + 1 = 2.

For the left limit we have

lim g(x) = lim− |x − 2| + 1 = 2


x→1− x→1

For the right limit we have

lim g(x) = lim+ −x2 + ax − 3 = −1 + a − 3 = a − 4


x→1+ x→1

For continuity, we require that all of these three quantities are the same, i.e. a − 4 = 2, so a = 6.

(d) Draw a function with a removable discontinuity at x = 2 on the Cartesian plane below. (2 marks)
Accept anything which is suitable. I.e., lim f (x) = L but f (2) , L or is not defined.
x→2

Please ask if you are unsure.

(Calculus Total 40)


56 2012–2020 Past Test and Examination Questions Booklet Mathematics I (Major)

4.4.2 Calculus Solutions April 2016


Mathematics I (Major) 2012–2020 Past Test and Examination Questions Booklet 57
58 2012–2020 Past Test and Examination Questions Booklet Mathematics I (Major)
Mathematics I (Major) 2012–2020 Past Test and Examination Questions Booklet 59
60 2012–2020 Past Test and Examination Questions Booklet Mathematics I (Major)
Mathematics I (Major) 2012–2020 Past Test and Examination Questions Booklet 61
62 2012–2020 Past Test and Examination Questions Booklet Mathematics I (Major)

4.4.3 Calculus Solutions June 2016

Question 1 [11]
Z π
(a) esin x cos x dx = (3)
0

A. −e
Partial Credit:
B. −1 Let u = sin x. Then du = cos x dx, X

C.m 0
and xZ= 0 ⇒ u = 0, x = πZ⇒ u = 0. (X).
π 0
∴ esin x cos x dx = eu du (X) = 0.
D. 1 0 0

E. e

(b) The function f (x) = x3 + x5 − 2 is invertible. (3)


Which of the following is true?

A. f −1 is not differentiable at 0

B. ( f −1 )0 (0) = 0 Partial Credit:


By inspection, f (1) = 0, so that f −1 (0) = 1. X
C. ( f −1 )0 (0) = 8 Then f 0 (x) = 3x2 + 5x4 (X), so that f 0 (1) = 8 (X).
1 1 1
D. −1 0
( f ) (0) = −8 ∴ ( f −1 )0 (0) = 0 −1 = 0 = . (X)
f ( f (0)) f (1) 8
E.m ( f −1 )0 (0) =
1
8
Z
2
(c) Evaluate x3 e x dx (5)

Answer:
First use integration by substitution.
Let w = x2 . Then dw = 2x dx. (X)
It follows that

Z Z
2 1 2
x3 e x dx = x2 e x 2x dx (X)
2
Z
1
= wew dw X
2
Next use integration by parts.
Put u = w and dv = ew dw. (X).
Then du = dw and v = ew . (X).
Then
Z Z
w
we dw = u dv
Z
= uv − v du
Z
= we − ew dw X
w

= wew − ew + C1
= (w − 1)ew + C1 (X)

Resubstituting w gives
Z Z
3 x2 1
x e dx = wew dw
2
1
= (w − 1)ew + C
2
1 2 2
= (x − 1)e x + C (X)
2
Mathematics I (Major) 2012–2020 Past Test and Examination Questions Booklet 63

Question 2 [10]
Find the limits
e x − sin x − 1
(a) lim (5)
x→0 x2 cos x
Answer:

e x − sin x − 1 “ 0”
lim X
x→0 x2 cos x 0

l’H e x − cos x “ 0”
= lim X X
x→0 2x cos x − x2 sin x 0
l’H e x + sin x
= lim X
x→0 2 cos x − 4x sin x − x2 cos x

1
= X
2

(b) limπ (sin x)sec x (5)


x→ 2

Answer:
Let

y = limπ (sin x)sec x (X)


x→ 2

Then

ln y = limπ ln(sin x)sec x (X)


x→ 2

= limπ sec x ln(sin x) “0 · ∞” X


x→ 2

ln(sin x) “ 0”
= limπ X
x→ 2 cos x 0
cos x
l’H sin x
= limπ X
x→ 2 − sin x
=0 (X)

∴ y = e0 = 1 (X)

Question 3 [10]
Thandi rides on her bicycle at a constant speed of 4 meters per second (14.4 km/h) towards a water tower, which
has a diameter of 20 meters. How fast does her horizontal angle of sight of the water tower increase when Thandi
is 30 meters away from the water tower?
(a) In the sketch of the aerial view below, label the quantities which you will use for your solution. (2)
Answer: These quantities are inserted in red, where x denotes Thandi’s distance from the water tower.
64 2012–2020 Past Test and Examination Questions Booklet Mathematics I (Major)

10

10 θ
x

(b) If 2θ is Thandi’s horizontal angle of sight of the water tower, find a relation between θ and Thandi’s distance
from the water tower. (2)
Answer: Due to the right angle,
10
sin θ = .
x + 10
(c) Solve the problem. (6)
Answer: Differentiating the equation in (b) with respect to time t one gets
dθ 10 dx
cos θ =− 2
X
dt (x + 10) dθ
When x = 30, then
10 1
sin θ = = X
40 4
so that √
1 15
p …
2
cos θ = 1 − sin θ = 1− = X(X)
16 4
Therefore
dθ 10 4 1
= − 2 √ (−4) = √ X
dt 40 15 10 15
1
Thandi’s horizontal angle of sight of the water tower increases by √ radians per second when Thandi is 30
10 15
meters away from the water tower. X(X)
Question 4 [8]
(a) What is the following theorem called? (2)
Theorem. If f has a local maximum or minimum at an interior point c of dom( f ) and if f is differentiable at c,
then f 0 (c) = 0.

A.n Fermat’s Theorem,


B. Intermediate Value Theorem,
C. Mean Value Theorem,
D. Rolle’s Theorem,
E. Taylor’s Theorem.

(b) Prove the theorem in case of a local maximum. (6)

Answer:
(Let f have a local maximum at c.)
Let a, b be such that the interval (a, b) belongs to the domain of f , a < c < b and
f (x) ≤ f (c) for x ∈ (a, b), i. e.,
f (x) − f (c) ≤ 0. XX
Mathematics I (Major) 2012–2020 Past Test and Examination Questions Booklet 65

Hence
f (x) − f (c)
≥0 if x ∈ (a, c) X
x−c
and
f (x) − f (c)
≤0 if x ∈ (c, b). X
x−c
Since f is differentiable at c, it follows that
f−0 (c) ≥ 0 and f+0 (c) ≤ 0 X
0
⇒ f (c) = f−0 (c) = f+0 (c) = 0. X

Question 5 [6]
2
Find the global maximum and the global minimum of 2x − 3x on [−1, 8]. 3

Answer:
2
Let f (x) = 2x − 3x 3 . Then
2 −1
Å ã
1
0
f (x) = 2 − 3 x 3 = 2 − 2x− 3 X
3
So
1
f 0 (x) = 0 ⇔ x− 3 = 1 ⇔ x = 1 X
and f is not differentiable at 0. (X)
Hence the critical numbers are 0 and 1. (X)
Evaluate f at the critical numbers:
f (0) = 0, (X)
f (1) = −1, (X)
f (−1) = −5, (X)
f (8) = 16 − 3 · 4 = 4, (X)
Comparing the values, if follows that
2
the global maximum of 2x − 3x 3 on [−1, 8] is 4 (X)
2
the global minimum of 2x − 3x 3 on [−1, 8] is −5 (X)

4.4.4 Calculus Solutions August 2016

Full workings must be shown.


Leave answers in exact form.

Question 1 [5]
1
What is the lower sum approximation of the area bounded by g(x) = between x = 1 and x = 5 using 4 partitions?
x
For the ith subinterval denoted [xi−1 , xi ] of [1, 5], minimum values of the function g(x) occur at xi .

5−1
∆x = xi − xi−1 = =1 X
4
xi = 1 + i∆x = 1 + i X
4
X
L(4) = g(xi )∆x X
i=1
1 1 1 1
= .1 + .1 + .1 + .1 X
2 3 4 5
77
= X
60
66 2012–2020 Past Test and Examination Questions Booklet Mathematics I (Major)

Question 2
What are the dimensions of the rectangle with the largest area that be inscribed in the ellipse below? [6]
y axis

(x,y)

−5 5 x axis

−3

Please follow student’s solution if workings are in y and A(y).


The ellipse is given by

x2 y2
+ =1
52 32
3√
y=± 25 − x2 X
5

Area = 4xy (X)


12 √
A(x) = x 25 − x2 X
5 Å
12 √ −2x
ã
A0 (x) = 25 − x2 + x √ X
5 2 25 − x2
2 2ã
12 (25 − x ) − x
Å
= √
5 25 − x2
12 25 − 2x2
= √ = 0X X
5 25 − x2

5
Implying x = ± √ . The bounds of x are 0 ≤ x ≤ 5 (X)with A(0) = 0 = A(5) (X) .
2
5 3
Thus maximum occurs at x = √ and y = √ (X).
2 2
Question 3

1 1+x
Å ã
−1
Prove the following formula tanh x = ln . [5]
2 1−x
Mathematics I (Major) 2012–2020 Past Test and Examination Questions Booklet 67

Let
1 1+x
Å ã
y= ln
2 1−x
1+x
Å ã
= ln (X)
1−x

then
Ç ãå
1+x
Å
tanh y = tanh ln X
1−x
 » 
1+x
sinh ln 1−x
=  »  X
1+x
cosh ln 1−x
√ 1+x √ 1−x
eln ( 1−x ) − eln ( 1+x )
= √ √ 1−x X
ln ( 1+x
1−x ) + eln ( 1+x )

1+x
 » 1−x 
1−x − 1+x
= »  » 1−x  (X)
1+x
1−x + 1+x
√ √
1 + x − (1 − x) 1−x 1+x
= √ √ ·
1 − x 1 + x 1 + x + (1 − x)
2x
=
2
=x (X).

If

x = tanh y then
−1
tanh x = y (X)

OR
Question 4 [5]

Prove that is f is continuous on a closed interval [a, b], then there exist a c in the closed interval such that

Zb
f (x) dx = f (c)(b − a).
a

Suppose f is continuous on [a, b]. By Extreme Value Theorem f attains its minimum and maximum on [a, b], say
at y and z both in [a, b]X. Thus,

f (y) ≤ f (x) ≤ f (z)X


Zb
f (y)(b − a) ≤ f (x) dx ≤ f (z)(b − a)X.
a

Since (b − a) is a constant g(x) = f (x)(b − a)X is a continuous function and


68 2012–2020 Past Test and Examination Questions Booklet Mathematics I (Major)

Zb
g(y)(≤ f (x) dx ≤ g(z).
a
Zb
If we let N = f (x) dx, by the Intermediate Value Theorem there must be a c ∈ (y, z) such that g(c) = NX i.e
a

g(c) = N
Zb
f (c)(b − a) = f (x) dxX.
a

Question 5 [3]
Using areas of known geometries and other properties of integrals evaluate the integral
Z2
(2 − |x|) dx.
−2

Give half a mark for correct diagram if answer is wrong.

Z2 Z2
(2 − |x|) dx = 2 (2 − |x|) dx X
−2 0
1
=2· ·2·2 X
2
=4 X

Question 6 [10]

a. What is the total area of the region bounded by x = 4y2 − 2y and x = 12y2 ? (5)
2 2
4y − 2y = 12y
0 = 8y2 + 2y
0 = 2y(4y + 1)
1
y = 0 (X) or y = − (X)
4

Z0
Area = [4y2 − 2y − 12y2 ] dy XX
− 41

Z0
= [−8y2 − 2y] dy
− 14
0
8
ï
= − y3 − y2 X
3 − 14
8 1 1
Å ã
=− − −
3 (−4)3 (−4)2
1
= X
48
Mathematics I (Major) 2012–2020 Past Test and Examination Questions Booklet 69


b. What is the volume of a solid generated by rotating h(x) = x + 4 about the line y = 2 on [1, 4]? (5)

Z4

Volume = π [ x + 4 − 2]2 dx X
1
Z4

=π x + 4 x + 4 dx X
1
4
x2 8 3
ï
=π + x 2 + 4x X
2 3 1
64 1 8
= π[8 + + 16 − − − 4] X
3 2 3
229
=π X
6

Total :[30] marks


Bonus : [4] marks
70 2012–2020 Past Test and Examination Questions Booklet Mathematics I (Major)

4.4.5 Calculus Solutions November 2016


Mathematics I (Major) 2012–2020 Past Test and Examination Questions Booklet 71
72 2012–2020 Past Test and Examination Questions Booklet Mathematics I (Major)
Mathematics I (Major) 2012–2020 Past Test and Examination Questions Booklet 73
74 2012–2020 Past Test and Examination Questions Booklet Mathematics I (Major)
Mathematics I (Major) 2012–2020 Past Test and Examination Questions Booklet 75
76 2012–2020 Past Test and Examination Questions Booklet Mathematics I (Major)
Mathematics I (Major) 2012–2020 Past Test and Examination Questions Booklet 77
78 2012–2020 Past Test and Examination Questions Booklet Mathematics I (Major)
Mathematics I (Major) 2012–2020 Past Test and Examination Questions Booklet 79
80 2012–2020 Past Test and Examination Questions Booklet Mathematics I (Major)
Mathematics I (Major) 2012–2020 Past Test and Examination Questions Booklet 81
82 2012–2020 Past Test and Examination Questions Booklet Mathematics I (Major)
Mathematics I (Major) 2012–2020 Past Test and Examination Questions Booklet 83

4.5 Calculus Solutions 2017


4.5.1 Calculus Solutions March 2017

Question 1 [8]
A step function of x which is the least integer greater than or equal to x also called the ceiling function of x is
usually written dxe. The graph of f (x) below is a graphical representation of the ceiling function. Use the graph
below to answer questions that follow.

f (x) = dxe

−3 −2 −1 1 2 3
−1

−2

a. a. What is lim − f (x)? −2X . (1)


x→−2

b. b. What is lim + f (x)? −1X . (1)


x→−2

c. c. What is lim f (x)? does not existX . (1)


x→−2

d. d. For any n ∈ Z is f (x) continuous at x = n? Respond by writing Yes or No


then give a reason to support your answer.

NoX

lim f (x) , lim+ f (x)X (2)


x→n− x→n

E. Circle the letter above the graph that represents of the function g(x) = x − dxe.

(3)

AXXX B

−2 −1 1 2
−1 −1
84 2012–2020 Past Test and Examination Questions Booklet Mathematics I (Major)

C D

2 2

−2 −1 1 2 −2 −1 1 2
−1 −1

Question 2 [6]

1
a. lim(1 + x) x = eX . (1)
x→0
1
2e x
b. Compute lim+ 1
, using only methods covered in class so far. (5)
x→0 1 + ex

1
ex 2
= lim+ 1
X
− 1x
x→0 e (e + 1)
x

lim+ 2
x→0
= 1
X
( lim+ e− x + lim+ 1)
x→0 x→0
2 1
= X ∵ lim+ = ∞X
lim e − 1x
+1 x→0 x
x→0+
=2X.

Question 3 [6]

2
What are the asymptotes of h(x) = π − 2 ?
x
Show workings on how you got to your answer using limits.

2
lim π − = πXX
x→∞ x2
OR
2
lim π − = πXX
x→−∞ x2
∴ y = π is a horizontal asymptote.X
2
lim π − 2 = −∞XX
x→0+ x
OR
2
lim− π − 2 = −∞XX
x→0 x
∴ x = 0 is a vertical asymptote.X
Mathematics I (Major) 2012–2020 Past Test and Examination Questions Booklet 85

Question 4 [6]
a. If 2 − x2 ≤ g(x) ≤ 2 cos(x) for all x, what is lim g(x)? (4)
x→0

lim 2 − x2 = 2X
x→0
lim 2 cos(x) = 2X
x→0
∴ By Sandwich or Squeeze or Pinch TheoremX
lim g(x) = 2X
x→0

b. Give the full statement of the theorem that you used to answer part a. of this question. (2)
If f (x) ≤ g(x) ≤ h(x) when x is near a and lim f (x) = L = lim h(x)X, then lim g(x) = L.X
x→a x→a x→a
Question √
5 √ [5]
x2 + 4 − 5
Find lim √ , if it exists.
x→1 x+3−2

√ √ √ √
x2 + 4 − 5 x2 + 4 + 5
= lim √ · √ √ X
x→1 x+3−2 x2 + 4 + 5
x2 + 4 − 5
= lim Ä √ äÄ√ √ äX
x→1 x+3−2 x2 + 4 + 5

x2 − 1 x+3+2
= lim Ä √ äÄ√ √ ä· √ X
x→1 x+3−2 x2 + 4 + 5 x+3+2

(x2 − 1)( x + 3 + 2)
= lim Ä√ √ äX
x→1 (x + 3 − 4) x2 + 4 + 5

(x + 1)( x + 3 + 2)
= lim Ä √ √ ä
x→1 x2 + 4 + 5
4
=√ X
5

Question 6 √ [9]
a. Detemine if the function g(x) = |x2 − 3| is continuous at x = 3.
If it is discontinuous, state the type of the discontinuity. (3)

lim
√ |x2 − 3| = lim 2
√ −(x − 3) = 0, 1/2
x→ 3− x→ 3−
2 2
lim
√ |x − 3| = lim
√ (x − 3) = 0, 1/2
x→ 3+ x→ 3−
√ √
g( 3) = |( 3)2 − 3| = 01/2

lim
√ g(x) = g( 3)X
x→ 3

∴ g(x) is continuous at x = 3. 1/2

√ √
b. Compute g0− ( 3) and g0+ ( 3) from first principles. (5)
86 2012–2020 Past Test and Examination Questions Booklet Mathematics I (Major)


√ g(x) − g( 3)
g0− ( 3) = lim
√ √ X
x→ 3− x− 3
−(x2 − 3)
= lim
√ √ 1/2
x→ 3− x − 3
√ √
−(x − 3)(x + 3)
= lim
√ √ 1/2
x→ 3− x− 3
√ √
= lim
√ −(x + 3) = −2 3 1/2
x→ 3−


√ g(x) − g( 3)
g0+ ( 3) = lim
√ √ X
x→ 3+ x− 3
(x2 − 3)
= lim
√ √ 1/2
x→ 3− x − 3
√ √
(x − 3)(x + 3)
= lim
√ √ 1/2
x→ 3+ x− 3
√ √
√ (x +
= lim 3) = 2 3 1/2
x→ 3+


c. Is g0 (x) continuous at x = 3?
If it is discontinuous, state the type of the discontinuity. (1)
NO 1/2. Jump discontunuity. 1/2

4.5.2 Calculus Solutions April 2017

Section A [8 marks]

Instructions:
For each multiple choice question, select the letter that corresponds to the correct answer.

[A1] The normal to the curve y = 3 tan x + x2 + 1 at the point with coordinates (0, 1) is

(A) y = −3x + 1.
(B) y = 3x − 1.
1
(C) y = x − 1.
3
1
(D) y = − x + 1.X
3
1
(E) y = x + 1. 2 marks
3
dy
[A2] If y = log10 (sin x) ; 0 < x < π, then =
dx
1
(A) ln 10 .
sin x
1
(B) .
ln 10 sin x
1
(C) cot x.X
ln 10
Mathematics I (Major) 2012–2020 Past Test and Examination Questions Booklet 87

1
(D) .
ln 10 cot x
1
(E) . 2 marks
ln 10 cos x
[A3] Let f and g be differentiable functions. Which of the following is equal to
d
( f (x + g(x)))?
dx
(A) f 0 (x) + f 0 (g(x))g0 (x).
(B) f 0 (x + g(x))g0 (x).
(C) f 0 (x + g(x))(x + g0 (x)).
(D) f 0 (x) + g0 (x)(1 + g0 (x)).
(E) f 0 (x + g(x))(1 + g0 (x)).X 2 marks
Z Å
1
ã
[A4] Which of the following is equal to + eπ dx?
7x
2
(A) − + C.
7x2
1
(B) ln |7x| + eπ+1 x + C.
π+1
ln |x|
(C) + π eπ x + C.
7
ln |x|
(D) + eπ x + C.X
7
x
(E) ln + eπ x + C.
7
2 marks

Section B [32 marks]

Instructions:
In this section you are expected to show all your working to earn the marks allocated.
Simplify your answers.

Question 1 - 13 marks
(a) Let y = xr for r ∈ R.
dy
Prove that = rxr−1 . 4 marks
dx
Solution
r
y = xr = eln x = erln x .
dy r
Then = erln x = xr r x−1 = r xr−1 .
dx x
Ç √ √ å
d 5 x+x−4 x
(b) Find . 3 marks
dx x
Solution
 √x √  √
d 5 +x−4 x d 1
Ä ä
x
dx x = dx 5 x−1 + 1 − 4 x− 2
√ √ 3
=5 x
ln 5 2 √1 x x−1 + 5 x
(−1)x−2 + 2 x− 2
√ √
x 1 x 1 2
=5 ln 5 3 −5 x2
+ 3
2x 2 x2
88 2012–2020 Past Test and Examination Questions Booklet Mathematics I (Major)

Question 2 - 13 marks
2
(a) Consider the equation x4 + cos y + e x y = x2 + y2 .
dy
Find . Show all your working. 6 marks
dx
Solution
2
x4 + cos y + e x y = x2 + y2
2
⇔ 4x3 − siny y0 + e x y (2xy + x2 y) = 2x + 2y y0
2 2
⇔ 4x3 − siny y0 + 2xye x y + x2 e x y y0 = 2x + 2yy0
2 2
⇔ 4x3 + 2xye x y − 2x = (siny − x2 e x y + 2y)y0
2
4x3 +2xye x y −2x
⇔ y0 =
sin y−x2 e x2 y +2y

(b) Find the differential dy if y = πt tπ . 2 marks


Solution
dy = (πt lnπ tπ + πt πtπ−1 )dt.

Question 3 - 13 marks

(a) Find the equation of the tangent line to the curve of


x
f (x) = at x = 1. 4 marks
x+1
Solution
x+1−x 1
f 0 (x) = (x+1) 2 = (x+1) 2

1 1
f (1) = 2 , f 0 (1) = 4
1
y = f (1) + f 0 (1)(x − 1) = 2 + 41 (x − 1) = 1
4 + 14 x
(b) For what value(s) of a does the tangent line to the curve pass through
the point (−5, a ) ? 3 marks
Solution
a = 14 + 41 (−5) = −1
(c) Use linearization to approximate the value of f (1.2). 2 marks
Solution
f (1.2) ≈ 14 + 41 (1.2) = 14 + 41 12
 22 11
10 = 40 = 20

Question 4 - 13 marks
Z 2
x−1
Evaluate √ dx. 8 marks
−1 x+2
Solution

u = x + 2, u2 = x + 2, x = u2 − 2, 2u du = dx,
x = −1 u = 1, x=2 u = 2,
Z 2 Z 2 2 Z 2 ï 3 ò2
x−1 u −2−1 u 8 1
ï ò
2
√ dx = 2u du = 2 (u − 3) du = 2 − 3u = 2 − 6 − + 3
−1 x+2 1 u 1 3 1 3 3
=-4 3
Mathematics I (Major) 2012–2020 Past Test and Examination Questions Booklet 89

4.5.3 Calculus Solutions June 2017

Question 1 - 4 marks
TRUE or FALSE:

Statement True/False
R R
s0 (t)r(t)dt = r(t)s(t) − s(t)r0 (t)dt T

If f is continuous on an interval I, then f −1 exists and is also continuous on I. F

The function g has an absolute minimum at c if g(c) ≥ g(x) ∀x in the domain of g. F

r0 (s) = 0 ⇒ the point (s, r(s)) is a local extremum of the function r(s). F

Question 2 - 4 marks
(a)
R Write in the block how many times you need to use integration by parts in order to integrate
eu cos u du? DO NOT do the integration! (1 mark)

2 (or any even number)


(b) Evaluate, showing all working: (3 marks)
Z 1
1

3
dt.
0 e − t2
Solution

Z 1
1 t 1
… Å ã
3 3
dt = arcsin √
0 e − t2 e 0
1
Ç å
0
Å ã
3
= arcsin √ − arcsin √
e e
1
ã Å
= arcsin √ .
3 e

Question 3 - 7 marks
d x
(a) Show all working to find dx x . 4 marks

Solution
Let y := x x and take logs of both sides:
ln y = ln x x = x ln x.
Then differentiate implicitly with respect to x
y0 1
= ln x + x = ln x + 1.
y x
90 2012–2020 Past Test and Examination Questions Booklet Mathematics I (Major)

Thus, since y = x x we get


d x
x = y0 = x x (ln x + 1).
dx

2t3 −t2 −t+1


(b) Show all working to find the slant asymptote of h(t) = t2 −1
. (3 marks)

Solution
Since
2t3 − t2 − t + 1 (t2 − 1)(2t − 1) − t t
h(t) = = = (2t − 1) − 2 .
t2 − 1 t2 − 1 t −1
The slant asymptote of h(t) is y = 2t − 1.

Question 4 - 4 marks
(a) Find the 3rd degree Taylor polynomial of sin x at x = 0. SHOW ALL YOUR WORKING. 3 marks

Solution
Let f (x) = sin x.
Then f 0 (x) = cos x.
And f 00 (x) = − sin x.
And f 000 (x) = − cos x.
Hence, we have

f (0) = 0
f 0 (0) = 1
f 00 (x) = 0
f 000 (0) = −1,

so that

x3
sin x ≈ P3 (x) = x − .
3!

(b) Differentiate your answer above to write down the 3rd degree Taylor polynomial of cos x at x = 0. (1
mark)

x2
cos x ≈ P03 (x) = 1 − .
2!

Question 5 - 9 marks
(a) Prove the following Theorem: 5 marks
If f 0 (x) = g0 (x) for all x in an interval (a, b), then f − g is a constant on (a, b), that is, f (x) = g(x) + C,
where C is a constant.
Proof
Mathematics I (Major) 2012–2020 Past Test and Examination Questions Booklet 91

Let F(x) = f (x) − g(x).


Then F 0 (x) = f 0 (x) − g0 (x) = 0 for all x ∈ (a, b).
By the above theorem, F is constant on (a, b), i. e., f − g is constant on (a, b).
Thus there is C ∈ R such that ( f − g)(x) = C for all x ∈ (a, b),
i. e., f (x) − g(x) = C or f (x) = g(x) + C for all x ∈ (a, b).

(b) State l’Hôpital’s Rule in full. Hint: Start with “Let f and g be differentiable functions...”. 4 marks

Statement:
Let f and g be differentiable functions where g0 , 0 near x = a (except possibly at a).
If we have that
lim f (x) = 0 and lim g(x) = 0
x→a x→a

or that
lim f (x) = ±∞ and lim g(x) = ±∞.
x→a x→a

0 ∞
(In other words, we have an indeterminate form of the type 0 or ∞ ). Then

f (x) f 0 (x)
lim = lim 0
x→a g(x) x→a g (x)

if the limit on the right hand side exists (or is ∞ or −∞).

Question 6 - 6 marks
Sketch a graph on the axes provided which satisfies the following criteria:

(a) f (x) is continuous for all positive real numbers.


(b) f (x) < 0 for x ∈ (0, 2) and f (x) > 0 for x ∈ (2, ∞)
(c) f 0 (x) > 0 for x < 5 and f 0 (x) < 0 for x > 5
(d) f 0 (x) = 0 when x = 5
(e) f 00 (x) < 0 for 0 < x < 7 and f 00 (x) > 0 for x > 7
(f) lim+ = −∞
x→0
92 2012–2020 Past Test and Examination Questions Booklet Mathematics I (Major)

y
6

f (x)

-x
-
−4−3−2−1 1 2 3 4 5 6 7 8 9 101112

Question 7 - 6 marks
You have 40cm of wire to form a square and a circle. How much of the wire should be used for the square and
how much for the circle to enclose the maximum total area?
Solution: Let s be the length of the side of the square. Let r be the radius of the circle.
Then the perimeter of the two shapes is 40 = 4s + 2πr. This means that r = 20−2s π .
The area of the two shapes together (the optimisation function) is
ã2
20 − 2s
Å
A(s) = s2 + πr2 = s2 + π
π
1 2
= (s π + (20 − 2s)2 )
π
1
= (s2 (4 + π) − 80s + 400)
π
and differentiating gives
1
A0 (s) = (2s(4 + π) − 80).
π
For critical points, we need to find where A0 (s) does not exist (nowhere) and where A0 (s) = 0, which occurs when
1 40
(2s(4 + π) − 80) = 0 ⇒ s =
π π+4

Other possible maxima occur at the end points (i.e., where s = 0 or s = 10)
Comparing the area for these three values gives:
400
A(0) =
π

ñÅ ã2 ô
40 1 40 40
Å ã ã Å
A = (4 + π) − 80
+ 400
π+4 π π+4 π+4
ï 2
1 40 40 · 80 1 400(π + 4) − 1600
ò ï ò
= − + 400 =
π π+4 π+4 π π+4
400
= .
π+4
1
A(10) = (400 + π(100) − 800 + 400) = 100.
π
Mathematics I (Major) 2012–2020 Past Test and Examination Questions Booklet 93

400
Thus (since π+4 < 100 < 400π ) the maximal area is obtained when all of the wire is used for the circle (and the
length of the side of the square is 0).
40−2πr
Alternative solution: The perimeter of the two shapes is 40 = 4s + 2πr. This means that s = 4 .
The area of the two shapes (the optimisation function) is

ã2
40 − 2πr
Å
A(r) = s2 + πr2 = + πr2
4

and differentiating gives

40 − 2πr π 20 − πr
Å ã Å ã
0
A (r) = 2 (− ) + 2πr = −π + 2πr
4 2 2

For critical points, we need to find where A0 (r) does not exist (nowhere) and where A0 (r) = 0, which occurs when

π 20
− (20 − πr) + 2πr = 0 ⇒ r =
2 π+4

40
Other possible maxima occur at the end points (i.e. where r = 0 or r = 2π )
Comparing the area for these three values gives:

A(0) = 100cm2

20
å2 ã2
40 − 2π π+4
Ç
20 20 400
Å ã Å
A = +π =
π+4 4 π+4 4+π

å2
40 − 2π 20
Ç ã2
20 20 400 2
Å ã Å
π
A = +π = cm
π 4 π π

400
Thus (since the largest area is π ), the maximal area is obtained when all of the wire is used for the circle.

Calculus Total 40

4.5.4 Calculus Solutions August 2017

Question 1 - 6 marks

TRUE or FALSE:
94 2012–2020 Past Test and Examination Questions Booklet Mathematics I (Major)

Statement True/False

The derivative of cosh x is − sinh x. F

For a decreasing function, the right-most endpoint will give the height for the

lower Riemann sum. T

d
R x2 2
dx −5 e2t dt = 2xe2x T

Question 2 - 7 marks

(a) State and prove the Fundamental Theorem of Calculus 2. (7 marks)


Statement of Fundamental Theorem of Calculus 2:
If f is continuous on the closed interval [a, b] and F is an antiderivative of f on the interval [a, b], then

Z b
f (x) dx = F(b) − F(a).
a

Proof of Fundamental Theorem of Calculus 2:


Partition the interval [a, b] into n subintervals with endpoints

a = x0 < x1 < x2 < · · · < xn−1 < xn = b

of length ∆x = xi − xi−1 .
By pairwise subtraction and addition of like terms, we can write as a telescoping series

F(b) − F(a) = F(xn ) − F(xn−1 ) + F(xn−1 ) − · · · − F(x1 ) + F(x1 ) − F(x0 )


Xn
= [F(xi ) − F(xi−1 )].
i=1

Since F is an antiderivative, F is continuous on [xi−1 , xi ] and differentiable on (xi−1 , xi ) for each i. By the
Mean Value Theorem, applied to F on these subintervals, for each i = 1, . . . , n there exist ci ∈ (xi−1 , xi )
Mathematics I (Major) 2012–2020 Past Test and Examination Questions Booklet 95

such that
F(xi ) − F(xi−1 )
F 0 (ci ) =
xi − xi−1

F(xi ) − F(xi−1 )
=
∆x
F(xi ) − F(xi−1 )
⇒ f (ci ) = ∵ F is an antiderivative of f, F 0 (ci ) = f (ci )
∆x
⇒ f (ci ) ∆x = F(xi ) − F(xi−1 )
n
X Xn
⇒ f (ci ) ∆x = (F(xi ) − F(xi−1 )) = F(b) − F(a)
i=1 i=1
n
X
⇒ lim f (ci ) ∆x = F(b) − F(a)
n→∞
i=1
Z b
⇒ f (x) dx = F(b) − F(a).
a

Question 3 - 7 marks
(a) Find the area of the region bounded by the graphs y = x2 and y = c in terms of the positive constant c. (4
marks)

Z √
c ò √c
2 x3
(c − x ) dx = cx −

− c 3 √
− c
√ 3

√ ( c) î √ (− c)3 ó
=c c− − −c c−
3 3
√ ã
√ c c
Å
=2 c c−
3

c c
=4 .
3
Or use the fact that it is an even function.
Or integrate with respect to y.

(b) Write down an expression for the area enclosed by x = cos y, x = 12 , y = 0 and y = 2π. DO NOT
evaluate your integral. (3 marks)

1 π 5π
Points of intersection: cos y = 2 =⇒ y = 3 or 3
Z π Z 5π Z 2π
3 1 3 1 1
(cos x − ) dx − (cos x − ) dx + (cos x − ) dx.
0 2 π
3
2 5π
3
2

Question 4 - 7 marks
(a) Write down an expression for the volume of the solid generated by rotating the region bounded by
y = 1x + 2 and y = −x + 10 about the line y = 0. DO NOT evaluate your integral. (5 marks)
96 2012–2020 Past Test and Examination Questions Booklet Mathematics I (Major)

Points of intersection:

1 √
+ 2 = −x + 10 =⇒ x2 − 8x + 1 = 0 =⇒ x = 4 ± 15.
x

Thus the volume is given by:

Z √
4+ 15
1
Å ã
π √ (−x + 10)2 − ( + 2)2 dx.
4− 15 x

(b) Write down an expression for the volume of the solid generated by rotating the region bounded by
y = 1x + 2 and y = −x + 10 about the line y = 10. DO NOT evaluate your integral. (2 marks)

The volume in this case is:

Z √
4+ 15
1
Å ã
π √ (10 − ( + 2))2 − (10 − (−x + 10))2 dx.
4− 15 x

Question 5 - 3 marks

Find the cross-sectional area of a pyramid with a height of h and a rectangular base with dimensions w and b,
sliced perpendicular to the y-axis. Note that the y-axis is perpendicular to the base of the pyramid which is
positioned at y = 0.

Cross-sectional area:

bw(h − y)2
h2

b2 (h−y)2 w2 (h−y)2
(You can give 2 marks if they assume it is square (i.e., h2
or h2
.)
Mathematics I (Major) 2012–2020 Past Test and Examination Questions Booklet 97

Calculus Total 30 Marks

4.5.5 Calculus Solutions November 2017

Section A:
Multiple choice questions [18 marks]

Instructions:
Questions MCQ1–MCQ6 are multiple choice questions.
In each of these questions, select the letter that corresponds to the correct answer.

t4 + 9
(MCQ1) Which of the following is the appropriate partial fraction decomposition of :
t2 (t2 + 9)
t4 + 9 A Bt + C
(A) ≡1+ + 2
t2 (t2 + 9) t2 t +9
t4 + 9 At + B C D
(B) 2 2 ≡ 2
+ +
t (t + 9) t t+3 t−3
t4 + 9 A B Ct + D
X(C) 2 2 ≡1+ + 2 + 2
t (t + 9) t t t +9
t4 + 9 A B C D
(D) 2 2 ≡ + 2+ +
t (t + 9) t t t+3 t−3
(MCQ2) Which of the following statements is NOT TRUE?
Z 3
1 3
(A) 2
dx = .
0 (x − 4) 4
Z 3 Z 2 Z 3
1 1 1
(B) 2
dx = 2
dx + 2
dx.
0 (x − 4) 0 (x − 4) 2 (x − 4)
Z 3
1
X(C) 2
dx has an interior infinite discontinuity.
0 (x − 4)
Z 3
1
(D) dx converges.
0 (x − 4)2
1
(E) The integrand is defined on (0 , 3) . [3]
(x − 4)2

X cos (nπ)
(MCQ3) Consider the series .
n=1
n
Which of the following statements is TRUE?
(A) The series is not conditionally convergent.
(B) The series does not converge.
(C) The series is absolutely convergent.
cos (nπ)
ß ™
(D) The sequence diverges.
n
X∞ X∞
cos (nπ) 1
X(E) = (−1)n . [3]
n=1
n n=1
n

(MCQ4) Which of the following statements is TRUE about the series



X
ln x + (ln x)2 + (ln x)3 + · · · = (ln x)n .
n=1
98 2012–2020 Past Test and Examination Questions Booklet Mathematics I (Major)

1
X(A) The series converges for < x < e.
e
1
(B) The series converges for ≤ x ≤ e.
e
(C) The series does not converge for any value of x.
an+1 1
(D) = 1 + → 1 as n → ∞.
an n
(E) The series is convergent. [3]

(MCQ5) Let g(x, y) = x2 y + cos y + y sin x.


Which of the following statements is TRUE?

(A) g x = 2xy + cos y + ycos x.


(B) gy = 2x − sin y + sin x.
(C) g x x = 2y − sin y + sin x.
X(D) g x y = gy x .
(E) gy y = 2x − cos y. [3]

(MCQ6) Consider the differential equation (2xy − yn ) dx + x2 dy = 0.


Which of the following statements is NOT TRUE?

(A) For n = 1 the differential equation is variable separable.


(B) For n = 2 the differential equation is homogeneous of degree 2.
(C) For n = 0 the differential equation is exact.
X(D) For n = 0 the differential equation is variable separable.
(E) For n = 1 the differential equation is first order linear. [3]

Section B [72 marks]

In this section you are expected to SHOW ALL YOUR WORKING on the provided space to earn the
allocated marks.
X = 1 mark, (X) = 12 mark

Question 1 - 3 marks

1
(a) Find the partial fraction decomposition of . [3]
z (1 + z)
solution
1 A B
≡ + X
z+1 z 1+z
1 = A(1 + z) + B z.
When z = 0, A = 1 when z = −1, B = −1 X.
1 1 1
Thus = − X.
z+1 z 1+z

Z
ln z
(b) Evaluate dz. [7]
(1 + z)2
solution
1 1 1
Let u = ln z, du = z dz X dv = 1+z2
dz, v = − 1+z X
Mathematics I (Major) 2012–2020 Past Test and Examination Questions Booklet 99

R ln z ln z
R 1 1
(1+z)2
dz = − 1+z − (− 1+z z ) dz X X

ln z
R 1
= − 1+z + (1+z)z dz

ln z
R
= − 1+z + ( 1z − 1
1+z ) dz X

ln z
= − 1+z + ln|z| − ln|1 + z| + C X X

Question 2 - 3 marks

Evaluate the following integrals:


Z
t
(a) √ dt.
t4 −1 Z
Hint : You may use the fact that sec θ dθ = ln |sec θ + tan θ| + C. [9]
solution
√ √
2
Let
R t t = sec θ X. Then 2t dt = sec θ tan θ dθ X X,
R sec t4 − 1 = sec2 θ − 1 = tanθ X.
1 θ tan θ

4
dt = 2 tan θ dθ X X
t −1

1
R
= 2 secθ dθ X

1
= 2 ln|secθ + tanθ| + C

1

= 2 ln t2 + t4 − 1 + C X X

Z 2
2s − 2
(b) √ ds. [5]
1 −s2 + 2s + 3
solution
u = −s2 + 2s + 3

du = (−2s + 2)ds X

du = −(2s − 2)ds
R R
√ 2s−2 ds = − √1 du X
−s2 +2s+3 u

R 1
= − u− 2 du

1
= −2u 2 + C X


= −2 −s2 + 2s + 3 + C

Thus
100 2012–2020 Past Test and Examination Questions Booklet Mathematics I (Major)

R2 √
√ 2s−2 ds = − −s2 + 2s + 3 |21
1 −s2 +2s+3

î√ √ ó
= −2 −4 + 4 + 3 − −1 + 2 + 3 X

Ä√ ä
= −2 3−2


= 4−2 3X

Question 3 - 3 marks
Z
1
(a) Evaluate dx. [4]
(1 + x2 )arctan x
solution
1
u = arctan x, du = dx X, (1 + x2 )du = dx
R 1
R1 +1 x2
(1+x2 )arctan x
dx = u du X

= ln | u | +C X

= ln | arctan x | +C X

Z 1
1
(b) Hence or otherwise determine whether the integral dx
0 (1 + x2 )arctan x
is convergent or divergent. [4]
solution
R1 1
R1 1
0 (1+x2 )arctan x
dx = lim+ t (1+x2 )arctan x
dx X X
t→0

= lim ln |arctanx|1t
t→0+

= lim ln(1) − ln | arctant |


t→0+

= lim −ln | arctant | X


t→0+

= ∞X

Z 1
1
Thus dx is divergent X.
0 (1 + x2 )arctanx

Question 4 - 3 marks
X∞ Å
1 2 1
ã
Consider the telescoping series − + .
r=1
r+2 r+1 r

Xn Å
1 2 1
ã
(a) Find the partial sum S n = − + . [5]
r=1
r+2 r+1 r
solution
Mathematics I (Major) 2012–2020 Past Test and Examination Questions Booklet 101

n
P n
P n
P
1 1 1
Sn = r+2 −2 r+1 + r X
r=1 r=1 r=1

n
P n
P n
P
ï ò
1 1 1 1 1 1 1
= r −1− 2 + n+1 + n+2 −2 r −1+ n+1 + r XXX
r=1 r=1 r=1

n
P n
P n
P
1 3 1 1 1 2 1
= r − 2 + n+1 + n+2 −2 r +2− n+1 + r
r=1 r=1 r=1

1 1 1
= 2 − n+1 + n+2 X

X∞ Å
1 2 1
ã
(b) Hence or otherwise show that the series − +
r=1
r+2 r+1 r
converges and find its sum. [3]
solution
X∞ Å
1 2 1 1 1 1 1
ã Å ã
− + = lim − + X = X.
r=1
r+2 r+1 r n→∞ 2 n+1 n+2 2
X∞ Å
1 2 1 1
ã
Therefore − + converges to X.
r=1
r + 2 r + 1 r 2

Question 5 - 3 marks

n3 − 1 + n2 sin n
ß ™
(a) Determine, giving reasons, whether the sequence {an } =
1 + 3n3
is convergent or divergent.
If it is convergent, find the value to which it converges. [4]
solution
1
n3 −1+n2 sin n 1− + sinn n
n3
lim 1+3n3
= lim 1 XX
n→∞ n→∞ n3
+3

1
= 3 (X)

1
Since lim = (X) then {an } is convergent X.
n→∞ 3

(b) Determine whether the following series converge or diverge


X n3 − 1 + n2 sin n
(i) . [2]
n=1
1 + 3n3
solution
X∞
1 n3 − 1 + n2 sin n
Since lim = , 0 X from part (a), therefore is divergent X.
n→∞ 3 n=1
1 + 3n3
102 2012–2020 Past Test and Examination Questions Booklet Mathematics I (Major)


X ãn
n3 − 1 + n2 sin n
Å
(ii) (−1)n . [4]
n=1
1 + 3n3
solution
By Root Test:
n − 1 + n2 sin n n n3 − 1 + n2 sin n
Å 3
1
ã
n
(−1)n 3
X= → by part (a) X.
1 + 3n 1 + 3n3 3
Since 31 < 1 X, by root test, the series converges X.

Question 6 - 3 marks
X ∞
1
Given that = 1 + x + x2 + x3 + · · · = xn .
1−x n=0

1
(a) Write down the power series of . [4]
1 + x2
solution
1 1
1+x2
= 1−(−x 2) X

= 1 − x2 + x4 − x6 + · · · XX


P
= (−1)n x2n X
n=0

(b) Hence or otherwise find a power series representation of f (x) = arctan x. [4]
solution R 1
arctan x + C = 1+x 2 dx X

R P∞
n
= n=0 (−1) x2n dx X


P x2n+1
=C+ (−1)n 2n+1 XX
n=0

π
(c) Write as a series. [2]
4
solution
Putting c = 0, x = 1,
X ∞
π 1
= arctan 1 X = (−1)n X.
4 n=0
2n +1

Question 7 - 3 marks
Ä ä2 Ä ä2
x x
dx y2 + y2 e y + 2x2 e y
Consider the differential equation = .
dy
Ä ä2
x
2x y e y

(a) Using the substitution x = vy or otherwise, find the general solution of the
Mathematics I (Major) 2012–2020 Past Test and Examination Questions Booklet 103

differential equation. [10]


solution
dx dv
x = vy; =v+y XX
dy dy
2 x 2
y2 +y2 e( y ) +2x2 e( y )
x
dx
dy = x 2
2xye( y )

2 2
dv y2 +y2 ev +2v2 y2 ev
v + y dy = XX
2vy2 ev2

2 2
1+ev +2v2 ev
= X
2vev2

2 2 2
dv 1+ev +2v2 ev −2v2 ev
y dy =
2vev2

2
dv 1+ev
y dy = XX
2vev2

Z 2 Z
2vev 1
2
dv = dy X
1+e v y
2
ln 1 + ev = ln |y| + C X
Ä ä2
x
ln 1 + e y = ln |y| + C X

(b) Find the particular solution of the differential equation given that y = 1 when x = 0. [2]
solution
y = 1, x = 0
ln 1 + e0 = ln(1) + C. Thus C = ln(2) (X).
The particular solution of the differential equation
Ä ä2
x
ln 1 + e y = ln |y| + ln2 (X)
Ä ä2
x
ln 1 + e y = ln |2y|
Ä ä2
x
1+e y = 2y X

4.6 Calculus Solutions 2018

4.6.1 Diagonal D Calculus Solutions March 2018

Question 1 - 4 marks

TRUE or FALSE:
Let c ∈ R and a ∈ R.
104 2012–2020 Past Test and Examination Questions Booklet Mathematics I (Major)

Statement True/False

lim |x| = 1. T
x→1

lim bxc = 1 F
x→1−

lim c = c T
x→c

Let lim f (x) = L and lim g(x) = M both exist, for L, M ∈ R.


x→a x→a

f (x) L
Then lim = M F
x→a g(x)

Marker’s comments:
For the 4th statement, M , 0 should be present.

Question 2 - 3 marks
−1
(a) Determine whether the function f (x) = x2
has a vertical asymptote at x = 0. Show all your working.
(3 marks)
Solution
We have
lim f (x) = −∞
x→0−

and thus the function f (x) has a vertical asymptote at x = 0.


(NOTE: Any such limit will do. You do not need to take the limit on both sides.)
Marker’s comments:
1. Saying the limit does not exist (or the denominator is zero) is not sufficient, you need to say if it is
positive or negative infinity.
2. Lost half a mark if wrote ”= 01 ”. You need to express this concept using limits.
3. Please sort out notation! The symbols =, ⇒, and ⇔ do NOT mean the same thing.
4. Writing lim without following it with an expression whose limit you want to find does not make
x→0
sense!
5. If two lines are equal SAY SO.
6. Be careful not to confuse an asymptote and a tangent.
7. Lost a mark if you said the limit was positive infinity.
8. Lost marks if you didn’t say what your working showed (i.e., is there a vertical asymptote or not)
Mathematics I (Major) 2012–2020 Past Test and Examination Questions Booklet 105

Question 3 - 8 marks
Prove that
1 − cos θ
lim = 0.
θ→0 θ
You may assume that lim sinθ θ = 1. (8 marks)
θ→0

Proof

1 − cos θ 1 − cos θ 1 + cos θ


Å ãÅ ã
lim = lim
θ→0 θ θ→0 θ 1 + cos θ
1 − cos2 θ
= lim
θ→0 θ(1 + cos θ)

sin2 θ
= lim
θ→0 θ(1 + cos θ)
sin θ sin θ
Å ãÅ ã
= lim
θ→0 θ 1 + cos θ
sin θ sin θ
Å ãÅ ã
= lim lim
θ→0 θ θ→0 1 + cos θ
ã lim sin θ
sin θ
Å
θ→0
= lim
θ→0 θ lim(1 + cos θ)
θ→0
lim sin θ
sin θ
Å ã
θ→0
= lim
θ→0 θ 1 + lim cos θ
θ→0
0
= (1)
1+1
= 0.

Marker’s comments:

1. You cannot use the statement in the proof (almost everyone who started with
2 2
lim 1−cos
θ
θ
= lim sin θ+cosθ θ−cos θ did this at some point)
θ→0 θ→0

2. Marks deducted for bad notation errors. Please fix in future! Ask before the next test if you are not sure.
sin θ 0
3. You need to show that lim 1+cos θ = 2 since if cos θ → −1 this limit has an indeterminate form. (1 mark off)
θ→0
106 2012–2020 Past Test and Examination Questions Booklet Mathematics I (Major)

Question 4 - 9 marks

(a) Complete the sentence:


The function g(x) is continuous on the interval [−1, 3) if and only if.... (2 marks)

Solution ... g(x) is continuous on the open interval (−1, 3) and g(x) is right-continuous at the point x = −1.

(b) For which value(s) of a ∈ R is the function

 2

 x +a for x<2

f (x) = 8 for x=2

 x3

for x>2

continuous on its entire domain?


Show your working and explain your reasoning. (6 marks)

Solution
The function f (x) is continuous on the intervals (−∞, 2) and (2, ∞) because it is a polynomial on those
intervals.
If a = 4 then lim− f (x) = lim+ f (x) = f (2) = 8 and hence f (x) is continuous at the point x = 2.
x→2 x→2
Therefore, f (x) is continuous everywhere on its domain (all real numbers).

Marker’s comments:
Don’t forget to consider the entire domain and not just the point x = 2.

(c) If a = 5 in part (b) above, then what kind of discontinuity exists at x = 2? (1 mark)

Solution
Jump discontinuity.

Question 5 - 7 marks

Use first principles to find f 0 (2) if f (x) = x2 + 3.

Solution
Mathematics I (Major) 2012–2020 Past Test and Examination Questions Booklet 107

√ √
0 x2 + 3 − 22 + 3
f (2) = lim
x→2 x−2
√ √
x2 + 3 − 7
= lim
x→2 x−2
√ √ Ç√ √ å
x2 + 3 − 7 x2 + 3 + 7
= lim √ √
x→2 x−2 x2 + 3 + 7
x2 + 3 − 7
= lim √ √
x→2 (x − 2)( x2 + 3 + 7)
x2 − 4
= lim √ √
x→2 (x − 2)( x2 + 3 + 7)
(x − 2)(x + 2)
= lim √ √
x→2 (x − 2)( x2 + 3 + 7)
x+2
= lim √ √
x→2 x2 + 3 + 7
2+2 4 2
= √ √ = √ = √ .
22 + 3 + 7 2 7 7

OR:


(2 + h)2 + 3 − 22 + 3
p
0
f (2) = lim
h→0 h

(2 + h)2 + 3 − 7
p
= lim
h→0 h
√ Çp √ å
(2 + h)2 + 3 − 7 (2 + h)2 + 3 + 7
p
= lim √
h (2 + h)2 + 3 + 7
p
h→0

(2 + h)2 + 3 − 7
= lim √
h( (2 + h)2 + 3 + 7)
p
h→0

4 + 4h + h2 − 4
= lim √
h→0 h( (2 + h)2 + 3 + 7)
p

h(4 + h)
= lim p √
h→0 h( (2 + h)2 + 3 + 7)
4+h
= lim p √
h→0 (2 + h)2 + 3 + 7
4 4 2
= p √ = √ = √ .
(2)2 + 3 + 7 2 7 7

Marker’s comments:
√ √ √
1. a+b, a+ b

2. Notation: be careful where you put your limits!


3. The limit of an indeterminate form ( 00 ) is not equal to zero (necessarily).
108 2012–2020 Past Test and Examination Questions Booklet Mathematics I (Major)

Question 6 - 4 marks
Determine whether the function f (x) = |2x − 5| has a vertical tangent at 25 . Explain why you came to that
conclusion.

Solution
5
No, f (x) does not have a vertical tangent at x = 2 since

5
2x − 5 if x >
(
2
f (x) =
−(2x − 5) if x < 52

and thus
lim+ | f 0 (x)| = lim+ |2| = 2 and lim− | f 0 (x)| = lim− | − 2| = 2,
x→ 52 x→ 25 x→ 52 x→ 25

and so
lim | f 0 (x)| = 2.
x→ 25

Consequently, f does not meet the criterion that

lim | f 0 (x)| = ∞
x→ 52

and therefore does not have a vertical tangent at x = 25 .


(NB - student does not need to provide all these details, as long as they have the right idea.)
Marker’s comments:

1. Check conditions for a vertical tangent.

2. You are not looking for where a function has a cusp (sharp point) or is zero, or where a line cuts it once.
3. It is NOT TRUE to write:
2x − 5 if x ≥ 0
(
|2x − 5| =
−2x + 5 if x < 0

Question 7 - 2 marks
(a) Is the function g(x) = tan x + 3 continuous at x = π2 ?

Solution No. (It is not defined there.)

(b) Using part (a) above, what can you say about the differentiability of tan x + 3 at x = π2 ?

Solution The function tan x + 3 is not differentiable at x = π2 .


Mathematics I (Major) 2012–2020 Past Test and Examination Questions Booklet 109

Question 8 - 3 marks

y
6
-x

The function g(x)

Above you are given the graph of the function g(x). Circle the letter of ONE of the graphs below which gives the
most accurate sketch of the derivative of g(x).

y y
6 6
-x -x

A: The function g0 (x)B:


Award 1 mark.g0 (x) Award 2 marks.
The function

y y
6 6
-x -x

C: The function g0 (x)D:


Award 3 marks.g0 (x) Award 0 marks.
The function

Calculus Total 40 Marks

4.6.2 Diagonal E Calculus Solutions March 2018

Question 1 - 4 marks

TRUE or FALSE:

Let c ∈ R.
110 2012–2020 Past Test and Examination Questions Booklet Mathematics I (Major)

Statement True/False

If lim− f (x) = 5 and lim+ f (x) = −5 then lim f (x) = |5|. F


x→1 x→1 x→1

limbxc = 4 F
x→4

lim 0 = 0 F
x→c

If n ∈ N then lim xn = cn T
x→c

Question 2 - 4 marks
Evaluate the limit
3x
lim √ .
x→0 1 − 2x − 1
Solution

Ç√ å
3x 3x 1 − 2x + 1
lim √ = lim √ √
x→0 1 − 2x − 1 x→0 1 − 2x − 1 1 − 2x + 1

3x( 1 − 2x + 1)
= lim
x→0 1 − 2x − 1

3x( 1 − 2x + 1)
= lim
x→0 −2x

3( 1 − 2x + 1)
= lim
x→0 −2

3( 1 + 1)
= = −3.
−2

Question 3 - 3 marks
1
Determine whether the function f (x) = x has a vertical asymptote at x = 0. Show all your working.

Solution
We have that

lim f (x) = −∞,


x→0−
Mathematics I (Major) 2012–2020 Past Test and Examination Questions Booklet 111

And hence f has a vertical asymptote at x = 0.


(Or show any similar limit goes to ±∞.)

Question 4 - 9 marks
(a) Complete the sentence:
The function g(x) is continuous from the left at c if.... (2 marks)

Solution

The function g(x) is continuous from the left at c if lim− f (x) = f (c).
x→c

(b) For which value(s) of a ∈ R is the function

 3

 x for x<1

f (x) = a for x=1

 x2

for x>1

continuous on its entire domain?


Show your working and explain your reasoning. (6 marks)
Solution
The function f (x) is continuous on the intervals (−∞, 1) and (1, ∞) as it is a polynomial there.
If a = 1 then

lim f (x) = lim+ f (x) = f (1) = 1.


x→1− x→1

Thus if a = 1 then the function f is continuous on its entire domain (all real numbers).

(c) If a = 5 in part (b) above then what kind of discontinuity exists at x = 1? (1 mark)

Solution
Removable discontinuity.

Question 5 - 7 marks
2
Use first principles to find f 0 (2) if f (x) = x2 +3
.
112 2012–2020 Past Test and Examination Questions Booklet Mathematics I (Major)

Solution
2 2 2 2
(2+h)2 +3
− 22 +3 (2+h)2 +3
− 7
f 0 (2) = lim = lim
h→0 h h→0 h
2(7) − 2((2 + h)2 + 3) 1
Å ã
= lim
h→0 7((2 + h)2 + 3) h
14 − 2(4 + 4h + h2 + 3)
= lim
h→0 7h(4 + 4h + h2 + 3)
−2h(4 + h)
= lim
h→0 7h(7 + 4h + h2 )
−2(4 + h) −2(4) −8
= lim 2
= = .
h→0 7(7 + 4h + h ) 7(7) 49

OR:

2 2 2
− −2
x2 +3
= lim x +3 7
22 +3
2
f 0 (2) = lim
x→2 x−2 x→2 x − 2
2
2(7) − 2(x + 3) 1
Å ã
= lim
x→2 7(x2 + 3) x−2
2
14 − 2x − 6
= lim
x→2 7(x − 2)(x2 + 3)

−2(x2 − 4)
= lim
x→2 7(x − 2)(x2 + 3)
−2(x − 2)(x + 2)
= lim
x→2 7(x − 2)(x2 + 3)
−2(x + 2) −2(2 + 2) −8
= lim = = .
x→2 7(x2 + 3) 7(22 + 3) 49

Question 6 - 4 marks
A function f (x) has a vertical tangent at a provided that which three conditions hold?
Solution

• f is continuous at a,

• f is differentiable near a, i. e., f 0 (x) exists for x , a with x close to a, and

• lim | f 0 (x)| = ∞.
x→a

Question 7 - 6 marks
Prove that if a function f (x) is differentiable at x = a, then f (x) must be continuous at x = a.
HINT: begin as follows:
If x is in the domain of f and x , a, then f (x) may be written as

f (x) − f (a)
Å ã
f (x) = f (a) + (x − a).
x−a

Solution
Mathematics I (Major) 2012–2020 Past Test and Examination Questions Booklet 113

Then (using hint given above)

f x) − f (a)
ï Å ã ò
lim f (x) = lim f (a) + (x − a)
x→a x→a x−a
f x) − f (a)
ïÅ ã ò
= lim f (a) + lim (x − a)
x→a x→a x−a
f x) − f (a)
Å ã
= f (a) + lim lim(x − a)
x→a x−a x→a
0
= f (a) + f (a) lim(x − a)
x→a
0
= f (a) + f (a) · 0
= f (a).

Hence f is continuous at x = a.

Question 8 - 3 marks
y
6
-x

The function g(x)

Above you are given the graph of the function g(x). Circle the letter of ONE of the graphs below which gives the
most accurate sketch of the derivative of g(x).
y y
6 6
-x -x

A: The function g0 (x):B:Award 0 marks


The function g0 (x): Award 2 marks
y y
6 6

-x -x

C: The function g0 (x):D:Award 1 markg0 (x): Award 3 marks


The function

Calculus Total 40 Marks

4.6.3 Calculus Diagonal D Solutions April 2018

Total marks [40 marks]

Question 1 - 3 marks
Find the linearization of the function f (x) = arctan(x) near 1 and use this linearization to approximate
arctan(1.01)
Leave your final answer as a fraction. 6 marks
π
f (x) = arctan x, f (1) = arctan(1) = 4
1 1
f 0 (x) = f 0 (1) =
1 + x2 2
114 2012–2020 Past Test and Examination Questions Booklet Mathematics I (Major)

π 1
L(x) = f (1) + f 0 (1)(x − 1) = + (x − 1)
4 2
π 1 π 1 π 1 50π + 1
arctan(1.01) ≈ L(1.01) = + (1.01 − 1) = + (1.01) = + =
4 2 4 2 4 200 200

Question 2 - 3 marks
Let x2 y = 1 + xy2 . Find y0 . 5 marks
d 2 d
(x y) = (1 + xy2 )
dx dx
2xy + x2 y0 = y2 + x · 2y · y0
(x2 − 2xy)y0 = y2 − 2xy
y2 − 2xy
y0 = 2 .
x − 2xy

Question 3 - 3 marks

Let f (x) = c x − log10 x. For what value(s) of c is f 0 12 = 10? 4 marks


1 ln x
f (x) = cx −
2 ln 10
0 1 1
f (x) = c − ·
2 ln 10 x
1 1 1 1 1
f 0 ( ) = 10 ⇔ c − · = 10 ⇔ c − = 10 ⇔ c = 10 + .
2 2 ln 10 12 ln 10 ln 10

Question 4 - 3 marks
 √
Let g(x) = x arcsin 4x + 16 − x2 .
Find g0 (2).
Simplify your answer. 7 marks
x x
y = arcsin ⇔ sin y = ,
4 4
1 1 1 1 4 1
⇒ cos y · y0 = ⇒ y0 = · ⇒ y0 = · √ = √ .
4 4 cos y 4 16 − x 2 16 − x2
 x 1 2x
Thus g0 (x) = arcsin +x· √ − √ .
4 16 − x 2 2 16 − x2
 x
g0 (x) = arcsin
Å4 ã
2 1 π
Å ã
0
g (2) = arcsin = arcsin = .
4 2 6

Question 5 - 3 marks
Z
1
Show that dx = ln |x| + C. 5 marks
x
Let f (x) = ln |x|.
ln(x) ; x > 0
®
Then f (x) = ln |x| =
ln(−x) ; x < 0
® 1
x ; x >0
f 0 (x) = 1
,
− −x ; x>0
® 1
0 x ; x >0
f (x) = 1
.
x ; x >0
Mathematics I (Major) 2012–2020 Past Test and Examination Questions Booklet 115

1
Then f 0 (x) = ; x , 0.
Z x
1
Thus dx = ln |x| + C.
x

Question 6 - 3 marks
Evaluate the following integrals:
Z
2t − 2
(a) √ dt. 4 marks
−t2 + 2t + 3
Z Z
2t − 2 −2t + 2
√ dt = − √ dt.
−t2 + 2t + 3 −t2 + 2t + 3
Let u = Z−t2 + 2t + 3 ; du = (−2t +Z2) dt.
−2t + 2 1 1
Then − √ dt = − √ du = −2u 2 + C.
Z −t2 + 2t + 3 u
2t − 2 √
Thus √ 2
dt = − −t + 2t + 3 + C.
−t2 + 2t + 3
Z
4 + z2
(b) √ dz. 4 marks
z
R 4+z2 R 1 1
√ dz = (4z− 2 + z2 · z− 2 ) dz
z
R 1 R 3
= 4 (z− 2 ) dz + z 2 dz
4 1 1 5
= 1 z2 + 3 z2 + C
2 2 +1
1 2 52
= 8z 2 + 5 z + C
Z 2
(c) (x − 1)e(x−1) dx. 5 marks
0
Z 2 Z
(x − 1)e(x−1) dx = tet dt.
0
uZ = t , du = dt Z dv = et dt v = et ,
tet dt = tet − et dt = tet − et + C,
Thus
R2
0
(x − 1)e(x−1) dx = e x−1 (x − 1 − 1)|20
= e(0) − e−1 (−2)
= 2e−1
116 2012–2020 Past Test and Examination Questions Booklet Mathematics I (Major)

4.6.4 Calculus Diagonal E Solutions April 2018

Total marks [40 marks]

Question 1 - 3 marks
Show that ddx (e x ) = e x . 5 marks
e x −1
Hint: You may use the fact that lim = 1.
x→0 x
d x e x+h −e x
dx (e ) = lim
h→0 h
x eh −1
= lim e ( h )
h→0
h
= e x lim e h−1
h→0
x
=e ·1
= ex

Question 2 - 3 marks
Let f (x) = logb (3x2 − 2) where b is a positive real number different from 1.
For what value(s) of b is f 0 (1) = 3? 4 marks
f (x) = log b (3x2 − 2)
ln(3x2 −2)
= ln b

f 0 (x) = ln1b 3x6x


2 −2

1 6
f 0 (1) =3⇔ ln b 1 =3
⇔ ln b = 2
⇔ b = e2

Question 3 - 3 marks
π x2
Å ã
Find the equation of the tangent line to the curve y = 2tan at the point (1, 2). 6 marks
4
2
y0 = 2sec2 ( π4x ) · 2π x
4
2
= sec2 ( π4x ) · π x
y0 (1) = sec2 ( π4 ) · π
1
= cos2 ( π4 )
·π
1
= ( √1 )2
· π
2
= 2π
Then y − 2 = 2π(x − 1) and y = 2π(x − 1) + 2

Question 4 - 3 marks
dy
Find if arctan(x + y) = xy2 . 6 marks
dx
1
1+(x+y)2
(1 + y0 ) = y2 + x 2y y0
1 y0
(x+y)2
+ 1+(x+y)2
= y2 + 2x y y0
1 1
( 1+(x+y)2)y
0
= y2 − 1+(x+y)2
Mathematics I (Major) 2012–2020 Past Test and Examination Questions Booklet 117

Question 5 - 3 marks

Let f (x) = cos x.


π
Find the third degree Taylor polynomial of f (x) at x = . 6 marks
3
1
f (x) = cos x f ( π3 ) = cos ( π3 ) =2

f 0 (x) = −sin x f 0 ( π3 ) = −sin ( π3 ) = − 2 3
f 00 (x) = −cos x f 00 ( π3 ) = −cos ( π3 ) = −1 2

f 000 (x) = sin x f 000 ( π3 ) = sin ( π3 ) = 23

(x− π3 )2 (x− π )3
T 3 (x) = f ( π3 ) + f 0 ( π3 )(x − π3 ) + f 00 ( π3 ) 2 + f 000 ( π3 ) 3!3
√ π 2 √ π 3
1 3 1 (x− 3 ) 3 (x− 3 )
T 3 (x) = 2 − 2 (x − π3 ) − 2 2 + 2 6

Question 6 - 3 marks

Evaluate the following integrals:

Z 2
x+2
(a) √dx. 6 marks
Z x2 + 4x
1
Z 1 Z
x+2 2 (2x + 4) 1 1 √
√ dx = √ dx = √ du = u + C.
2
x + 4x 2
x + 4x 2 u
Z 2 √ √ √
x+2 2
Thus √ dx = ( x2 + 4x)|1 = 12 − 5.
1 x2 + 4x
Z

(b) x ln x dx. 7 marks



 u = ln x  dv = x dx
 
 
du = 1x dx du = 1x dx
3
x du = dx = 32 x 2
 
v
 

Then
R √ 3 R 3
x ln x dx = 23 x 2 ln x − 2
x 2 · 1x dx
3 R3 1
= 32 x 2 ln x − 2
3 x dx2

3 2 2 23
= 32 x 2 ln x − 3 · 3x +C
3 4 32
= 23 x 2 ln x − 9x +C

4.6.5 Calculus Solutions June 2018

Full workings must be shown.


Leave answers in exact form.

Question 1 [4]

x
Differentiate y = x .
118 2012–2020 Past Test and Examination Questions Booklet Mathematics I (Major)


ln y = ln x x

ln y = x ln x
d d √
ln y = x ln x
dx dx
y0 1 √ 1
= √ ln x + x
y 2 x x
√ 1 1
Å ã
0 x
y =x √ ln x + 1
x 2

Question 2 [3]

x ∂f ∂f
Å ã
If f (x, y) = sin , calculate and .
1+y ∂x ∂y
∂f x 1
Å ã
= cos
∂x 1+y 1+y
∂f x −x
Å ã
= cos
∂x 1 + y (1 + y)2
Question 3 [4]
Evaluate lim (x − ln x).
x→∞

ln x 1
Å ã
lim (x − ln x) = lim x 1 −
x→∞ x→∞ x 2
ln x
Å ã
= lim x lim 1 − lim
x→∞ x→∞ x→∞ x
1
Ç å
l’H x
= lim x 1 − lim
x→∞ x→∞ 1
=∞(1 − 0)
=∞

Question 4 [16]
Let f (x) = (4 − x2 )5 .
(a) Where does f cut the x-axis? (1)

0 =(4 − x2 )5
±2 =x
Mathematics I (Major) 2012–2020 Past Test and Examination Questions Booklet 119

OR

(−2, 0) and (2, 0)

(b) Where does f cut the y-axis? (1)

y = 45
OR

(0, 45 )

(c) What are the critical number(s) of f ? (3)

f 0 (x) = 5(4 − x2 )4 (−2x)

0 = 5(4 − x2 )4 (−2x)
x = −2, 0, 2
120 2012–2020 Past Test and Examination Questions Booklet Mathematics I (Major)

(d) State the open interval(s) where f is increasing, decreasing or constant. (3)

(−∞, −2) -2 (−2, 0) 0 (0, 2) 2 (2, ∞)

−10x + + + 0 - - -

4 − x2 - 0 + + + 0 -

−10x(4 − x2 ) - 0 + 0 - 0 +

INC/DEC & → % → & → %

f is increasing on (−2, 0) and (2, ∞)


decreasing on (−∞, −2) and (0, 2)
and constant at x = −2, 0, 2.
(e) If f has local extrema, give point/s at which they occur and specify if the local extrema is a minimum or
maximum. Otherwise, state if it has none. (2)
Local minima at (−2, 0) and (2, 0)
Local maximum at (0, 45 )
(f) State the open interval(s) where f is concave up or concave down and point(s) of inflection. (6)

f 00 (x) = − 10 (4 − x2 )4 + x4(4 − x2 )3 (−2x)


 

= − 10(4 − x2 )3 4 − x2 + x4(−2x)
 

= − 10(4 − x2 )3 4 − 9x2
 

» » » » » »
4 4 4 4 4 4
(−∞, −2) -2 (−2, − 9) − 9 (− 9, 9) 9 ( 9 , 2) 2 (2, ∞)

−10 - - - - - - - - -

(4 − x2 )3 - 0 + + + + + 0 -

4 − 9x2 - - - 0 + 0 - - -

−10(4 − x2 )3 4 − 9x2 - 0 0 - 0 0 -
 
+ +

CONCAVITY CD POI CU POI CD POI CU POI CD

» »
The function f is concave up on (−2, − 49 ) and ( 49 , 2) and
» »
concave down on (−∞, −2), (− 49 , 49 ) and (2, ∞). Points of inflection are at

(−2, (4 − (−2)2 )5 ) = (−2, 0)

, Ç … å Ç … å
4 4 25 4 20 5

− , (4 − (− )) = − ,( ) )
9 9 9 9
(2, (4 − (2)2 )5 ) = (2, 0)
Mathematics I (Major) 2012–2020 Past Test and Examination Questions Booklet 121

, Ç… å Ç… å
4 4 25 4 20 5

, (4 − ( ) ) = ,( ) )
9 9 9 9

Question 5 [6]
State and prove Fermat’s theorem.
If f has a local maximum or minimum at an interior point c of dom( f ) and if f is differentiable at c, then
f 0 (c) = 0.

Proof. Assume f has a local maximum at c.


Let a, b be such that the interval (a, b) belongs to the domain of f , a < c < b and f (x) ≤ f (c) for x ∈ (a, b), i. e.,

f (x) − f (c) ≤ 0.

Hence
f (x) − f (c)
≥0 if x ∈ (a, c)
x−c
and
f (x) − f (c)
≤0 if x ∈ (c, b).
x−c
Since f is differentiable at c, it follows that

f−0 (c) ≥ 0 and f+0 (c) ≤ 0


0
⇒ f (c) = f−0 (c) = f+0 (c) = 0.

A similar proof holds if f has a minimum at c.


OR

Proof. Assume f has a local minimum at c.


Let a, b be such that the interval (a, b) belongs to the domain of f , a < c < b and f (x) ≥ f (c) for x ∈ (a, b), i. e.,

f (x) − f (c) ≥ 0.

Hence
f (x) − f (c)
≤0 if x ∈ (a, c)
x−c
and
f (x) − f (c)
≥0 if x ∈ (c, b).
x−c
Since f is differentiable at c, it follows that

f−0 (c) ≤ 0 and f+0 (c) ≥ 0


0
⇒ f (c) = f−0 (c) = f+0 (c) = 0.

A similar proof holds if f has a maximum at c.


122 2012–2020 Past Test and Examination Questions Booklet Mathematics I (Major)

Question 6 [7]
A conical tank 8 meters across the top and 6 meters deep is full of water. The tank springs a leak at the bottom
and loses water at a rate of 2 cubic meters per min. How fast is the water level dropping at the instance when the
water is exactly 3 meters deep?

Solution. Let r be the radius of the remaining cone of water and h its corresponding height.

r
6

Then r and h are related by similar triangles,


r 4 2
= = .
h 6 3
dV
Since the cone is losing water, the volume V of the cone decreases at the given rate so that = −2. The
dt
dh 1 2 2h
objective is to find . Since the volume of a cone is V = πr h with r = we have
dt h=3 3 3
1 2
V= πr h
3
Å ã2
1 2h
= π h
3 3
4 3
∴V= πh
27
dV 4 dh
⇒ = π(3)h2
dt 27 dt
4 2 dh
= πh
9 dt
4 2 dh
⇒ −2 = πh
9 dt
4 2 dh
⇒ −2 = π(3 )
9 dt h=3
dh 1
⇒ =− .
dt h=3 2π
1
Hence, at the instant when the water is exactly 3 meters deep, the water level is dropping at a rate of meters

per minute.

Total :[40] marks


Mathematics I (Major) 2012–2020 Past Test and Examination Questions Booklet 123

4.6.6 Calculus Solutions August 2018

See extra pages of booklet.

4.6.7 Calculus Solutions November 2018

Section A:
[12 marks]

Instructions:
Answer Questions A1–A6 with True T or False F.
In each of the questions, select one choice by encircling the letter T or F.
Each question is worth 2 marks.
Z 0
1
(A1) Let g be a function defined by g(x) = dt.
−x 1 + t2
0 1
Then g (x) = − . T FX
1 + x2

Z 0
2x
(A2) The integral dx is an improper integral. TX F
−5 1 − x2


X ∞
X
cos (nπ) 1
(A3) = (−1)n . TX F
n=1
n n=1
n

X ∞
1
(A4) If = 1 + x + x2 + x3 + · · · = xn , then
1−x n=0
X∞
1 2 4 6
= 1 + x + x + x + · · · = x2n . T FX
1 + x2 n=0

(A5) Let g(x, y) = x2 y + cos y + y sin x.

Then gy = 2x − sin y + sin x. T FX

Z
sin 18 t
(A6) dt = 2sin 9t + C. T FX
sin 9t
124 2012–2020 Past Test and Examination Questions Booklet Mathematics I (Major)

Section B [78 marks]

In this section you are expected to SHOW ALL YOUR WORKING on the
provided space to earn the allocated marks.

Question 1 - 13 marks
Z t2
2
Consider the function g(t) = (x5 − 4 · 3 x ) dx.
−t

0
Find g (t). [4]

4 2
g0 (t) = (t10 − 4 · 3t ) 2t − (−t5 − 4 · 3t )(−1)

Question 2 - 13 marks
Evaluate the following integrals:
Z
2t − 3
(a) √ dt. [7]
4 − t2
Z Z Z
2t − 3 2t 1
√ dt = √ dt − 3 √ dtX
4−t 2 4−t 2 4 − t2
u = 4 − t2 , du = −2t dtX
R R
√ 2t dt =− √1 du
4−t2 u

R 1
= − u− 2 duX


= −2 u + CX


= −2 4 − t2 + CX

R
√1 dt = arcsin 2t + DX
4−t2

Z √
2t − 3 t
Thus √ dt = −2 4 − t2 − 3 arcsin + K.X
4−t 2 2
Mathematics I (Major) 2012–2020 Past Test and Examination Questions Booklet 125

Z
sin3 x cos4 x
(b) √ dx. [6]
cos x
R 3
sin√ x cos4 x
R sin x (sin2 x)cos4 x
cos x
dx = √
cos x
dxX

R sin x (1−cos2 x)cos4 x


= √
cos x
dxX

Let u = cos x; du = −sin x dxX


R sin x (1−cos2 x)cos4 x
R (1−u2 ) u4

cos x
dx =− √
u
du(X)

R 1 1
= (u6 u− 2 − u4 u− 2 ) du

Then R 11 7
= (u 2 − u 2 ) du(X)

2 13 9
= 13 u
2 − 29 u 2 + CX

R 3
sin√ x cos4 x 2 √ √
Thus cos x
dx = 13 ( cos x)13 − 29 ( cos x)9 + C.X

Question 3 - 13 marks
2u2
(a) Express in its partial fraction decomposition. [8]
(u − 1)2 (u + 1)2
2u2 A B C D
≡ + + + XX
(u − 1)2 (u + 1)2 u − 1 (u − 1)2 u − 1 (u − 1)2

⇔ 2u2 = A(u − 1)(u + 1)2 + B(u + 1)2 + C(u + 1)(u − 1)2 + D(u − 1)2

1
Let u = 1 2 = B(1 + 1)2 ⇒B= X
2
1
Let u = −1 2 = D(−1 − 1)2 ⇒D= X
2

Equating coefficients:

u3 : 0 = A+C (i)
1 1
constants : 0 = −A + B + C + D ⇒ 0 = −A + +C + ; 1 = A−C (ii)
2 2

1 1
(i) + (ii) : 2A = 1; ⇒ A = .X Then C=− X
2 2

2u2 1 1 1 1
Thus 2 2
= + 2
− + XX.
(u − 1) (u + 1) 2(u − 1) 2(u − 1) 2(u + 1) 2(u + 1)2
126 2012–2020 Past Test and Examination Questions Booklet Mathematics I (Major)

Z √
x+1
(b) Hence, or otherwise, evaluate dx.
x2
[7]
√ 2
Let u = x + 1, u = x + 1, 2u du = dx X
R √
x+1
R u·2u
x2
dx = (u2 −1)2
duX

R 2u2
= ((u−1)(u+1))2
du

R 2u2
= (u−1)2 (u+1)2
duX

R Ä 1 1 1 1
ä
= 2(u−1) + 2(u−1)2
− 2(u+1) + 2(u+1)2
du

1 1 1
ln|u − 1| − − ln|u + 1| − + CXX

= 2 u−1 u+1

Z √ Å √ √
x+1 1 1 1
ã
dx = ln| x + 1 − 1| − √ − ln| x + 1 + 1| − √ + C .XX
x2 2 x+1−1 x+1+1

Question 4 - 13 marks
Z ∞
1
Determine whether the improper integral dx converge or diverge. [4]
R∞ 1 Rt 1 x3
1 x3
dx = lim 1 x13 dxX
t→∞

= lim − 2x12 |t1 X


t→∞

= lim − 2t12 + 1

t→∞ 2

= 12 X

Z ∞
1
Thus dx converges.X
1 x3
X∞
1
Alternatively use the p series test for 3
, (p = 3).
n=1
n

Question 5 - 13 marks

X
(a) Prove that if the series an converges, then lim an = 0. [6]
n→∞
n=1
n
X
Let S n = an = a1 + a2 + a3 + · · · + an−1 + an . X
i=1

Then S n − S n−1 = an X

X
Since an converges then lim S n = S exists. X
n→∞
n=1
Mathematics I (Major) 2012–2020 Past Test and Examination Questions Booklet 127

Since n − 1 → ∞ as n → ∞ then lim S n−1 = S . X


n→∞
Hence lim an = lim (S n − S n−1 )X = lim S n − lim S n−1 = S − S = 0. X
n→∞ n→∞ n→∞ n→∞
3n
(b) Consider the sequence {an }∞
n=1 where an = .
4n − 1

X
Does the series an converge?
n=1
Justify your answer. [2]

3n 3
lim = ,0X
n→∞ 4n − 1 4
X∞
3n
Then the series does not converge. X
n=1
4n − 1
1
(c) Consider the sequence {an }∞
n=1 where an = 3
.
n +1
2

X
Does the series an converge?
n=1
Justify your answer. [4]

1 1
3
<3
Xfor all n ≥ 1.
n +1
2 n2
X∞
1
The series 3
converges X(p-series , p = 32 ) (X).
n=1 n
2

X∞
1
By comparison test, (X) the series 3
is convergent.X
n=1 n + 1
2

Question 6 - 13 marks

X 1
(a) Show that the series converges to 1. [8]
r=1
r(r + 1)
1 A B
≡ + X
r(r + 1) r r+1
r = 0; A = 1 r = −1, B = −1 X
n
P n
P
1
Sn = r(r+1) = ( 1r − 1
r+1 )X
r=1 r=1

n
P n
P
1 1 1
= r+1 +1− n+1 − r+1 X
r=1 r=1

1
=1− n+1 X

1
Å ã
S = lim S n = lim 1 − X = 1. X
n→∞ n→∞ n+1

X 1
Then the series converges to 1. X
r=1
r(r + 1)
X∞
(b) When is a series an said to be absolutely convergent? [2]
n=1
128 2012–2020 Past Test and Examination Questions Booklet Mathematics I (Major)


X ∞
X
A series an said to be absolutely convergent if |an | is convergent.
n=1 n=1

X∞
sin((2n + 1) π2 )
(c) Show that the series is absolutely convergent.
n=1
n2
Justify your answer. [4]
X∞ X∞ X∞
sin((2n + 1) π2 ) (−1)n 1
2
= 2
X = .X
n=1
n n=1
n n=1
n2
X∞
1
The series 2
is convergent (X) (p-series p=2). (X)
n=1
n
X∞
sin((2n + 1) π2 )
Therefore, the series is absolutely convergent. X
n=1
n2

Question 7 - 13 marks

X (x − 2)n
For which values of x does the series converge?
n=1
n 2n
Justify your answer. [8]
(x−2)n+1 n 2n
lim an+1 = lim (n+1) X
n→∞ an n→∞ 2n+1 (x−2)n

(x−2) n
= lim
n→∞ 2(n+1)

n
= lim |x − 2|
n→∞ 2(n+1)

= 21 |x − 2|X

By ratio test, the series is convergent when


1
|x − 2| < 1(X) ⇔ |x − 2| < 2 ⇔ −2 < x − 2 < 2 ⇔ 0 < x < 4. (X)
2

When |x − 2| = 2, then x − 2 = 2 or x − 2 = −2. X


X X∞ X∞
(x − 2)n 2n 1
(i) x − 2 = 2. Then = = . (X)
n=1
n 2n n=1
n2n
n=1
n

The harmonic series is divergent. (X)


X ∞
X ∞
X
(x − 2)n (−2)n (−1)n
(ii) x − 2 = −2. Then = = . (X)
n=1
n 2n n=1
n 2n n=1
n

bn = 1n , lim bn = 0, bn ≥ bn+1 X
n→∞

The alternating series is convergent. (X)


X (x − 2)n
The series converges for x ∈ (0, 4]. X
n=1
n 2n
Mathematics I (Major) 2012–2020 Past Test and Examination Questions Booklet 129

Question 8 - 13 marks
Find the general solution of the differential equation

dy π π
= y tan x + sec x, − <x< . [8]
dx 2 2
dy
− tan x = sec x
dx

p(x) = −tan x(X); q(x) = sec x (X)


Z
− tan x dxX = ln |cos x| X

π π
µ(x) = eln|cos x| = |cos x|X = cos x(X), cos x > 0; − < x < (X)
2 2
dy
cos x − y cos x tan x = cos x sec x
dx
dy
=⇒ cos x − y sin x = 1
dx
dy
=⇒ (y cos x) = 1
dx
Z
=⇒ (y cos x) = dx + CX = x + C X

=⇒ y = (x + C)sec x X
130 2012–2020 Past Test and Examination Questions Booklet Mathematics I (Major)

4.7 Calculus Solutions 2019


4.7.1 Calculus Solutions March (D diagonal) 2019

Question 1 [7] The sketch of the function f is given below.

f (x) = dxe

−3 −2 −1 1 2 3
−1

−2

Using the sketch or otherwise, evaluate if it exists

i) lim f (x) = 0 (1)


x→0−
ii) lim f (x) = 1 (1)
x→0+
iii) lim f (x) does not exist (1)
x→0

Let g(x) = x, then evaluate if it exists

i) lim g(x) = 0 (1)


x→0−
ii) lim g(x) = 0 (1)
x→0+
iii) lim g(x) = 0 (1)
x→0

g(x)
Let h(x) = with f (x) and g(x) as above, then evaluate if it exists
f (x)
i) lim h(x) = 0 (1)
x→0+
Mathematics I (Major) 2012–2020 Past Test and Examination Questions Booklet 131

Question 2 [8]

a) The Dirichlet function d is defined as (2)

1 if x ∈ Q, i.e., x is rational
(
d(x) =
0 if x ∈ R\Q, i.e., x is irrational.

i) d(3) = 1. (1)
ii) lim d(x) does not exist (1)
x→3

b) State whether the function


1
f (x) = e x
has asymptotes or not. If it has no asymptotes, then, write none. Otherwise, identify asymptotes as either vertical
or horizontal and provide a reason for each asymptote. Assume that lim et = 1. (6) Answer:
t→0
1
i) lim e = 1 implies that y = 1 is an horizontal asymptote.
x
x→−∞
1
ii) lim e x = 1 implies that y = 1 is an horizontal asymptote.
x→∞
1
iii) lim e x = ∞ implies that x = 0 is a vertical asymptote.
x→0+

Question 3 [6]
Find the limit

1 1
Å ã
lim √ − ,
x→0 x 1+x x
if it exists.
Hint: What is the common denominator of the difference function in the limit above?
Answer:

11
Å ã
lim √ − =∞−∞
x→0 x 1+x x

1 1 1− 1+x
Å ã
lim √ − = lim √
x→0 x 1+x x x→0 x 1 + x
Ä √ äÄ √ ä
1− 1+x 1+ 1+x
= lim √ Ä √ ä
x→0 x 1+x 1+ 1+x
−x
= lim √ Ä √ ä
x→0 x 1 + x 1 + 1+x
−1
= lim − √ Ä √ ä
x→0 1+x 1+ 1+x
1
=−
2
Therefore,
1 1 1
Å ã
lim √ − =−
x→0 x 1+x x 2
132 2012–2020 Past Test and Examination Questions Booklet Mathematics I (Major)

Question 4 [7]
Let

x2 − 1
g(x) = .
|x − 1|
i) Determine if the one sided limits of g at x = 1 exists.
ii) Does the limit of g at x = 1 exist? Give reason.
iii) What is the value of g at x = 1 if it exist? Specify if does not exist.
Answer:

i)

x2 − 1
lim− g(x) = lim−
x→1 x→1 −(x − 1)
(x − 1)(x + 1)
= lim−
x→1 −(x − 1)
= lim− −(x + 1)
x→1
= −2 ∈ R

x2 − 1
lim+ g(x) = lim+
x→1 x→1 x − 1
(x − 1)(x + 1)
= lim+
x→1 x−1
= lim+ (x + 1)
x→1
=2∈R

The one sided limits exist.


ii) lim x→1 g(x) does not exist since lim x→1− g(x) , lim x→1+ g(x).
0
iii) g(1) = 0 is undefined, thus g(1) does not exist.
Mathematics I (Major) 2012–2020 Past Test and Examination Questions Booklet 133

Question 5 [12]

a) Give the statement of the Sandwich or Squeeze or Pinched theorem.

Answer: If f (x) ≤ g(x) ≤ h(x) when x is near a, except possibly at a, and lim f (x) = L = lim h(x), then
x→a x→a
lim g(x) = L. (3)
x→a

b) Hence, prove that


sin θ
lim = 1.
θ→0 θ
You may use that sin θ < θ < tan θ for 0 < θ < π2 .

Answer: For 0 < θ < π2 ,


sin θ < θ < tan θ.
Since sin θ > 0, this implies that
θ 1
1< < .
sin θ cos θ
By inverting,
sin θ
cos θ < < 1.
θ
Since lim cos θ = 1 = lim 1, and using the the Sandwich theorem in a) above, then
θ→0 θ→0

sin θ
lim = 1.
θ→0 θ
Similarly for − π2 < θ < 0. Try it! (6)

c) Evaluate
sin θ
lim
θ→0 θ + tan θ

sin θ 0
Answer: lim =
θ→0 θ + tan θ 0
sin θ 1
lim = lim 1
θ→0 θ + tan θ θ→0 θ
+
sin θ cos θ
1
= lim
sin θ −1
θ→0
+ (cos θ)−1

θ
1
= , see b) above
1+1
1
=
2
(3)

Total : [40] marks


134 2012–2020 Past Test and Examination Questions Booklet Mathematics I (Major)

4.7.2 Calculus Solutions April (E diagonal) 2019

Question 1 [15]
dy
Find the derivatives dx of the following functions:

1) y = sec2 (πx) (3)


dy d
= 2 sec(πx) sec(πx)
dx dx
d
= 2 sec(πx) sec(πx) tan(πx) (πx)
dx
= 2π sec2 (πx) tan(πx)

2) y = sin(cos(2x − 5)) (3)


dy d
= cos(cos(2x − 5)) cos(2x − 5)
dx dx
d
= cos(cos(2x − 5))[− sin(2x − 5)] (2x − 5)
dx
= −2 cos(cos(2x − 5)) sin(2x − 5)

3) y = x2 cot x (3)

dy dx2 d
= cot x + x2 cot x
dx dx dx
= 2x cot x − x2 cosec2 x
x −10
4) y= −1 (3)
2
dy x −11 d  x 
= −10 −1 −1
dx 2 dx 2
x −11
= −5 −1
2

x3 − 1
5) y= (3)
x3 + 1
dy (x3 − 1)0 (x3 + 1) − (x3 − 1)(x3 + 1)0
=
dx (x3 + 1)2
3x2 (x3 + 1) − (x3 − 1)3x2
=
(x3 + 1)2
6x2
= 3
(x + 1)2
Mathematics I (Major) 2012–2020 Past Test and Examination Questions Booklet 135

Question 2 [5]

Consider the greatest integer function f (x) = b x c .


1) Discuss the continuity and differentiability of f at the integers x = n.
2) Sketch the graph of the function f and its derivative f 0 .
Answer:
1) f (n) = b n c = n, for any integer n .
lim− f (x) = lim− b x c = n − 1 , n = f (n) .
x→n x→n
Thus, f (x) = b x c is not left-continuous at x = n .
Hence, f (x) = b x c is not continuous at x = n .
By contrapositive, f (x) = b x c is not differentiable at x = n .

2) Graphs of f and f 0 :
f (x) = b x c

−3 −2 −1 1 2 3 x

−2

−3

f 0 (x)

−3 −2 −1 1 2 3 x
−1

−2

−3
136 2012–2020 Past Test and Examination Questions Booklet Mathematics I (Major)

Question 3 [5]

Find the equations of the tangent line and the normal line to the graph of the function

f (x) = x

at x = 1.

Answer: x = 1 =⇒ y = f (1) = 1 .

For the tangent line : y − f (1) = mT (x − 1).

1 1
f 0 (x) = √ =⇒ mT = f 0 (1) = .
2 x 2
Thus, the equation of the tanget line at x = 1 is
1 1
y= x+ .
2 2
For the normal line: y − f (1) = mN (x − 1), where mN = − m1T = −2. Thus, the equation of the normal line at x = 1
is
y = −2x + 3 .

Question 4 [5]

Find the value(s) of a and b which make the function

4x if x ≤ −1,



f (x) = ax + b if −1 < x ≤ 2 ,
if x > 2,

−5x

continuous at x = −1 but not continuous at x = 2.

Answer:

(1) f is continuous at x = −1 :

f (−1) = 4(−1) = −4, lim f (x) = lim − 4x = −4 and lim + f (x) = lim + ax + b = b − a. Thus,
x→−1− x→−1 x→−1 x→−1

b − a = −4

(2) f is not continuous at x = 2 :

f (2) = 2a + b, lim f (x) = lim− ax + b = 2a + b and lim+ f (x) = lim+ −5x = −10. Thus,
x→2− x→2 x→2 x→2

2a + b , −10

(3) Hence, from (1) and (2) above, f is continuous at x = −1 but not continuous at x = 2 if and only if b = a − 4
and a , −2. Alternatively, a = b + 4 and b , −6.
Mathematics I (Major) 2012–2020 Past Test and Examination Questions Booklet 137

Question 5 [5]

d
Using the limit definition, prove that dx sin x = cos x.

Hint: Use the following formulas


1) sin(A + B) = sin A cos B + sin B cos A,
sin x 1 − cos x
2) lim = 1, and lim =0.
x→0 x x→0 x
Answer:
d sin(x + h) − sin x
sin x = lim
dx h→0 h
sin x cos h + sin h cos x − sin x
= lim , by formula 1)
h→0 h
sin x(cos h − 1) sin h
ï ò
= lim + cos x
h→0 h h
cos h − 1 sin h
Å ã Å ã
= (sin x) lim + (cos x) lim
h→0 h h→0 h

= (sin x)(0) + (cos x)(1), by formulas 2)


= cos x

Question 6 [5]

Find the equation of any horizontal tangent line to the curveof the function

y = 1 + sin(x + sin x) .

Answer:

The tangent line is horizontal to the function y = f (x) if y0 = f 0 (x) = 0. That is,

cos(x + sin x).(x + sin x)0 = 0 ,

cos(x + sin x).(1 + cos x) = 0 ,


cos(x + sin x) = 0 or 1 + cos x = 0 .
Take 1 + cos x = 0, i.e. cos x = −1. Then, x = (2k + 1)π for all k ∈ Z and

y = f ((2k + 1)π)
= 1 + sin [(2k + 1)π + sin((2k + 1)π)]
= 1 + sin [(2k + 1)π + 0]
= 1 + sin[(2k + 1)π]
=1+0=1

Hence, the equation of the horizontal tangent line at x = (2k + 1)π, for every k ∈ Z, is y = 1.

Total: [40] marks


138 2012–2020 Past Test and Examination Questions Booklet Mathematics I (Major)

4.7.3 Calculus Solutions June 2019

Question 1 [5 marks]
Write down whether you think the statement is true (T) or false (F):

(a) The slope of the tangent line to the function f (x) = e x at x = 0 is 1.


Z b Z a
(b) f (t) dt = − f (s) ds .
a b
(c) If f is differentiable at c and f 0 (c) = 0 then there is a max or a min at x = c .
(d) If f is differentiable at c and there is a max or a min at x = c, then f 0 (c) = 0 .
(e) If c ∈ dom( f ) is a critical number of the function f then f 0 (c) = 0 .

Answers: (a) T (b) T (c) F (d) T (e) F .

Question 2 [5 marks]
Prove that
d 1
ln x = .
dx x
d x x
You may assume that dx e = e . Hint: If y = ln x, then e = x .
y

Answer: Let y = ln x .

=⇒ ey = x
d y d
=⇒ e = x
dx dx
=⇒ ey y0 = 1
1
=⇒ y0 = y
e
0 1
=⇒ y =
x
Therefore,
d 1
ln x = .
dx x
Question 3 [5 marks]
(a) Write down the first four derivatives of f (x) = cos x when x = 0 .
Answer:

f (x) = cos x =⇒ f (0) = 1


f (x) = − sin x =⇒ f 0 (0) = 0
0

f 00 (x) = − cos x =⇒ f 00 (0) = −1


f 000 (x) = sin x =⇒ f 000 (0) = 0
f (4) (x) = cos x =⇒ f (4) (0) = 1

(b) Hence or otherwise write down the fourth degree Taylor polynomial about x = 0 .
Hint: The nth -degree Taylor polynomial of f about a is

f 00 (a)(x − a)2 f 000 (a)(x − a)3 f n (a)(x − a)n


T n (x) = f (a) + f 0 (a)(x − a) + + + ··· + +
2! 3! n!
Answer:
x2 x4
T 4 (x) = 1 − +
2 4!
Mathematics I (Major) 2012–2020 Past Test and Examination Questions Booklet 139

Question 4 [4 marks]
If f (x) = x x , what is f 0 (x)?

Answer: Let y = x x . Take logs of both sides: ln y = ln x x . Hence by log laws, ln y = x ln x .


Now differentiate implicitly using the product and chain rules:
1 0 1
y = 1 ln x + x = ln x + 1 .
y x
Making y0 the subject of the formula and substituting for y gives
d x
x = y0 = x x (ln x + 1) .
dx

Question 5 [5 marks]
1
lim x x =?
x→∞

1
Answer: Let y = lim x x . Then
x→∞

1
ln y = ln lim x x
x→∞
1
= lim ln x x
x→∞
ln x
= lim
x→∞ x
1
x
= lim , by l’Hospital rule
x→∞1
1
= lim
x→∞ x
= 0.
1
Thus ln y = 0 so that lim x x = e0 = 1 .
x→∞

Question 6 [4 marks]
xy
Let z = 5 . Determine z xy .

Answer:
z x = 5 xy (ln 5)(y)
Therefore

z xy = 5 xy (ln 5)(x)(ln 5)(y) + 5 xy (ln 5)


= 5 xy (ln 5)[xy(ln 5) + 1].

Question 7 [3 marks]
x2 −4x+3
Given the function f (x) = x−2 in the interval [1, 3], discuss the validity of the MVT.

Answer:
Since 2 ∈ [1, 3] but 2 is not in the domain of f , f is not defined at 2, and hence not continuous at 2. Therefore the
conditions of the MVT are not satisfied, and the MVT is thus not applicable.
140 2012–2020 Past Test and Examination Questions Booklet Mathematics I (Major)

Question 8 [9 marks]
Evaluate the following integrals.
Z 1
(a) x2 sin x3 − 5 dx

0

Answer: By substitution. Let u = x3 − 5, then du 2 1 2


dx = 3x so 3 du = x dx .
Also, x = 0 =⇒ u = −5 and x = 1 =⇒ u = −4 .
Thus,
Z 1 Z −4
2 3 1
x sin x − 5 dx = sin u du

0 −5 3
1
= [(− cos u)]−4 −5
3
1
= [− cos(−4) − (− cos(−5))]
3
1
= (cos 5 − cos 4)
3
Z
(b) x sin x dx

Answer: Let u = x and dv = sin x dx. Then du = dx and v = − cos x . Hence, by IBP,
Z Z
x sin x dx = −x cos x − (− cos x) dx

= −x cos x + sin x + C
Z
1
(c) √
1 − 9t2
Answer: This can be done by substitution with u = 3t .
Z Z
1 1
√ dt = dt
1 − (3t)2
p
1 − 9t2
1
= arcsin(3t) + C
3

Total: [40] marks


Mathematics I (Major) 2012–2020 Past Test and Examination Questions Booklet 141

4.7.4 Calculus Solutions August (D diagonal) 2019

Question 1 [10 marks]


Let f , g and h be continuous functions on R with f = g + h, g an even function and h an odd function.
Indicate whether each statement is True (T) or False (F).
Z b Z b
(a) f (x) dx = | f (x)| dx
a a

Z b Z a
(b) f (x) dx = − f (x) dx
a b
Z a Z a
(c) g(x) dx = 2 g(x) dx
−a 0
Z a Z a
(d) f (x) dx = 2 g(x) dx
−a 0
Z 1
(e) g(x)h(x) dx = 0
−1

Answers: (a) F (b) T (c) T (d) T (e) T .

Question 2 [4 marks]
Prove that if the functions f and g are continuous with f (x) ≥ g(x) on the interval [a, b], then

Z b Z b
f (x) dx ≥ g(x) dx .
a a

(You may use any of the properties of integrals except this one.)

Answer: Let f (x) ≥ g(x), for all x ∈ [a, b] .

=⇒ f (x) ≥ g(x) , for all x ∈ [a, b]


=⇒ f (x) − g(x) ≥ 0 , for all x ∈ [a, b]
Z b
=⇒ ( f (x) − g(x)) dx ≥ 0
Za b Z b
=⇒ f (x) dx − g(x) dx ≥ 0
Za b Za b
=⇒ f (x) dx ≥ g(x) dx .
a a

Question 3 [7 marks]
(a) Show that the “right Riemann sum” (i.e. using the right end-points of the subintervals) with n rectangles of
equal width for approximating the area of the region bounded by the graphs y = −x2 + 1 and y = x2 − 1 is given by

n Å
16 X 12
ã
i − i .
n2 i=1 n

Answer:
2
1 − x2 = x2 − 1 =⇒ x = ±1. So partition [−1, 1] into n subintervals of width ∆x = n .
142 2012–2020 Past Test and Examination Questions Booklet Mathematics I (Major)

For right end-points, xi = −1 + n2 i, for i = 1, 2, . . . , n. So, the right Riemann sum is


n n ñ
X 2X 2 2
ãô
2 2
Å
2 − 2xi 2 − 2 −1 + i
 
=
i=1
n n i=1 n
n
2X 8 8
Å ã
= i − 2 i2
n i=1 n n
n Å
16 X 12
ã
= 2 i− i .
n i=1 n

(b) Determine
n Å
16 X 12
ã
lim i − i
n→∞ n2 n
i=1

without calculating the limit. (Hint: Evaluate a suitable integral)

Answer:
n Å ã Z 1
16 X 12
lim i − i 2 − 2x2 dx

2
=
n→∞ n
i=1
n −1
ò1
2
ï
= 2x − x3
3 −1
8
=
3

Question 4 [6 marks]
The region bounded by the parabola y = x2 and the line y = x in the first quadrant is revolved about the line x = 1.
Calculate the volume of the solid generated.

Answer: The graphs intersect at (0, 0) and (1, 1). Check it! Thus, the volume is
Z 1
√ 2 ó
(1 − y)2 − 1 −
î
V=π y dy
0
Z 1
√ 
=π y2 − 3y + 2 y dy
0
ò1
1 3 4
ï
= π y3 − y2 + y3/2
3 2 3 0
π
=
6

Question 5 [3 marks]
R ln x
0 −2
Let F(x) = 0
2 cosh t dt. Show that F (e) = 1 + e .

Answer: F 0 (x) = 2 cosh (ln x) . 1x implies

1 e + e−1
F 0 (e) = 2 cosh(1). = = 1 + e−2
e e

Total: [40] marks


Mathematics I (Major) 2012–2020 Past Test and Examination Questions Booklet 143

4.7.5 Calculus Solutions November 2019

Section A: Multiple choice questions

Instructions:
Section A, Questions 1–6 are multiple choice questions. There is only one correct answer in each case. Circle the
correct answer of your choice.
Question 1
Which of the following substitutions is most appropriate to evaluate
Z
1
√ dx ?
x x2 − 1
4


A. x= u
B. x = sin u
C. x = tan u
n
D. x = sec u
E. x = arctan u (2)

Question 2
A partial fraction decomposition of
x3 + 3x2 + 4
x(x2 + 1)2 (x − 1)
is of the form
A B 4 2
A. + + +
x2 + 1 (x2 + 1)2 x x − 1

B.m 2
Ax + B Cx + D 4 2
+ 2 − +
x +1 (x + 1)2 x x − 1
Ax + B 4 2
C. + −
(x2 + 1)2 x x − 1
Ax + B 4 2
D. 2 2
− −
(x + 1) x x−1
Ax + B Cx + D 4 2
E. 2
+ 2 2
+ + (3)
x +1 (x + 1) x x−1

Question 3
Which of the following statements is incorrect?

A. If c , 0 and (an ) converges, then (can ) converges.


B. If c , 0 and (can ) converges, then (an ) converges.
C.m If (an bn ) converges, then (an ) and (bn ) converge.
D. If (an ) and (bn ) converge, then (an bn ) converges.
E. If (an ) and (bn ) converge, then (an + bn ) converges. (2)
144 2012–2020 Past Test and Examination Questions Booklet Mathematics I (Major)

Question 4
The series
X∞
5
(−1)n n+1
n=0
3

A.n is a convergent geometric series with



X 5 5 1
(−1)n =
n=0
3n+1 3 1 + 1
3
X∞
5 5 1
B. is a convergent geometric series with (−1)n =
n=0
3n+1 3 1− 1
3
X∞
5 1
C. is a convergent geometric series with (−1)n =5
n=0
3n+1 1− 1
3
D. converges but does not converge absolutely
E. diverges (3)

Question 5
1 1
Using the geometric series = 1 + x + x2 + x3 + . . . , it follows that has the power series expansion
1−x 1 + x3
A.n 1 − x3 + x6 − x9 + . . .
B. 1 − x3 + x9 − x27 + . . .
C. 1 + x3 + x9 + x27 + . . .
D. 1 + x3 + x6 + x9 + . . .
E. 1 − x3 − x6 − x9 + . . . (2)

Question 6
Which of the following is a linear differential equation?
A. y0 = xy2 + 2x
B. y0 = xyy0 + 2x
R
C. y0 + xy = y(x) dx + x3
D.n y0 + xy = x3 + x


E. y0 + xy = y3 + y (2)
Mathematics I (Major) 2012–2020 Past Test and Examination Questions Booklet 145

Section B

Question 1 [20]
Let
x2 − 3
f (x) = .
x+2
You may use that
x2 + 4x + 3 2
f 0 (x) = and f 00 (x) = .
(x + 2)2 (x + 2)3

(a) Find the domain, the asymptotes (if any) of f and classify each asymptote as vertical, horizontal or slant.
(5)

Answer: The domain is: dom ( f ) = {x ∈ R : x , −2}


For x → −2 : x2 − 3 → 1 and x + 2 → 0.
Hence x = −2 is a vertical asymptote.
Long division gives

x2 + 2x − 2x − 4 + 1
f (x) =
x+2
1
= x−2+ .
x+2
Then
1
=0
lim
x+2
x→±∞

shows that y = x − 2 is an oblique (slant) asymptote to f .

(b) Find the intercepts on the x- and y-axes (if any) and discuss symmetry. (2)

3
Answer: y-intercept: f (0) = − .
2 √
x-intercepts: x2 − 3 = 0 ⇔ x = ± 3
no symmetry [accepted without reasoning; one may argue that the asymptotes are not symmetric or e. g. that
f (−1) , ± f (1) ]

(c) Find the maxima and minima of f (if any). (5)

Answer: The critical points are given by f 0 (x) = 0:

x2 + 4x + 3 = 0 ⇔ (x + 3)(x + 1) = 0
⇔ x = −3 or x = −1

2
Since f 00 (−3) = < 0, f has a maximum at x = −3
(−1)3
2
146 2012–2020 Past Test and Examination Questions Booklet Mathematics I (Major)

(d) Discuss the concavity of f . (2)

Answer: f 00 (x) < 0 for x ∈ (−∞, −2)


f 00 (x) > 0 for x ∈ (−2, ∞)
Therefore
f is concave down on (−∞, −2)
f is concave up on (−2, ∞)

(e) Find the points of inflection (if any). (1)

Answer: Since 2 < dom( f ), there are no points of inflection.

(f) Hence sketch the graph of f indicating intercepts, maxima and minima and points of inflection (if any) .
(5)
Mathematics I (Major) 2012–2020 Past Test and Examination Questions Booklet 147

−8 −6 −4 −2 2 4 6

−2

−4

−6

−8

−10

−12

−14
148 2012–2020 Past Test and Examination Questions Booklet Mathematics I (Major)

Question 2 [12]
Evaluate
Z 2
3
(a) 2
dx (6)
0 x − 6x + 12

Answer:
Z 2 Z 2
3 3
dx = dx X
0 x2 − 6x + 12 0 (x − 3)2 + 3
3 x−3 2
= √ arctan √ XX
3 3 0
√ Å Å
1
ã Ä √ äã
= 3 arctan − √ − arctan − 3 X
3
√  π π 
= 3 − + X(X)
6 3

3 π
Å ã
= π = √ (X)
6 2 3

Z
x2
(b) √ dx (6)
4 − x2

Answer:
Substitute x = 2 sin θ. Then dx = 2 cos θ dθ. X
Therefore
Z Z
x2 4 sin2 θ
√ dx = 2 cos θ dθ X
4 − (2 sin θ)2
p
4 − x2
Z
= 4 sin2 θ dθ (X)
Z
= 2 (1 − cos(2θ)) dθ X
1
Å ã
= 2 θ − sin(2θ) + C X
2
= 2θ − sin θ cos θ + C X
 x 1 √
= 2 arcsin − x 4 − x2 + C X
2 2
Mathematics I (Major) 2012–2020 Past Test and Examination Questions Booklet 149

Question 3 [8]
9 − x2
(a) Write 3 in terms of its partial fractions. (6)
x + 9x

Answer:
The formal partial fractions decoposition is

9 − x2 A Bx + C
= + 2 . X(X)
x3 + 9x x x +9
Multiplying through by x and substituting x = 0 gives

1=A X

Multiplying through by x and letting x → ∞ gives

−1 = A + B = 1 + B ⇒ B = −2 X

Substituting x = 1 gives
8 B+C −2 + C
= A+ =1+ ⇒C=0 X(X)
10 10 10
Hence
9 − x2 1 2x
3
= − 2 . X(X)
x + 9x x x + 9

Z
9 − x2
(b) Hence evaluate dx. (2)
x3 + 9x

Answer: By part (a):


Z Z Z
9 − x2 1 2x
dx = dx − dx = ln |x| − ln(x2 + 9) + C
x3 + 9x x x2 + 9
150 2012–2020 Past Test and Examination Questions Booklet Mathematics I (Major)

Question 4 [12]
(a) Explain when an integral is called improper. (2)

Z b
Answer: The integral f (x) dx is called improper if
a
1. one or both of the limits of integration are infinite or
2. f has a finite number of infinite discontinuities in the interval [a, b].

(b) Determine whether the improper integral


Z ∞
ln x
dx
1 (x + 1)2
converges or diverges. If it converges, find its value. (10)

Answer: First find (for x > 0)


Z Z
ln x ln x 1
dx = − + dx XX
(x + 1)2 x+1 x(x + 1)
Partial fractions
1 A B
= +
x(x + 1) x x+1
gives
x = 0: 1 = A, x = −1: −1 = B X
so that
Z Z
ln x ln x 1
dx = − + dx
(x + 1)2 x+1 x(x + 1)
ln x
=− + ln x − ln(x + 1) + C X
x+1

Z ∞
ln x
The integral dx is improper at ∞. X
1 (x + 1)2
Mathematics I (Major) 2012–2020 Past Test and Examination Questions Booklet 151

So consider
Z ∞ òt
ln x ln x
ï
dx = lim − + ln x − ln(x + 1) X
1 (x + 1)2 t→0+ x+1 1
ln t
ï ò
= lim ln 2 − + ln t − ln(t + 1) X
t→∞ t+1
ln t t+1
Å ã
= ln 2 − lim + lim ln X
t→∞ t + 1 t→∞ t
1
1
Å ã
= ln 2 − lim t − lim ln 1 + = ln 2 X
t→∞ 1 t→∞ t
Therefore the indefinite integral converges and
Z ∞
ln x
dx = ln 2 X
1 (x + 1)2

Question 5 [7]

X
Prove that if the series an converges, then lim an = 0.
n→∞
n=1

Answer: Let
sn = a1 + a2 + . . . an . X(X)
Then
an = sn − sn−1 . X(X)

X
Since an converges,
n=1
lim sn = s
n→∞

exists. X Since also n − 1 → ∞ as n → ∞,

lim sn−1 = s. X
n→∞

Hence
lim an = lim (sn − sn−1 ) = lim sn − lim sn−1 = s − s = 0. XX
n→∞ n→∞ n→∞ n→∞
152 2012–2020 Past Test and Examination Questions Booklet Mathematics I (Major)

Question 6 [10]
(a) Determine, giving reasons, whether the sequence

5 − n + 3n2
ß ™
{an } =
7n2 − 19
is convergent or divergent. If it is convergent, state to what value it converges. (4)

Answer:
5 − n + 3n2
lim an = lim X
n→∞ n→∞ 7n2 − 19
5 1
2 − n + 3
= lim n X
n→∞ 7 − 19
n2
0−0+3
=
7+0
3
= X
7
Hence the sequence converges, and its limit is 37 . X
(b) Decide (with reasons) if the following series converge or diverge. (6)
X∞
5 − n + 3n2
(i) (−1)n
n=1
7n2 − 19

Answer: From part (a) we have


X∞ ∞
5 − n + 3n2 X
(−1)n = an
n=1
7n2 − 19 n=1

with
3
lim an =
, 0. X
n→∞ 7
Hence the series does not converge (see Question 5). X

∞ Å
X ãn
5 − n + 3n2
(ii)
n=1
7n2 − 19

Answer:
ãn
5 − n + 3n2 5 − n + 3n2
Å
= X
7n2 − 19 7n2 − 19
3
→ by part (a) X
7
3
Hence the series converges by the root test since 7 < 1. XX
Mathematics I (Major) 2012–2020 Past Test and Examination Questions Booklet 153

Question 7 [7]
(a) Find the general explicit solution of the differential equation y0 = (x2 + 3)(y2 + 1). (5)

Answer:
The differential equation is separable: X
dy
= (x2 + 3)dx X
y2 + 1
The general solution therefore is
1 2
arctan y = x +C XX
3
Then the general explicit solution is

1 2
Å ã
y = tan x +C X
3

(b) Find the particular solution of the differential equation in (a) which satifies the initial condition y(0) = 1. (2)

Answer: Substituting x = 0 and y = 1 above gives

1 = tan C X
π
Choosing C = gives the particular solution
4
1 2 π
Å ã
y = tan x + X
3 4
Note: Any other solution of 1 = tan C is also correct. Due to the periodicity of tan it would give the same solution.

Total: [90] marks


154 2012–2020 Past Test and Examination Questions Booklet Mathematics I (Major)

4.8 Calculus Solutions 2012


4.8.1 Calculus Solutions March 2012

Section B

Question 1
(a)
√ √ Ä√ √ äÄ√ √ ä
x2 + 4 − x + 4 x2 + 4 − x + 4 x2 + 4 + x + 4
lim = lim Ä√ √
2x2 − 2
ä
x→1 x→1 2 x2 − 1 x2 + 4 + x + 4
x2 + 4 − (x + 4)
= lim Ä√ √ ä
x→1 2 (x − 1) (x + 1) x2 + 4 + x + 4
x (x − 1)
= lim Ä√ √ ä
x→1 2 (x − 1) (x + 1) x2 + 4 + x + 4
x 1
= lim Ä√ √ ä= Ä√ √ ä
x→1 2 (x + 1) x2 +4+ x+4 2(1 + 1) 5+ 5
1
= √
8 5

(b)

4

√ |x| 9 − 2
9x2 − 4 x
lim = lim
3 − 2x 3
Å ã
x→−∞ x→−∞
x −2
x
4

(−x) 9 − 2
x
= lim
3
Å ã
x→−∞
x −2
x
4

− 9− 2
x
= lim
x→−∞ 3
−2
x
−3 3
= =
−2 2

(c)

x2 − 4 −3 √
lim + f (x) = lim + √ = √ =− 3
x→−1 x→−1 4 − x2 3
2
x −4
lim f (x) = lim −
x→−1− x→−1 x − 2
(x − 2)(x + 2)
= lim − = lim − (x + 2)
x→−1 x−2 x→−1
=1

Since lim + f (x) , lim − f (x), lim f (x) does not exist.
x→−1 x→−1 x→−1
Mathematics I (Major) 2012–2020 Past Test and Examination Questions Booklet 155

Question 2
(b)
1
− 1 ≤ sin ≤1
x2
1
⇒ − x2 ≤ x2 sin 2 ≤ x2
x
But

lim(−x2 ) = lim x2 = 0.
x→0 x→0

Thus by the Sandwich Theorem,


1
lim x2 sin = 0.
x→0 x2
Alternatively:

1
0 ≤ sin ≤1
x2
1
⇒0 ≤ x2 sin 2 ≤ x2
x
But

lim 0 = lim x2 = 0.
x→0 x→0

Thus by the Sandwich Theorem,

1 1
lim x2 sin 2
= 0 ⇒ lim x2 sin 2 = 0.
x→0 x x→0 x

Question 3
(b)

x3 − 4x2 + 4x x(x2 − 4x + 4)
g(x) = =
x2 − x x(x − 1)
Hence the rational function g is defined for all real numbers except 0 and 1, and these are the only points of
discontinuity.
For all x in the domain of g:
(x − 2)2
g(x) =
x−1
(x − 2)2 4
lim g(x) = lim = = −4
x→0 x→0 x − 1 −1
Thus g has a removable discontinuity at x = 0.

(x − 2)2
lim− g(x) = lim− = −∞
x→1 x→1 x−1
Thus g has an infinite discontinuity at x = 1.
156 2012–2020 Past Test and Examination Questions Booklet Mathematics I (Major)

4.8.2 Calculus Solutions May 2012

Section B

Question 2
(a)
2
x2
g0 (x) = ecos (−2 cos x2 sin x2 )(2x)

2
x2
= −4x cos x2 sin x2 ecos
î ó

(b)
1
Å ã
2 x 3 x
(3x − 2 ln 2)(1 + ln x) − (x − 2 )
x
f 0 (x) =
(1 + ln x)2

Question 3
(a) Implicit differentiation gives

y + xy0 + cos yy0 = e x


⇒ (x + cos y)y0 = e x − y
ex − y
⇒ y0 =
x + cos y

(b) By (a), substituting x = 0 and y = π, the slope of the tangent to the curve is

e0 − π 1−π
mt = = =π−1
0 + cos π −1

Then the slope of the normal is


1 1 1
mn = − =− =
mt π−1 1−π
so that the equation of the normal line is

1
y = π + mn (x − 0) = π + x
1−π

Question 4
(a) Substitute u = cos(5x), and therefore
du = −5 sin(5x) dx
Then
Z Z
3 1
sin(5x) cos (5x) dx = − u3 du
5
1
= − u4 + C
20
1
= − cos4 (5x) + C
20

(b) Integration by parts with u = x and dv = e x dx.


Mathematics I (Major) 2012–2020 Past Test and Examination Questions Booklet 157

Then du = dx and v = e x , and


Z 4 Z 4
x
xe dx = u, dv
1 1
Z 4
4
= uv − v, du
1 1
Z 4
4
= xe x − e x dx
1 1
4
= 4e − e − e x
4
1
= 4e4 − e − (e4 − e)
= 3e4

4.8.3 Calculus Solutions June 2012

Section B

In this section you are expected to show all your working to earn the marks allocated.
Question 1
We know that
f 0 (x) = 2 x ln 2
Hence
f (n) (x) = 2 x (ln 2)n

Question 2
(a) The domain of f is R.
(b)
√ 2x
(1) 1 + x2 − x √
1 2 1 + x2 1
f 0 (x) = 2

x2 1+x 1 + x2
1−
1 + x2

1 + x2 1 + x2 − x2 1
= √ √ −
2 2 2
1 + x − x (1 + x ) 1 + x 2 1 + x2

1 1
= −
1 + x2 1 + x2

=0

(c) f is constant
(d) From (c) and
f (0) = 1 + 0 − arctan 0 = 1
we conclude that f (x) = 1 for all x in the domain of f .
158 2012–2020 Past Test and Examination Questions Booklet Mathematics I (Major)

Question 3
x 1
(a) Let u = . Then du = dx.
a a
Then
Z Z
dx a
= du
a + x2
2 a2
+ a2 u2
Z
a 1
= 2 du
a 1 + u2
1
= arctan u + C
a
1 x
= arctan + C
a a

(b)
Z 3
dx 1 x 3
√ = √ arctan √ √
3 3 + x2 3 3 3
1 3
Å ã
= √ arctan √ − arctan 1
3 3
1 π π
= √ −
3 3 4
π
= √
12 3

Question 4
Assume y = limπ − (tan x)cos x exists. Then
x→ 2

ln y = ln limπ − (tan x)cos x


x→ 2

= limπ − ln (tan x)cos x


 
x→ 2

= limπ − cos x ln(tan x) (”0 · ∞”)


x→ 2

ln(tan x) 0
Å ã
= limπ −
x→ 2 1 0
cos x
sec2 x
= limπ − tan x
x→ 2 1
− 2 (− sin x)
cos x
1
= limπ −
x→ 2 sin x tan x
cos x
= limπ −
x→ 2 sin2 x

=0

Therefore limπ − (tan x)cos x exists and equals


x→ 2
y = eln y = e0 = 1
Mathematics I (Major) 2012–2020 Past Test and Examination Questions Booklet 159

4.8.4 Calculus Solutions August 2012

Section B

In this section you must answer each question as fully as possible to earn full marks.
Question 1
The area is Z e
(1 + ln x)3
A= dx
1 x
dx
Using the substitution u = 1 + ln x we get du = Then x = 1 ⇒ u = 1 and x = e ⇒ u = 2 give
x
Z 2
u4 2 16 1 15
A= u3 du = = − =
1 4 1 4 4 4

Alternatively, calculate Z Z
(1 + ln x)3 u4 1
dx = u3 du = + C = (1 ln x)4 + C
x 4 4
Then
1 e (1 + ln e)4 (1 + ln 1)4 16 1 15
A= (1 + ln x)4 = − = − =
4 1 4 4 4 4 4
Question 2 From the sketch it is seen that the region lies between y = 0 and y = 23 = 8, and using the disc
1
method, R = 2 − y 3 , so that the volume is
Z 8
1
V=π (2 − y 3 )2 dy
0
Z 8
1 2
=π (4 − 4y 3 + y 3 ) dy
0
4 3 5
= π[4y − 3y 3 + y 3 ]80
5
3
= π[32 − 3 · 16 + · 32]
5
96
= π[ − 16]
5
16π
=
5

Question 4

Z 4 Z
x dx 1 4 2x − 4 + 4
= dx
0 x2 − 4x + 8 2 0 x2 − 4x + 8
Z 4 Z 4
1 2x − 4 dx
= dx + 2
2 0 x2 − 4x + 8 0 x 2 − 4x + 8
Z Z 4
1 4 2x − 4 dx
= dx + 2
2 0 x2 − 4x + 8 0 x − 2) 2+4

1 4 2 x−2 4
= ln(x2 − 4x + 8) + arctan
2 0 2 2 0
1
= (ln 8 − ln 8) + arctan 1 − arctan(−1)
2
π
=
2
160 2012–2020 Past Test and Examination Questions Booklet Mathematics I (Major)

4.9 Calculus Solutions 2020


4.9.1 Calculus Solutions March (D diagonal) 2020

Question 1 [6 marks]

The graph of the function y = f (x) is given below.

−1 1 2 3 x
−1

−2

Using this graph of f (x), which of the following statements are true, and which are false?
1) lim f (x) exists. TRUE X (1)
x→0
2) lim f (x) = 0. TRUE X (1)
x→0
3) lim f (x) = 1. FALSE X (1)
x→0
4) lim f (x) = 1. FALSE X (1)
x→1
5) lim f (x) = 0. FALSE X (1)
x→1
6) lim f (x) does not exist. TRUE X (1)
x→1
Mathematics I (Major) 2012–2020 Past Test and Examination Questions Booklet 161

Question 2 [6 marks]

3−x if x < 2,
®
Let f (x) = x
2 +1 if x > 2.

f (x)
x
y=3−x y= 2 +1

0 x
1 2 3 4

1) Find lim+ f (x) and lim− f (x). (2)


x→2 x→2
2) Does lim f (x) exist? If so, what is it? If not, why not? (2)
x→2
3) Find lim− f (x) and lim+ f (x). (1)
x→4 x→4
4) Does lim f (x) exist? If so, what is it? If not, why not? (1)
x→4

Answer 2
1) lim f (x) = 2 and lim− f (x) = 1. X X
x→2+ x→2
2) lim f (x) does not exist since lim+ f (x) , lim− f (x). X X
x→2 x→2 x→2
3) lim f (x) = 3 and lim+ f (x) = 3. (X) (X)
x→4− x→4
4) lim f (x) = 3. X
x→4
162 2012–2020 Past Test and Examination Questions Booklet Mathematics I (Major)

Question 3 [11 marks]

Use limits to determine the equations for all vertical and horizontal asymptotes.
x+1
1) f (x) = (5)
1−x

x2 + 1
2) g(x) = (6)
x
Answer 3
x+1 2 x+1 2
1) lim− = + = ∞ or lim+ = − = −∞ . X
x→1 1 − x 0 x→1 1 − x 0
Thus, the line x = 1 is a vertical asymptote. X
x+1 x 1 1
x+1 x x + x 1+ x 1+0
lim = lim 1−x
= lim = lim = = −1. XX
x→±∞ 1 − x x→±∞ x→±∞ 1 − x x→±∞ 1 −1 0−1
x x x x

Thus, the line y = −1 is a horizontal asymptote. X


√ √
x2 + 1 1 x2 + 1 1
2) lim− = − = −∞ or lim+ = + = ∞. X
x→0 x 0 x→0 x 0
Thus, the line x = 0 is a vertical asymptote. X
√ √ »
x2
»
1
x2 + 1
x2 +1
x x2
+ x12 1+ x2 √ √
lim = lim x = lim x = lim = 1+0 = 1 X, where x2 = |x| = x,
x→∞ x x→∞
x
x→∞
x
x→∞ 1
since x > 0.

Thus, the line y = 1 is a horizontal asymptote. X


√ √
x2 +1
»
x2 1
»
1
x2 + 1 x
− x2
+ x2
− 1+ x2 √
lim = lim x = lim x = lim = −1 X, where x2 = |x| = −x,
x→−∞ x x→−∞
x
x→−∞
x
x→−∞ 1
since x < 0.

Thus, the line y = −1 is a horizontal asymptote. X


√ √ »
1
»
1
»
1
»
1
x2 + 1 x2 1+ x2
|x| 1+ x2
±x 1+ x2
±1 1+ x2
Or, lim = lim = lim = lim = lim =
x→±∞ x x→±∞ x x→±∞ x x→±∞ x x→±∞ 1
lim ±1 = ±1.
x→±∞
Mathematics I (Major) 2012–2020 Past Test and Examination Questions Booklet 163

Question 4 [9 marks]

Evaluate the limit, if it exists.


1 1
Å ã
1) lim √ − (5)
x→0 x 1 + x x
e x − e−x
2) lim (4)
x→−∞ e x + e−x

Answer 4

1)

1 1 1− 1+x
Å ã
lim √ − = lim √ X
x→0 x 1+x x x→0 x 1 + x
Ä √ ä Ä √ ä
1− 1+x 1+ 1+x
= lim Ä √ äÄ √ ä X
x→0 x 1+x 1+ 1+x
1 − (1 + x)
= lim Ä √ äÄ √ ä (X)
x→0 x 1 + x 1+ 1+x
−x
= lim Ä √ äÄ √ ä (X)
x→0 x 1 + x 1+ 1+x
−1
= lim Ä √ äÄ √ ä, since x , 0 X
x→0 1+x 1+ 1+x
−1 1
= Ä√ √ ä =− X
2
äÄ
1+0 1+ 1+0

2)

e x − e−x e−x e2x − 1



lim = lim −x 2x XX
x→−∞ e x + e−x x→−∞ e e +1


e2x − 1
= lim X
x→−∞ e2x + 1
0−1
= = −1 X
0+1
164 2012–2020 Past Test and Examination Questions Booklet Mathematics I (Major)

Question 5 [8 marks]

1) Give the statement of the Sandwich or Squeeze or Pinched Theorem. (3)


2) Use the Sandwich Theorem to evaluate (3)

lim sin θ.
θ→0

HINT: If θ → 0, then 0 < | sin θ | < | θ |.


3) Use your answer in 2) to evaluate (2)

lim cos θ.
θ→0

Answer 5

1) If f (x) ≤ g(x) ≤ h(x) when x is near a (except possibly at a) and lim f (x) = L = lim h(x), then lim g(x) = L.
x→a x→a x→a
XXX

2) As θ → 0,
0 < | sin θ | < | θ |. X
Since lim 0 = 0 and lim θ = 0 = lim | θ |, by the Sandwich Theorem, lim | sin θ | = 0. X Consequently,
θ→0 θ→0 θ→0 θ→0

lim sin θ = 0. X
θ→0

3)
p
lim cos θ = lim 1 − sin2 θ X
θ→0 θ→0

= 1 − 02 , using 2) (X)
=1 (X)

Total: [40] marks


Mathematics I (Major) 2012–2020 Past Test and Examination Questions Booklet 165

4.9.2 Calculus Solutions March (E diagonal) 2020

Question 1 [6 marks]

The graph of the function y = f (x) is given below.

−1 1 2 3 x
−1

−2

Using this graph of f (x), which of the following statements are true, and which are false?
1) lim f (x) does not exist. FALSE X (1)
x→2
2) lim f (x) = 2. FALSE X (1)
x→2
3) lim f (x) does not exist. TRUE X (1)
x→1
4) lim f (x) = 0. TRUE X (1)
x→0
5) lim f (x) = 0. FALSE X (1)
x→2
6) lim f (x) does not exist. FALSE X (1)
x→−1
166 2012–2020 Past Test and Examination Questions Booklet Mathematics I (Major)

Question 2 [6 marks]

 3−x if x < 2,


Let f (x) = 2 if x = 2,
 x
if x > 2.

2

f (x)

y=3−x

3 x
y= 2

0 x
−1 1 2 3 4

1) Find lim+ f (x) and lim− f (x). (2)


x→2 x→2
2) Does lim f (x) exist? If so, what is it? If not, why not? (1)
x→2
3) Find lim − f (x) and lim + f (x). (2)
x→−1 x→−1
4) Does lim f (x) exist? If so, what is it? If not, why not? (1)
x→−1

Answer 2
1) lim+ f (x) = 1 and lim− f (x) = 1. X X
x→2 x→2
2) lim f (x) = 1. X
x→2
3) lim − f (x) = 4 and lim + f (x) = 4. X X
x→−1 x→−1
4) lim f (x) = 4. X
x→−1
Mathematics I (Major) 2012–2020 Past Test and Examination Questions Booklet 167

Question 3 [11 marks]

Use limits to determine the equations for all vertical and horizontal asymptotes. If there is no vertical asymptote
or horizontal asymptote, then write so. Otherwise, identify them.
x+1
1) f (x) = (5)
2−x

x2 + 3
2) g(x) = (6)
x−1
Answer 3
x+1 3 x+1 3
1) lim− = + = ∞ or lim+ = − = −∞. X
x→2 2 − x 0 x→2 2 − x 0
Thus, the line x = 2 is a vertical asymptote. X
x+1 x 1 1
x+1 x x + x 1+ x 1+0
lim = lim 2−x
= lim = lim = = −1. XX
x→±∞ 2 − x x→±∞ x→±∞ 2 − x x→±∞ 2 −1 0−1
x x x x

Thus, the line y = −1 is a horizontal asymptote. X


√ √
x2 + 3 2 x2 + 3 2
2) lim− = − = −∞ or lim+ = + = ∞. X
x→1 x−1 0 x→1 x−1 0
Thus, the line x = 1 is a vertical asymptote. X
√ √ »
x2
»
3 √
x2 + 3
x2 +3
x x2
+ x32 1+ x2 1+0 √
lim = lim x−1
= lim x 1
= lim 1
= =1 X, where x2 = |x| = x,
x→∞ x − 1 1− 1−0
x − x
x→∞ x→∞ x→∞
x x
since x > 0.

Thus, the line y = 1 is a horizontal asymptote. X


√ √
x2 +3
»
x2 3
»
3
x2 + 3 x
− x2
+ x2
− 1+ x2 √
lim = lim x−1
= lim x 1
= lim 1
= −1 X, where x2 = |x| = −x,
x→−∞ x − 1 x→−∞ x→−∞ − x→−∞ 1+
x x x x
since x < 0.

Thus, the line y = −1 is a horizontal asymptote. X


√ √ »
3
»
3
»
3
»
3
x2 + 3 x2 1+ x2
|x| 1+ x2
±x 1+ x2
±1 1+ x2
Or, lim = lim = lim = lim = lim =
x→±∞ x − 1 x→±∞ x x→±∞ x x→±∞ x x→±∞ 1
lim ±1 = ±1.
x→±∞
168 2012–2020 Past Test and Examination Questions Booklet Mathematics I (Major)

Question 4 [9 marks]

Evaluate the limit, if it exists. If not, justify.


x2 − 1
1) lim √ √ (4)
x→1 2x + 1 − 3
2x
2) lim (2)
x→−∞ 3 x
|x + 1|
3) lim (3)
x→−1 x + 1

Answer 4

1)
2
Ä√ √ ä
x −1 2 x − 1 2x + 1 + 3
lim √ √ = lim Ä √ √ äÄ√ √ ä X
x→1 2x + 1 − 3 x→1 2x + 1 − 3 2x + 1 + 3
Ä√ √ ä
x2 − 1 2x + 1 + 3
= lim X
x→1 2x + 1Ä− 3
√ √ ä
(x − 1)(x + 1) 2x + 1 + 3
= lim X
x→1 2(x − 1)
Ä√ √ ä
(x + 1) 2x + 1 + 3
= lim , since x , 1 (X)
x→1
Ä√ 2
√ ä
(1 + 1) 2.1 + 1 + 3 √
= = 2 3 (X)
2
2x 2 x 2
Å ã
2) lim x = lim = ∞, since 0 < < 1. X X
x→−∞ 3 x→−∞ 3 3
|x + 1| −(x + 1)
3) lim − = lim − = lim − −1 = −1, since x < −1. X
x→−1 x+1 x→−1 x+1 x→−1

|x + 1| x+1
lim = lim + = lim + 1 = 1, since x > −1. X
x→−1+ x+1 x→−1 x + 1 x→−1

|x + 1|
Hence, lim does not exist. X
x→−1 x+1
Mathematics I (Major) 2012–2020 Past Test and Examination Questions Booklet 169

Question 5 [8 marks]

1) Give the statement of the Sandwich or Squeeze or Pinched Theorem. (2)

2) Consider 0 < θ < π


2 and sin θ < θ < tan θ. Use the Sandwich Theorem and lim+ cos θ = 1 to prove that (4)
θ→0

sin θ
lim = 1.
θ→0+ θ

3) Deduce that (2)

1 − cos2 θ
lim+ = 0.
θ→0 θ

Answer 5

1) If f (x) ≤ g(x) ≤ h(x) when x is near a (except possibly at a) and lim f (x) = L = lim h(x), then lim g(x) = L.
x→a x→a x→a
(X) (X) X

2) For 0 < θ < π2 ,

sin θ
sin θ < θ < tan θ =⇒ sin θ < θ < (X)
cos θ
θ 1
=⇒ 1 < < , since cos θ > 0 X
sin θ cos θ
sin θ
=⇒ 1 > > cos θ (X)
θ
sin θ
=⇒ cos θ < < 1.
θ
Since lim+ cos θ = 1 and lim+ 1 = 1, using the Sandwich Theorem, X
θ→0 θ→0

sin θ
lim+ = 1. X
θ→0 θ

1 − cos2 θ sin2 θ sin θ sin θ


3) lim+ = lim+ = lim+ = 1.0 = 0, using 2). (X) (X) X
θ→0 θ θ→0 θ θ→0 θ 1

Total: [40] marks


170 2012–2020 Past Test and Examination Questions Booklet Mathematics I (Major)

4.9.3 Calculus Solutions May 2020

Question 1 : [ 6 marks ]

x when x < −2
®
For what value of b is the function g(x) = 2
bx when x ≥ −2
continuous at every x?

1
A. b=
2
1
B. b=−
2

C. b = −2

D. b=2

E. None of these.

Answer 1 : B.

g(−2) = b(−2)2 = 4b, lim − g(x) = lim− x = −2, and lim + g(x) = lim + bx2 = 4b .
x→−2 −2 x→−2 x→−2

1
Thus, g(x) is continuous at x = −2 if 4b = −2, that is, if b = − .
2
1
Hence, g(x) is continuous at every x if b = − .
2

Question 2 : [ 6 marks ]

3x when x<0
®
The function f (x) = 3
x when x≥0
is differentiable at x = 0. True or false?

Answer 2 : False .

f (x) − f (0) 3x − 0 3x
f−0 (0) = lim− = lim− = lim− = lim− 3 = 3, and
x→0 x−0 x→0 x x→0 x x→0

f (x) − f (0) x3 − 0
f+0 (0) = lim+ = lim+ = lim+ x2 = 0.
x→0 x−0 x→0 x x→0

=⇒ f−0 (0) , f+0 (0) =⇒ f is not differentiable at x = 0 .


Mathematics I (Major) 2012–2020 Past Test and Examination Questions Booklet 171

Question 3 : [ 7 marks ]
Z π
2
e−x sin x dx =
0
π
A. e− 2

1 π
B. 1 − e− 2

2

1 π
C. e− 2 − 1

2

1 π
D. e− 2 + 1

2

E. None of these.

Answer 3 : B.
Z
I= e−x sin x dx, u = e−x =⇒ du = −e−x dx, dv = sin x dx =⇒ v = − cos x
Z
I = −e −x
cos x − e−x cos x dx .
Z
I1 = e−x cos x dx, u = e−x =⇒ du = −e−x dx, dv = cos x dx =⇒ v = sin x
Z
I1 = e−x sin x + e−x sin x dx .
Z
=⇒ I = −e −x
cos x − e −x
sin x − e−x sin x dx = −e−x (cos x + sin x) − I

1
=⇒ 2I = −e−x (cos x + sin x) + C =⇒ I = − e−x (cos x + sin x) + C
2
Z π ò π2 ï
1 1 π π π 1
ï ò ï ò
2
e−x sin x dx = − e−x (cos x + sin x) = − e− 2 (cos + sin ) − − e0 (cos 0 + sin 0)
0 2 0 2 2 2 2
1 π 1 1 π
= − e− 2 + = 1 − e− 2 .
2 2 2
172 2012–2020 Past Test and Examination Questions Booklet Mathematics I (Major)

Question 4 : [ 7 marks ]

Z e2
(ln x)2
dx =
e x
3
A. 2

7
B. 3

8
C. 3

D. 3

E. None of these.

Answer 4 : B .

1
u = ln x =⇒ du = dx, x = e =⇒ u = 1, x = e2 =⇒ u = 2
x
Z e2 Z 2 ò2
(ln x)2 u3 8 1 7
ï
2
dx = u du = = − =
e x 1 3 1 3 3 3

Question 5 : [ 7 marks ]
π π
Find the equation of the tangent line at the point , to the curve
4 4
π
x sin y + y cos y = √
2 2
π
A. y= x+ 2

π
B. y = −x − 2

π
C. y= x− 2

π
D. y = −x + 2

E. None of these.
Mathematics I (Major) 2012–2020 Past Test and Examination Questions Booklet 173

Answer 5 : D .

d d π dy dy dy
Å ã
(x sin y + y cos y) = √ =⇒ sin y + x cos y + cos y − y sin y =0
dx dx 2 2 dx dx dx
√ √ √ √ √ √
π π 2 π 2 dy 2 dy π 2 dy 2 2 dy
x = and y = =⇒ + + − = 0 =⇒ + =0
4 4 2 4 2 dx 2 dx 4 2 dx 2 2 dx
dy
=⇒ m = = −1 .
dx
π  π π  π
The equation of the tangent line is : y − =m x− =⇒ y − = −1 x −
4 4 4 4
π
=⇒ y = −x +
2

Question 6 : [ 7 marks ]

Using the linear approximation, which one of the following value approximates the value of f (t) = t2 et at t = 1.001
?

A. 1.003 e

B. 0.003 e

C. 0.997 e

D. 0.004 e

E. None of these.

Answer 6 : A .

f (1 + 0, 001) ≈ f (1) + f 0 (1) (0, 001), f (t) = t2 et =⇒ f (1) = e

f (t) = t2 et =⇒ f 0 (t) = 2tet + t2 et =⇒ f 0 (1) = 3e

Hence, f (1, 001) ≈ e + 3e (0, 001) = e (1 + 0, 003) = 1, 003 e

Total: 40 marks
174 2012–2020 Past Test and Examination Questions Booklet Mathematics I (Major)

4.9.4 Calculus Solutions June 2020

Question 1 [2 marks]

x2 − 4
Given the function f (x) = .
x2 − 2

The x-intercepts and y-intersept of f are:

A) x = ±2 and y = −2

B) x = 2 and y = −2

C) x = −2 and y = 2

D) x = ±2 and y = 2

E) None above.

Answer 1 : D.

x-intercepts : f (x) = 0 =⇒ x2 − 4 = 0 =⇒ x = ±2 .

y-intercept: y = f (0) =⇒ y = 2 .

Question 2 [2 marks]

x2 − 4
Given the function f (x) = .
x2 − 2

The asymptotes of f are:



A) x = ± 2 and y = 1

B) x = 2 and y = 1

C) x = − 2 and y = 1

D) x = ± 2 and y = 2

E) None above.
Mathematics I (Major) 2012–2020 Past Test and Examination Questions Booklet 175

Answer 2 : A.

Vertical asymptote: x2 − 2 = 0 =⇒ x = ± 2.

Horizontal asymptote: lim f (x) = 1 =⇒ y = 1.


x→±∞

Question 3 [4 marks]

x2 − 4
Given the function f (x) = .
x2 − 2

The intervals of increase and decrease of f are respectively:

A) (0, 2) ∪ (2, ∞) and (−∞, −2) ∪ (−2, 0)


Ä √ ä Ä√ ä
B) 0, 2 ∪ 2, ∞ and (−∞, −2) ∪ (−2, 0)
Ä √ ä Ä √ ä
C) (0, 2) ∪ (2, ∞) and −∞, − 2 ∪ − 2, 0
Ä √ ä Ä√ ä Ä √ ä Ä √ ä
D) 0, 2 ∪ 2, ∞ and −∞, − 2 ∪ − 2, 0

E) None above.

Answer 3 : D.

2x(x2 − 2) − 2x(x2 − 4) 4x
f 0 (x) = = 2
(x2 − 2)2 (x − 2)2

Critical x-value: f 0 (x) = 0 =⇒ 4x = 0 =⇒ x = 0


.

Critical x-value: f 0 (x) = UND =⇒ (x2 − 2)2 = 0 =⇒ x = ± 2
.

√ √
x − 2 0 2

f 0 (x) – – k – – 0 + + k + +

f (x) & & k & & 2 % % k % %


V.A. Min V.A.

Ä √ ä Ä √ ä Ä √ ä Ä√ ä
=⇒ f decreases on −∞, − 2 ∪ − 2, 0 whilst it increases on 0, 2 ∪ 2, ∞ .

Question 4 [2 marks]

x2 − 4
Given the function f (x) = .
x2 − 2

Classify the critical point of f :


176 2012–2020 Past Test and Examination Questions Booklet Mathematics I (Major)

A) The point (0, 2) is a local maximum.

B) The point (0, 2) is a local minimum.

C) The point (2, 0) is a local maximum.

D) The point (2, 0) is a local minimum.

E) None above.

Answer 4 : B.

From Answer 3, the point (0, 2) is a local minimum.

Question 5 [4 marks]

x2 − 4
Given the function f (x) = .
x2 − 2

The intervals of concavity of f :


Ä √ ä Ä√ ä Ä √ √ ä
A) f is concave up on −∞, − 2 ∪ 2, ∞ and concave down on − 2, 2
Ä √ √ ä Ä√ ä Ä √ ä
B) f is concave up on − 2, 2 ∪ 2, ∞ and concave down on −∞, − 2
Ä √ ä Ä √ √ ä Ä√ ä
C) f is concave up on −∞, − 2 ∪ − 2, 2 and concave down on 2, ∞
Ä √ √ ä Ä √ ä Ä√ ä
D) f is concave up on − 2, 2 and concave down on −∞, − 2 ∪ 2, ∞

E) None above.
Mathematics I (Major) 2012–2020 Past Test and Examination Questions Booklet 177

Answer 5 : D.

4(x2 − 2)2 − 16x2 (x2 − 2) 4(x2 − 2) − 16x2 −12x2 − 8


f 00 (x) = = =
(x2 − 2)4 (x2 − 2)3 (x2 − 2)3

√ √
x − 2 0 2

f 00 (x) – – k + + + + + k – –

f (x) CD CD k CU CU 2 CU CU k CD CD
V.A. Min V.A.
Ä √ ä Ä√ ä Ä √ √ ä
=⇒ f is concave down on −∞, − 2 ∪ 2, ∞ whilst it is concave up on − 2, 2 .

Question 6 [8 marks]
A rectangle has its base on the x-axis and its upper two vertices on the parabola

y = 12 − x2 .

The largest area the rectangle can have is A = . . . (units squared) (4)

and its dimensions are such that x = . . . (units) and y = . . . (units). (4)

Answer 6:

12

y = 12 − x2

√ √
−2 3 −x 0 x 2 3

√ √
A = 2xy =⇒ A(x) = 2x(12 − x2 ) = 24x − 2x3 , where −2 3 ≤ x ≤ 2 3 .

dA î √ √ ó
= 24 − 6x2 = 0 =⇒ x = 2 ∈ −2 3, 2 3 .
dx
178 2012–2020 Past Test and Examination Questions Booklet Mathematics I (Major)


x = −2 3 =⇒ A = 0 .

x = 2 =⇒ A = 32 .

x = 2 3 =⇒ A = 0 .

The largest area the triangle can have is A = 32 (units squared).

Its dimensions are such that x = 2 (units) and y = 8 (units).

Question 7 [4 marks]

Let y = (ln x)ln x .

dy
=
dx
1
A) (ln x)ln x (2 ln x + 1)
x
1
B) (ln x)ln x [(ln(ln x)) + 1]
x
1
C) (ln x)ln x (ln x)2 + 1
 
x
1
D) (ln x)ln x (ln x + 1)
x

E) None above.

Answer 7: B.

y = (ln x)ln x =⇒ ln y = ln(ln x)ln x =⇒ ln y = (ln x)(ln(ln x)).

d d
=⇒ (ln y) = [(ln x)(ln(ln x))]
dx dx
1 dy 1 1 1
=⇒ = (ln(ln x)) + (ln x)
y dx x ln x x
1 dy 1 1
=⇒ = (ln(ln x)) +
y dx x x
dy 1 1
ï ò
=⇒ = y (ln(ln x)) +
dx x x
dy 1 1
ï ò
=⇒ = (ln x)ln x (ln(ln x)) +
dx x x
dy 1
=⇒ = (ln x)ln x [ln(ln x) + 1] .
dx x

Question 8 [6 marks]

lim (ln x − ln(sin x)) =


x→0+
Mathematics I (Major) 2012–2020 Past Test and Examination Questions Booklet 179

A) 1

B) 0

C) −∞

D) ∞

E) None above.

Answer 8: B.

lim (ln x − ln(sin x)) = ln 0+ − ln(sin 0+ ) = −∞ − ln 0+ = −∞ + ∞ , ind. form.


x→0+
 x   x 
Å
1
ã
lim+ (ln x − ln(sin x)) = lim+ ln = ln lim+ = ln lim+ = ln 1 = 0 ,
x→0 x→0 sin x x→0 sin x x→0 cos x

since ln is continuous over its domain.


180 2012–2020 Past Test and Examination Questions Booklet Mathematics I (Major)

Question 9 [4 marks]

Given g(x, y) = x sin y + y sin x + xy.

∂2 g
=
∂y∂x

A) cos y + sin x + 1

B) sin y + cos x + 1

C) cos(x + y) + 1

D) sin y + sin x + 1

E) None above.

Answer 9: E.

∂g ∂2 g
g(x, y) = x sin y + y sin x + xy =⇒ = sin y + y cos x + y =⇒ = cos y + cos x + 1 .
∂x ∂y∂x

Similarly,

∂g ∂2 g
g(x, y) = x sin y + y sin x + xy =⇒ = x cos y + sin x + x =⇒ = cos y + cos x + 1 .
∂y ∂x∂y

Question 10 [4 marks]

A surface area of a cube increases at the rate of 72 cm2 /sec. At what rate is the cube’s volume changing when the
edge length is x = 3 cm ?

A) 72 cm3 /sec

B) 2 cm3 /sec

C) 36 cm3 /sec

D) 54 cm3 /sec

E) None above.

Answer 10: D.

dA dx dx dx dx
A = 6x2 =⇒ = 12x =⇒ 72 = 12(3) =⇒ 72 = 36 =⇒ = 2 cm/sec
dt dt dt dt dt
dV dx dV
V = x3 =⇒ = 3x2 =⇒ = 3(3)2 (2) = 54 m3 /sec .
dt dt dt

Total: 40 marks
Mathematics I (Major) 2012–2020 Past Test and Examination Questions Booklet 181

4.9.5 Calculus Solutions August 2020

Question 1 [4 marks]
3
Find the area, denoted by A, of the region bounded by the curves φ(x) = 8x and ψ(x) = 2x. Then 4A = . . .
Answer 1 : 1

Question 2 [6 marks]
A manufacturer drills a hole through the center of a metal sphere of radius 15 cm. The hole has a radius of 9 cm.
What is the volume of the resulting metal ring? Volume = 16π . . . cm3 .
(Type in a value so that the product of 16π multiplied by the value that you type in, gives the volume of the
resulting solid in cm3 ).
Answer 2 : 144

Question 3 [3 marks]
The base of a solid is the disc centred at (1, 0) with radius 2 and the cross-sections (parallel to the x-axis) are
equilateral triangles, where the length of each side is the same as the distance from one end of the √ circle to the
other at each fixed x value. Is it true or is it false, that the area of the largest of these triangles is 4 3?
Answer 3 : True

Question 4 [4 marks]
The base of a solid is the disc centred at (1, 0) with radius 2 and the cross-sections (parallel to the x-axis) are
equilateral triangles, where the length of each side is the same as the distance from one end of the circle to the
other at each fixed x value. The volume is given by,
Z 2
√ » 2
A) 3 4 − y2 + 1 dy
−2
Z √
3 » 2
B) 2 4 − y2 + 1 dy
−2
Z √
. 3 » 2
C) 2 4 − y2 dy
−2
Z 2
√ » 2
D) 3 4 − y2 dy
−2
Z √ 3
√ » 2
E) 3 4 − y2 dy
−2

Answer 4: D

Question 5 [4 marks]
2
Find the volume of the solid generated by rotating the region enclosed by y = x and y = x about the yaxis. The
value of 6 multiplied by volume divided by π is . . ..
Answer 5: 1

Question 6 [3 marks]
2
Find the volume of the solid generated by rotating the region enclosed by y = x and y = x about the xaxis. The
value of 30 multiplied by volume divided by π is . . ..
Answer 6: 1
182 2012–2020 Past Test and Examination Questions Booklet Mathematics I (Major)

Question 7 [2 marks]
Is the difference of the volume of the solids generated in Question 5 and 6 zero? True or False?
Answer 7: False

Question 8 [4 marks]
Z x2
Given f (x) = t2 dt + x − 5 .
0

0
The value f (2) is . . . .
Answer 8: 65
Mathematics I (Major) 2012–2020 Past Test and Examination Questions Booklet 183

4.9.6 Calculus Solutions November Exam 2020

Question 1 [10 marks]


Which FIVE of the following statements are FALSE ?

cos(nπ) ∞
ß ™
A. The sequence diverges
n n=1

X (−1)n 9n x4n
B. cos −3x2 =

n=0
(2n)!
Z 0
d 1
C. cos(t2 ) dt = − √ sin x
dx x

2 x
Z 1
1
D. The integral dx converges
0 x
X∞
e
E. e−n =
n=0
e−1
ß ™
F. The sequence cos(nπ) diverges

n+1
ß ™
G. The sequence converges
n
H. Every decreasing sequence is convergent
X∞
1
I. The series converges
n=1
5n

X cos2 n
J. The series converges
n=1
n3/2

Answer 1
cos(nπ) 1 cos(nπ)
A. lim = lim = 0 =⇒ lim = 0 ∈ R.
n→∞ n n→∞ n n→∞ n
Hence, the sequence converges.

2n ∞
2
 X (−1)n −3x2 X (−1)n 9n x4n
B. cos −3x = = .
n=0
(2n)! n=0
(2n)!
Z 0
d √ 2 √ 0 1
C. cos(t2 ) dt = cos(02 ) 00 − cos x x = − √ cos x .
dx √ x 2 x
Z 1 Z 1 ò1
1 1
ï
D. dx = lim+ dx = lim+ ln |x| = lim+ (− ln s) = −(−∞) = ∞ . Diverges.
0 x s→0 s x s→0 s s→0
∞ ∞
X X 1 n 1 e
Å ã
E. e−n = = 1
= .
n=0 n=0
e 1− e e − 1
F. an = cos(nπ) = (−1)n =⇒ {an } diverges.
n+1
G. lim = 1 ∈ R. Hence the sequence converges.
n→∞ n
H. Every decreasing sequence and bounded below is convergent
X∞ ∞
1 1X 1
I. The series = is a scalar multiple of the harmonic series. Hence it diverges.
n=1
5n 5 n=1 m
184 2012–2020 Past Test and Examination Questions Booklet Mathematics I (Major)

X 1 ∞
cos2 n 1
J. The series 3/2
≤ 3/2 , for all n ≥ 1, and the p-series converges since p = 3/2 > 1. Hence the
n n n=1
n3/2

X cos2 n
series converges as well.
n=1
n3/2

Question 2 [10 marks]


Z 2
x
√ dx =
−1 10 + 2x + x2
Ä √ ä Ä √ ä
A. 3 1 − 2 − ln 1 + 2
Ä√ ä Ä √ ä
B. 3 2 − 1 − ln 1 + 2
Ä √ ä Ä√ ä
C. ln 1 + 2 − 3 2−1
Ä √ ä Ä√ ä
D. ln 1 + 2 + 3 2−1
E. None of these

Answer 2
Let x + 1 = 3 tan θ. Then x = −1 =⇒ θ = 0 and x = 2 =⇒ θ = π/4
Z 2 Z 2
x x
√ dx = dx
9 + (x + 1)2
p
−1 10 + 2x + x2 −1

Z π/4
3 tan θ − 1
= 3 sec2 θ dθ
0 3 sec θ
Z π/4
= (3 tan θ sec θ − sec θ) dθ
0

ï ò π/4
= 3 sec θ − ln (tan θ + sec θ)
0

Ä√ ä Ä √ ä
= 3 2 − 1 − ln 1 + 2 .

Question 3 [10 marks]


Z 2
x2 − x + 4
2
dx =
1 (x + 1)(x + 1)

27 π
Å ã
A. ln + arctan 2 −
20 4
π 27
Å ã
B. arctan 2 − − ln
4 20
π 27
Å ã
C. − arctan 2 − ln
4 20
π 20
Å ã
D. − arctan 2 − ln
4 27
E. None of these

Answer 3
Mathematics I (Major) 2012–2020 Past Test and Examination Questions Booklet 185

Z 2 Z 2 Z 2 Z 2
x2 − x + 4 3 2x 1
dx = dx − dx + dx
1 (x + 1)(x2 + 1) 1 x+1 1 x2 + 1 1 x2 + 1
ï ò2
2
3 ln(x + 1) − ln x + 1 + arctan x

=
1

27 π
Å ã
= ln + arctan 2 − .
20 4
Question 4 [5 marks]
Z π/4
sin 3x cos 5x dx =
0

1
A.
2
1
B. −
2
1
C.
4
1
D. −
4
E. None of these
Answer 4
Z π/4 Z π/4 ò π/4
1 1 1 1
ï
sin 3x cos 5x dx = (sin 8x − sin 2x) dx = − cos 8x + cos 2x =− .
0 0 2 16 4 0 4

Question 5 [5 marks]
Using the Lower Riemann Sum, the approximate area under the hyperbola xy = 1 between x = 1 and x = 2
divided into n rectangles of equal width is
n
X 1
A.
j=1
n+ j
n
X n
B.
j=1
n+ j
n
X n
C.
j=1
n+ j−1
n
X 1
D.
j=1
n+ j−1
E. None of these
Answer 5
2−1 1
Partition [1, 2] into n subintervals of equal length ∆x = = . Then the area of the jth inscribed rectangle is
n n
j 1 n 1 1
R j = f (m j )∆x where m j = 1 + and f (m j ) = j
. That is, R j = · = . Hence, the approximate
n 1+ n n + j n n + j
X n X n
1
area is Rj = .
j=1 j=1
n + j
Question 6 [10 marks]
Z ∞
4
2
dx =
0 x +2
186 2012–2020 Past Test and Examination Questions Booklet Mathematics I (Major)

π
A. −√
2

B. 2π
π
C. √
2

D. − 2π
E. None of these

Answer 6
Z ∞ Z t ãò t
4 4 4 x
ï Å
dx = lim dx = lim √ arctan √
0 x2 + 2 t→∞ 0 x2 + 2 t→∞ 2 2 0

4
Å
t
ã √
= lim √ arctan √ = 2π.
t→∞ 2 2
Question 7 [10 marks]

X (−1)n (x + 2)n
The interval of convergence of the series is
n=1
n

A. (−3, −1]
B. [−3, −1]
C. (−3, −1)
D. [−3, −1)
E. None of these

Answer 7

(−1)n+1 (x + 2)n+1 n n
lim · = lim | x + 2| = | x + 2|.
n→∞ n+1 (−1)n (x + 2)n n→∞ n + 1
The series converges for | x + 2 | < 1, that is, for −3 < x < −1 .
X ∞
1
If x = −3, then the series is the harmonic series which diverges.
n=1
n

X (−1)n
If x = −1, then the series is which converges by the AST.
n=1
n

X (−1)n (x + 2)n
Hence, the interval of convergence of the series is (−3, −1] .
n=1
n
Question 8 [10 marks]
Which FIVE of the following statements are TRUE ?
∞ Å
X 4n − 3 n
ã
A. The series converges
n=1
3n − 4
X∞
cos(nπ)
B. The series diverges
n=1
n
X∞ Å 3 ãn
n n −1
C. The series (−1) converges
n=1
1 + 3n3
1 1 1 1 1 1 1
D. 1+ + + + + + + + ··· = 2
2 4 8 16 32 64 128
Mathematics I (Major) 2012–2020 Past Test and Examination Questions Booklet 187

X ∞
1
E. 3
= (−1)n x3n for x ∈ (−1, 1)
1+x n=0

X 1
F. The series diverges
n=2 n (ln n)2

X 1
G. The series diverges
n=1
2 + 3n

X 1
H. The series (−1)n+1 converges
n=1
n5n

X n!
I. The radius of convergence of the series xn is R = 1
n=1
3 · 6 · 9 · · · 3n

X 1
J. The series 2
converges
n=1
n + 2n + 3

Answer 8
n
n 4n − 3 4n − 3 4
A. lim = lim = > 1 . Hence the series diverges.
n→∞ 3n − 4 n→∞ 3n − 4 3

X ∞
X
cos(nπ) (−1)n
B. = .
n=1
n n=1
n
1
Let bn = . Then bn > bn+1 for all n ≥ 1 and bn → 0 as n → ∞. Hence the series converges.
n
n −1 n n3 − 1
Å 3
1
ã
n
C. lim (−1) n = lim = < 1. Hence the series converges.
n→∞ 1 + 3n3 n→∞ 1 + 3n3 3
X∞ Å ãn
1 1 1 1 1 1 1 1 1
D. 1+ + + + + + + + ··· = = 1
= 2.
2 4 8 16 32 64 128 n=0
2 1 − 2
X X ∞ ∞
1 1 3 n
E. = = (−x ) = (−1)n x3n for −x3 < 1, that is, for x ∈ (−1, 1) .
1 + x3 1 − (−x3 ) n=0 n=0
1
F. Let f (x) = is continuous, positive, decreasing on [2, ∞] .
x (ln x)2
Z ∞
1 1 t 1 1 1 1
ï ò Å ã
Thus dx = lim − = lim − + =0+ = .
2 x (ln x)2 t→∞ ln x 2 t→∞ ln t ln 2 ln 2 ln 2
X ∞
1
Hence, the series converges.
n=2 n (ln n)2
X∞ X ∞ Å ãn
1 1 1 1
G. n
≤ n for all n ≥ 1 and the geometric series n
= converges.
2+3 3 n=1
3 n=1
3
X ∞
1
Hence the series converges as well.
n=1
2 + 3n
X ∞
1 1
H. Let bn = n
. Then bn > bn+1 , for all n ≥ 1, and lim bn = 0. Hence the series (−1)n+1 n converges.
n5 n→∞
n=1
n5
(n + 1)!xn+1 3 · 6 · 9 · · · (3n) |x| |x|
I. · = → as n → ∞.
3 · 6 · 9 · · · (3n)3(n + 1) n!xn 3 3
|x|
Thus series converges if < 1, that is, |x| < 3 .
3
Hence the radius of convergence is R = 3.
188 2012–2020 Past Test and Examination Questions Booklet Mathematics I (Major)

X∞
1 1 1
J. For all n ≥ 1, 2
≤ 2
, for all n ≥ 1. and the p-series converges. Hence the series
n + 2n + 3 n n=1
n2

X 1
converges.
n=1
n2 + 2n + 3

Question 9 [10 marks]


The general solution of the differential equation (2x + y)dx + (x − 2y)dy = 0 is
A. x2 + xy − y2 = C
B. x2 − xy + y2 = C
C. x2 + xy + y2 = C
D. x2 + 2xy − y2 = C
E. None of these
Answer 9
Let M = 2x + y and N = x − 2y. Then My = 1 = N x . That is, the ODE is exact. Thus, there is f (x, y) such that
f x = M and fy = N. Then
f x = M =⇒ f x = 2x + y
Z
=⇒ f (x, y) = (2x + y) dx
=⇒ f (x, y) = x2 + xy + C(y)
=⇒ N = fy = x + C 0 (y)
=⇒ x − 2y = x + C 0 (y)
=⇒ C 0 (y) = −2y
Z
=⇒ C(y) = − 2y dy = −y2 + K .

Thus, f (x, y) = x2 + xy − y2 + K. Hence, the general solution to the ODE is


x2 + xy − y2 = C .

Question 10 [10 marks]


dy cos x 2y
The particular solution of the differential equation = 2 − under the initial conditions x = π and y = 1 is
dx x x
sin x − π2
A. y=
x2
2
π − sin x
B. y=
x2
sin x + π2
C. y=
x2
sin x + π2
D. y=−
x2
E. None of these
Answer 10
dy cos x 2y dy 2y cos x R 2
= 2 − =⇒ + = 2 =⇒ µ = e x dx = e2 ln x = x2 .
dx x x dx Zx x
d
x2 y = cos x =⇒ x2 y = cos x dx =⇒ x2 y = sin x + C .

dx
If x = π and y = 1, then C = π2 . Hence the particular solution of the ODE if x = π and y = 1 is
sin x + π2
y= .
x2
TOTAL : 90 marks.

You might also like